Sie sind auf Seite 1von 240

Neuropathology

Review
Richard A. Prayson, MD

Humana Press
Contents

NEUROPATHOLOGY REVIEW
2 Contents
Contents

NEUROPATHOLOGY
REVIEW

By
RICHARD A. PRAYSON, MD
Department of Anatomic Pathology
Cleveland Clinic Foundation, Cleveland, OH

HUMANA PRESS
TOTOWA, NEW JERSEY
4 Contents

2001 Humana Press Inc.


999 Riverview Drive, Suite 208
Totowa, New Jersey 07512

For additional copies, pricing for bulk purchases, and/or information about other Humana titles, contact Humana at the above address or at any of
the following numbers: Tel.: 973-256-1699; Fax: 973-256-8341; E-mail:humana@humanapr.com; Website: http://humanapress.com

All rights reserved.

No part of this book may be reproduced, stored in a retrieval system, or transmitted in any form or by any means, electronic, mechanical, photocopying,
microfilming, recording, or otherwise without written permission from the Publisher.

Due diligence has been taken by the publishers, editors, and authors of this book to assure the accuracy of the information published and to describe
generally accepted practices. The contributors herein have carefully checked to ensure that the drug selections and dosages set forth in this text are
accurate and in accord with the standards accepted at the time of publication. Notwithstanding, as new research, changes in government regulations, and
knowledge from clinical experience relating to drug therapy and drug reactions constantly occurs, the reader is advised to check the product information
provided by the manufacturer of each drug for any change in dosages or for additional warnings and contraindications. This is of utmost importance
when the recommended drug herein is a new or infrequently used drug. It is the responsibility of the treating physician to determine dosages and treatment
strategies for individual patients. Further it is the responsibility of the health care provider to ascertain the Food and Drug Administration status of each
drug or device used in their clinical practice. The publisher, editors, and authors are not responsible for errors or omissions or for any consequences
from the application of the information presented in this book and make no warranty, express or implied, with respect to the contents in this publication.

Cover illustrations: Inset gross picture depicts a brain with lissencephaly. The background microscopic picture is of subacute sclerosing panencephalitis.
Artwork courtesy of Richard A. Prayson, MD.
Cover design by Patricia F. Cleary.

This publication is printed on acid-free paper.


ANSI Z39.48-1984 (American National Standards Institute) Permanence of Paper for Printed Library Materials.

Photocopy Authorization Policy:


Authorization to photocopy items for internal or personal use, or the internal or personal use of specific clients, is granted by Humana Press Inc.,
provided that the base fee of US $10.00 per copy, plus US $00.25 per page, is paid directly to the Copyright Clearance Center at 222 Rosewood Drive,
Danvers, MA 01923. For those organizations that have been granted a photocopy license from the CCC, a separate system of payment has been
arranged and is acceptable to Humana Press Inc. The fee code for users of the Transactional Reporting Service is: [1-58829-024-7/01 $10.00 + $00.25].

Printed in the United States of America. 10 9 8 7 6 5 4 3 2 1

Library of Congress Cataloging-in-Publication Data

Prayson, Richard A.
Neuropathology review / by Richard A. Prayson.
p. ;cm.
Includes bibliographical references and index.
ISBN 1-58829-024-7 (alk. paper)
1. Nervous systemDiseasesOutline, syllabi, etc. . 2. Nervous systemDiseasesExaminations, questions, etc. I. Title.
[DNLM: 1. Nervous System DiseasesOutlines. WL 18.2 P921n 2001]
RC347.P724 2001
616.80076dc21 2001016673
Contents
PREFACE
The scope of neuropathology continues to expand, as evidenced by increasing numbers of
multivolume and specialty texts that have been published in recent years. For those in the neuroscience
disciplines, the ever increasing amount of information one needs to assimilate and master can be challeng-
ing and even at times daunting.
Neuropathology Review attempts to summarize, in outline form, the essentials of neuropathology.
The objective is twofold: (1) to provide an overview of neuropathology, for those initially encountering
the discipline and, (2) to provide a framework for review for those preparing for in-service and board
examinations in the disciplines of neurology, neurosurgery, and pathology that require some knowledge
of neuropathology.
The text is divided into three main sections. The first part (Chapters 111) presents, in outline form,
basic information on the spectrum of neurologic-related disease. The second part (Chapters 12 and 13)
will present pertinent pictorial examples of a spectrum of neuropathologic conditions in a question format
with answers and brief explanation provided. The final section (Chapters 14 and 15) will include a series
of written questions provided with answers and explanations in order to review the material presented in
the first section of the text.
The author would like to thank Ms. Denise Egleton for her help in preparing this manuscript and to
acknowledge Dr. Caroline Abramovich for her review of the text.
Richard A. Prayson, MD

v
6 Contents
CONTENTS
Contents

Preface .................................................................................................................................... v
1 Normal Histology .............................................................................................................1
2 Vascular Lesions .............................................................................................................. 7
3 Tumors ............................................................................................................................ 15
4 Trauma ............................................................................................................................ 35
5 Congenital Malformations, Perinatal Disease, and Phacomatoses ............................... 41
6 Demyelinating and Dysmyelinating Diseases ............................................................... 51
7 Metabolic and Toxic Diseases ....................................................................................... 55
8 Degenerative Diseases ................................................................................................... 59
9 CNS Infection ................................................................................................................. 65
10 Skeletal Muscle .............................................................................................................. 75
11 Peripheral Nerve ............................................................................................................. 85
12 Figures with Questions................................................................................................... 91
13 Answers to Figures with Questions ............................................................................. 185
14 Written Self-Assessment Questions............................................................................. 193
15 Answers to Written Self-Assessment Questions ......................................................... 211
Bibliography ....................................................................................................................... 217
Index ................................................................................................................................... 219

vii
8 Contents
1 Normal Histology

I. Central Nervous System Synaptophysin immunostain positivemem-


brane protein of synaptic vesicles
A. Neurons
Lipofuscin (aging pigment, lipochrome)
5100 m in size
Variety of shapes: stellate, oval, round, pyra- Increases with age
midal, bipolar and unipolar Intracytoplasmic
Nucleus Light yellow-brown color on routine
Vesicular with prominent nucleolus hematoxylin and eosin staining
ScallopedPurkinje cells and pyramidal Hyaline (colloid inclusion)
cells in cortical layer V Cytoplasmic
Cytoplasm Endoplasmic reticulum dilated cisternae
Mitochondria prominent Marinesco bodies
Nissl substancegranular basophilic mate- Eosinophilic intranuclear inclusions
rial, rough endoplasmic reticulum May be multiple within one nucleus
Neurofilaments (60100 A, transport
Prominently seen in pigmented neurons
function)
Increases with age
May have pigments such as lipofuscin
and melanin A variety of other inclusions within neurons
may be seen in association with certain dis-
Cell processes
ease processes (e.g., Lewy bodies, tangles,
Axonconducts away from cell body, Hirano bodies, Pick bodies, granulovacuolar
arises from axon hillock which contains degeneration, viral inclusions, etc.)
no Nissl substance; axoplasm with mito- Ischemic changesred and dead
chondria, neurotubules, and neurofila- shrunken cell body with dark nucleus, indis-
ments tinguishable nucleolus, and eosinophilic cyto-
Dendriteconducts toward cell body, con- plasm.
tains Nissl substance Central chromatolysisrelated to axonal or
Pigmented (neuromelanin) neuronscoarse, retrograde degeneration, cytoplasmic swell-
dark brown granules ing with loss of Nissl substance, and periph-
eralization/flattening of the nucleus, may be
Substantia nigra (midbrain)
reversible to a point
Locus ceruleus (pons) Ferruginization of neurons (fossilized neu-
Dorsal motor nucleus of the vagus nerve rons)encrusted with calcium and/or iron,
(medulla) may be seen in old infarcts.

1
2 NEUROPATHOLOGY REVIEW

Neuronophagiaphagocytosis of cell body Alzheimer type II astrocyte


Axonal spheroidsfocal dilatation of axon Nuclear swelling and chromatin clearing
with neurofilaments and organelles, reaction
Prominent nucleolation
to axon injury, Purkinje cell axonal spher-
oids called torpedoes Cell cytoplasm not readily apparent
B. Astrocytes Prominently seen in globus pallidus in
Largest of glial cells hepatic diseases that produce elevated
Vesicular nucleus without nucleolus ammonia levels
Cytoplasm highlighted with GFAP (glial Viral inclusions in astrocytes
fibrillary acidic protein) antibody Cowdry Aintranuclear, eosinophilic, or
Processes often invest vessels basophilic viral inclusion; displaces the
nucleolus (e.g., subacute sclerosing
A variety of shapes
panencephalitis (SSPE), progressive multi-
Star-shaped fibrillary (white matter) and focal leukoencepholopathy (PML), herpes,
protoplasmic (gray matter) cells cytomegalovirus (CMV)
Pilocytic astrocytes of periventricular Cowdry Bintranuclear, eosinophilic
region and cerebellum viral inclusion; often multiple and smaller
Bergmann astrocytes of cerebellum than Cowdry A inclusions, generally do
not displace the nucleolus (e.g., polio)
Corpora amylacea
C. Oligodendrocytes
Laminated, basophilic polyglucosan
Most numerous cells in the central nervous
bodies
system (CNS)
Associated with astrocytic foot processes
Small cells with no cytoplasmic staining
Increase with age with hematoxylin and eosin
Frequently seen in subpial regions Thinner and fewer cell processes than astro-
cytes
Reactive astrocytes (gemistocytes)
Tend to arrange themselves around neurons
Cell hypertrophyincreased cell cyto-
(neuronal satellitosis)
plasm
Fried egg appearance is a delayed post-
Gliosis may be associated with increased formalin-fixation artifact (will not be seen at
cell number frozen section)
Rosenthal fiber Cells often arranged in small rows in the
Cytoplasmic inclusion white matter between myelinated fibers
Brightly eosinophilic, elongated structures Involved in the formation and maintenance
of myelin
Observed in a number of entities includ-
Only weak immunostaining with GFAP, posi-
ing gliosis, Alexanders disease, pilocytic
tive staining with S-100 protein antibody
astrocytoma
Creutzfeldt astrocyte May contain intranuclear inclusions in PML
and SSPE.
Reactive astrocytes with multiple small
D. Ependyma
nuclei
Epithelioid cells that line ventricular walls
Seen in demyelinative processes and central canal of the spinal cord
Alzheimer type I astrocyte Columnar/cuboidal cells with cilia
Large cells often with multiple nuclei or Cilia attached to cell body by blepharoplast
irregularly lobulated nuclei Ependyma-lined central canal is patent in
Seen in progressive multifocal leukoence- childhood but generally becomes obliterated
phalopathy at puberty
CHAPTER 1 / NORMAL HISTOLOGY 3

Ependymal loss with injury accompanied by II. External granularsmall neurons


a proliferation of subependymal glia (granu- with short axons
lar ependymitis) III. Outer pyramidalmedium and large
E. Microglia neurons
Small, dark, elongated nuclei with scant cyto- IV. Internal granularsmall stellate
plasm neurons
HAM56 positive V. Inner pyramidalmedium size neu-
May proliferate in a diffuse pattern (micro- rons, Betz cells
gliosis) or nodular pattern (microglial nod- VI. Polymorphic layerinnermost layer
ule)commonly seen with viral encephalitis Cerebellum
With destruction of parenchymal tissue, will
Three layers
see a macrophage response (gitter cells)
Molecular layeroutermost layer
Macrophages seen in a variety of patholo-
gies (e.g., infarct, demyelinating disease, Purkinje cell layersingle layer of
radiation, abscess) neurons
F. Choroid plexus Granular cell layerhypercellular,
Produces cerebrospinal fluid (CSF), usually small cells
intraventricular in location Outer granular cell layer in infant overlies
May see choroid plexus in subarachnoid molecular layer, disappears by age 1 yr
space in cerebellopontine angle region White matter
Fibrovascular cores lined by epithelial cells, Fibers run at right angles to cortical
cobblestoned contour surface
May see small nests of arachnoidal (men-
Includes centrum semiovale
ingothelial) cells intermixed
May be focally calcified Two bundles in gray matter parallel to
surface
G. Arachnoid cap cells
Inner line of Baillarger between layers
Seen principally in the arachnoid membrane
3 and 4
Associated with psammoma body formation Stria of Gennari between layers 5 and
(laminated calcifications)
6 in occipital lobe
Dorsal leptomeninges of cord may contain
Basal ganglia
white firm plaques (arachnoiditis ossifi-
cans)laminated dense hyalinized fibrous Includes caudate, putamen, globus pal-
tissue lidus, and amygdala
May see melanocytes in the meninges, more Gray matter tissue
prominent in darker skinned individuals
Caudate and putamen contain thin fasci-
Dura overlying the leptomeninges comprised cles of myelinated fibers (pencil bundles
primarily of dense fibrous connective tissue of Wilson)
H. Architecture Hippocampus
Cortex
Consists of dentate gyrus, Ammons horn,
Six layers, parallel to surface and subiculum
Neurons within the layers have their Ammons horn (CA = cornu Ammons)
processes oriented perpendicular to the divided into four regions
surface CA4 endplate region within the hilus
Layers of the dentate gyrus
I. Molecularfew small neurons, glia, CA3 connects endplate to resistance
outermost layer sector (CA2)
4 NEUROPATHOLOGY REVIEW

CA1 Sommer sector (most sensitive Perifascicular atrophyischemia related,


area of brain to anoxic damage) characteristic of dermatomyositis
Spinal cord Fascicular atrophyspinal muscular atro-
Enlargement of anterior (ventral) horns in phy, hereditary motor and sensory neurop-
cervical and lumbosacral region athies, vasculitis
Central canal in middle Type I atrophymyotonic dystrophy, con-
genital myopathies, spinocerebellar degen-
Terminates with filum terminale
eration
Pineal gland
Type II atrophydisuse, steroids (type
Vaguely nodular architectural arrangement IIB), collagen vascular disease, myasthe-
of cells with glial background nia gravis, cachexia, paraneoplastic neuro-
Corpora arenaceacalcifications, present myopathy
after puberty Hypertrophy
Pineal cystsgliotic cysts often with Normal in athletes (type II fibers)
Rosenthal fibers
Often associated with central nuclei and
Pituitary gland split fibers.
Adenohypophysisnests of epithelioid E. Muscle fiber types (type I, type IIA, type IIB)
cells separated by delicate fibrovascular generally distributed in a mosaic/checkerboard
septae, cell types in a given nest are pattern
mixed type, geographic predominance of Type I fibershigh oxidative, low glyco-
certain cell types lytic activity
Pars intermediaat interface between Type II fiberslow oxidative, high glyco-
adenohypophysis and neurohypophysis, lytic activity
glandular formations may be seen
Different muscles contain different propor-
Neurohypophysisspindled cells loosely tions of the three fiber types
arranged Loss of the mosaic distribution of fiber
II. Skeletal Muscle typesfiber type grouping, indicative of
A. Muscle fibers arranged in bundles (fascicles) chronic denervation and reinnervation
surrounded by perimysium connective tissue; F. Muscle cells of variable lengthrun from ori-
endomysium is connective tissue between mus- gin to insertion
cle fibers G. Muscle cells made up of myofilaments
B. Nuclei multiple, peripherally arranged Two main types: thick myosin filaments and
Central nuclei abnormal if seen in >35% of thin actin filaments
myofibers; Sarcomere segment is the functional contrac-
Increased central nuclei in myotonic dystro- tile unit of muscle formed by the orderly
phy, myotubular myopathy, Charcot-Marie- arrangement of myofilaments
Tooth disease Sarcomeric banding patterns
C. At muscletendon junction, fibers get smaller
Dark central A (anisotropic) band consist-
and often contain internalized nuclei.
ing of mostly thick myosin filaments
D. In cross section, fibers are generally the same
size A band crossed at midpoint by a dark nar-
row transverse M line consisting of cross-
Atrophy
bridges which link adjacent, myosin fila-
Scattered versus grouped (neurogenic), ments together
tend to have angulated contours M band surrounded by paler H band
Rounded fiber atrophymore likely myo- which varies in width as the sarcomeric
pathic length changes
CHAPTER 1 / NORMAL HISTOLOGY 5

Pale I (isotropic) band situated on either K. Neuromuscular junctioninterface between the


side of the A band consisting mostly of nerve and muscle, acetylcholine is released by
thin, actin filaments in combination with the nerve to activate the muscle; the electrical
troponin and tropomyosin proteins impulse reaches the sarcoplasmic reticulum via
the T system.
Z band divides I band at midpoint and
marks the longitudinal boundary of each III. Peripheral Nerve
sarcomere A. Connective tissue layers
During contraction of the fiber, I filaments Epineuriumbinds fascicles together, contin-
slide toward the center of the A band with uous with dura mater at junction of spinal
shortening of the I and H bands nerves and spinal nerve roots
Desmin intermediate filaments link Z discs Perineuriumconcentric layers of flattened
together and join them to the plasmalemma cells which are epithelial membrane antigen
(EMA) positive, separated by collagen that
H. Sarcoplasm
surrounds individual fascicles, forms part of
Represents the cytoplasm of the myofiber the bloodnerve barrier, continuous with the
Consists of a variety of structures/organelles pia arachnoid
Mitochondriatype I > type II Endoneuriumcollagen compartment that
surrounds individual axons, Schwann cells,
T system (transverse tubular system)net- and fibroblasts
work of tubules which is continuous with
B. Mast cells also present in endoneurium,
the muscle cell plasma membrane, allows
increase in number with axonal degeneration
for the rapid passage of depolarization
and neurofibroma
into the interior of the myofiber
C. Renaut bodiescylindrical hyaline bodies
Sarcoplasmic reticuluma series of flat- attached to inner aspect of the perineurium;
tened sacs between and around myofbrils composed of collagen fibers, fibroblasts and epi-
Lipiddroplets lie between myofibrils neurial cells; function not known, increase in
and adjacent to mitochondria, type I > number in compressive neuropathies.
type II D. Blood supply comes from vasa nervorum
Glycogenmore prominent in region of I E. Nerve fibers
band than A band, type II > type I Either myelinated or unmyelinated
Ribosomes, Golgi membranes, intermedi- Class A fibers
ate filaments, microtubules, and lipofuscin Myelinated
also seen.
120 m in diameter; conduction velocity
I. Muscle spindle
12100 m/s
Collections of small intrafusal fibers (314) Class B fibers
enclosed in a connective tissue capsule
Myelinated preganglionic autonomic
Center of spindle is swollen, ends are
fibers
tapered
Located between muscle fascicles in perimy- Up to 3 m in diameter; conduction veloc-
sial connective tissue ity 315 m/s
Consists of two types of intrafusal fibers: Class C fibers
nuclear bag fibers and nuclear chain fibers Unmyelinated
More numerous in muscles involved in deli- Small fibers (0.21.5 m diameter); con-
cate movements duction velocity 0.31.6 m/s
Associated with sensory nerve endings F. Myelin
J. Motor unitconsists of anterior horn cell, Can be highlighted with certain stains
nerve fiber arising from it and the muscle fibers (Luxol fast blue, Loyez, osmium, periodic
supplied by it. acid schiff (PAS), Masson trichrome)
6 NEUROPATHOLOGY REVIEW

Formed by fusion of Schwann cell mem- Distance between nodes is proportional to


branes myelin thickness
75% lipid and 25% protein Conduction of impulses along myelinated
Major lipids comprising myelin: cholesterol, fibers proceeds in a discontinuous manner
sphingomyelin, galactolipid from node to node (saltatory conduction)
One Schwann cell per myelinated axon (in I. Myelinated axons
CNS, one oligodendrocyte supplies myelin Axon delimited by axolemma membrane
to multiple axons)
Axolemma separated from adjacent Schwann
Schwann cells S-100 positive by immunohis-
cell by periaxonal space of Klebs
tochemistry
Axonal cytoplasm contains mitochondria,
Pi granules of Reichlamellated structures,
smooth endoplasmic reticulum, glycogen,
cytoplasm accumulates along with lipofuscin
ribosomes, peroxisomes, neurotransmitter
with increasing age
vesicles, and filaments/tubules
Corpuscles of Erzholzspherical bodies in
Schwann cell cytoplasm. Filaments include actin, neurofilaments, and
microtubules
G. Schmidt-Lanterman clefts
Cleft splits the cytoplasmic membranes and J. Unmyelinated axons
serves as a route of passage for substances Best evaluated and studied by electron
from the outer cytoplasmic layer through the microscopy
myelin sheath to the inner cytoplasm More numerous than myelinated axons:
The number of clefts correlate with the diam- 34 : 1 unmyelinated : myelinated axons
eter of the axon Frequently associated with Schwann cells;
H. Node of Ranvier may be more than one axon per Schwann
Gaps in the myelin sheath cell
2 Vascular Lesions

I. Anoxia ties exist; the entire brain not uniformly


involved (selective vulnerability)
A. Anoxia observed in a wide variety of clinical
Neurons most sensitive to anoxia
conditions and forms
Hippocampal, Sommer sector (CA1)
Anoxic anoxiainsufficient oxygen
reaches blood (e.g., drowning) most sensitive

Anemic anoxiainsufficient oxygen con- Cerebral cortex, layers III, V, and VI


tent in blood (e.g., carbon monoxide poi- (which contain larger-size neurons)
soning) Cerebellar Purkinje cells (if patient sur-
Histotoxic anoxiapoisons interfere with vives for a period of time, may see
oxygen utilization (e.g., cyanide, sulfide) Bergmann gliosis)
Stagnant anoxiamost common, Caudate and putamen
decreased cerebral perfusion (e.g., cardiac Accentuated in the boundary zones
arrest); factors that determine amount of between vascular distributions
brain damage after cardiac arrest include D. Gross pathology
duration of ischemia, degree of ischemia,
Swollen, soft
temperature during the event and blood glu-
cose levels Gray matter is dusky
Areas of cavitation in a laminar pattern
B. Precise mechanisms not completely eluci-
may be observed
dated; however, a number of consequences
occur which have a negative effect on the Pseudolaminar is used to describe
CNS involvement of more than one cortical
Edema layer

Lactic acid accumulation, decreased pH Laminar describes involvement of a sin-


gle cortical layer
Increase in free fatty acids
E. Microscopic pathology
Increase in extracellular potassium and
Dendrite and astrocyte swelling (i.e., spong-
ammonia
iness of the neuropil)
Abnormalities in calcium flux Ischemic (homogenized) neurons (i.e., red
Reperfusion problems and dead)
C. Morphology of ischemia highly variable and Endothelial hyperplasia
unpredictable. Microglial reaction
Numerous regional and cellular vulnerabili- Dissolution of neurons after several days

7
8 NEUROPATHOLOGY REVIEW

II. Infarction (2) interstitial edema, (3) pyknosis, (4)


A. General hypereosinophilia of neurons, (5) microva-
cuolization of neurons (swollen mito-
Cerebral blood flow 20% of cardiac out-
chondria)
put, 15% of oxygen consumption of body.
24 h: macrophage infiltration begins, axo-
Main anterior flow (70%) through internal
nal swelling, may see neutrophilic infiltra-
carotid arteries; posterior flow via vertebral
tion (ceases by d 5)
arteriesthese systems anastomose via the
anterior and posterior communicating arter- 34 d: prominent macrophage infiltration
ies to form the circle of Willis 710 d: astrocytic proliferation and hyper-
Disruption in flow can result in infarct trophy evident
(stroke) 30 d: intense gliosis
Stroke is a clinical term referring to May have a hemorrhagic component (18
abrupt onset of focal or global neurologi- 48%); most hemorrhagic infarcts are
cal symptoms caused by ischemia or hem- embolic artery events; hemorrhage in
orrhagesymptoms >24 h; if symptoms infarcts due to reperfusion of necrotic ves-
resolve in less than 24 h = transient isch- sels or occlusion of venous drainage
emic attack (TIA) May encounter degeneration of tracts distal
Risk of stroke increased with increasing to infarct
age, male > female, smoking, hyperten- E. Lacunar infarct
sion, atrial fibrillation, carotid artery steno-
sis, hyperlipidemia, diabetes, heart surgery, The infarcts range in size from 34 mm
antiphospholipid antibodies, high-estrogen up to 1.5 cm
oral contraceptives Most common sites: putamen, caudate, thal-
Venous thrombosis associated with preg- amus, pons, internal capsule, and convolu-
nancy and oral contraceptive use tional white matter

Most common sites of venous thrombo- Causes: (1) lipohyalinosis, (2) occlusion of
small penetrating vessels, (3) dissection,
sis in superior sagittal sinus, lateral
(4) emboli
sinuses, and straight sinus
Small perivascular cavities common in
Septic thrombosis most common in cav-
basal ganglia and deep white matter; etat
ernous sinus, often due to contiguous lacunaire (gray matter) and etat crible
spread from soft tissue/sinus infection (deep white matter)
B. Determining the age of an infarct either F. Respirator brain
grossly or microscopically can only be done
within a wide range. The times given are Also known as (AKA) diffuse anoxic
guidelines rather than absolutes. encephalopathy
C. Gross pathology Permanent global ischemia due to nonper-
fusion of entire brain
Unequivocal alterations require up to 24 h;
early changes include edema, congestion, Brain perfusion depends on mean arterial
softening blood pressure exceeding the intracranial
pressure (cerebral perfusion pressure =
48 h: crackingseparation of the
mean arterial blood pressure intracranial
necrotic tissue from intact tissue
pressure)
72 h: infarcted area usually clearly deline-
The most extreme form of stagnant anoxia
ated; cortex friable and soft
Grosslydusky brown discoloration of cor-
A 1-cm cavity takes 23 months to form
tex, blurring of graywhite junction, gen-
Once cavitation begins, it is difficult to eral friability of tissue (brain often does
determine the age of the infarct. not fix well in formalin)
D. Microscopic pathology Microscopicallyischemic neurons every-
Earliest changes: (1) astrocytic swelling, where infarcts
CHAPTER 2 / VASCULAR LESIONS 9

G. Carbon monoxide (CO) poisoning Risk of rupture, greatest in aneurysms > 1


A form of anemic anoxia due to displace- cm in size
ment of oxygen from its binding site on Giant aneurysm defined as > 2.5 cm, usu-
hemoglobin ally causes compressive or embolic
CO binds irreversibly to hemoglobin, symptoms
reducing its oxygen-carrying capacity Etiology
CO directly binds to iron-rich areas of Many theories
brain (globus pallidus and pars reticulata
Congenital defects of part or all of the
of substantia nigra)
media at the arterial bifurcation, does
Pathologically marked by necrosis of glo- not explain why most arise in adulthood
bus pallidus and substantia nigra.
Remnants of embryonic vessels
May see demyelination and cerebral white
matter destruction (Grenkers myelin- Focal destruction of the internal elastic
opathy) membrane due to hemodynamic alter-
H. Air embolism ations, about 39% of patients with arte-
riovenous malformation (AVM) have
May be related to decompression sickness
aneurysms
the bendsnitrogen gas in blood, can
cause spinal cord microinfarcts Abnormalities in specific collagen
May also occur related to cardiac bypass subsets
surgery Associated conditions:
I. Hypoglycemic brain damage Polycystic kidney disease, Potter type
Causes selective neuronal necrosis-like 3
ischemia EhlersDanlos syndrome (types IV
Different mechanism of injury than and VI)
ischemia Coarctation of the aorta

Decreased lactate and pyruvate Marfans syndrome

Tissue alkalosis Pseudoxanthoma elasticum

Neuronal necrosis in cerebral cortex super- Pathology


ficial layers, hippocampus (CA1 and den- Media usually defective
tate) and caudate; no Purkinje cell necrosis
Intimal hyperplasia and sometimes a
III. Aneurysms
gap in the internal elastic membrane
A. Saccular aneurysms seen
AKA: berry, congenital or medical Aneurysm wall usually contains fibrous
defect tissue
Incidence in general population, 25%;
Atherosclerotic changes occasionally
female : male 3 : 2.
seen
Usually present at arterial bifurcation;
most asymptomatic Phagocytosis and hemosiderin deposi-
tion may be present
85% in anterior circle of Willis
B. Infectious (septic) aneurysms
25% multiple
Most related to bacterial endocarditis
20% bilateral
Arterial wall weakened by pyogenic bac-
Most common sites teria which usually reach the wall by an
Middle cerebral artery trifurcation infected embolus
Anterior communicating artery junction May be multiple
Internal carotid arteryposterior com- Located on distal branches of the mid-
municating artery junction dle cerebral artery
10 NEUROPATHOLOGY REVIEW

Organisms usually of low virulence, partic- Less compact than AVM or cavernous mal-
ularly Streptococcus viridans and Staphylo- formation
coccus aureus May have a large central draining vein
Mycotic aneurysms typically refer to fun- Varix is a single dilated/large vein
gal aneurysms; Aspergillus the most com-
mon organism responsible D. Cavernous malformation (angioma)
C. Fusiform aneurysms Large, sinusoidal-type vessels in apposition
to each other
Related to dolichoectasia (elongation, wid-
ening, and tortuosity of a cerebral artery) Little or no intervening parenchyma
Supraclinoid segment of internal carotid Compact malformations
artery and basilar artery most common Mineralization and ossification common;
sites occasionally massive
Fusiform aneurysm refers to dilated seg- May bleed
ment of artery E. Capillary telangiectasia
Seen commonly with advanced atheroscle-
Capillary sized vessels
rosis
Separated by normal neural parenchyma
IV. Vascular Malformations
Common in the striate pons
A. General information
True malformations result from the embry- Often an incidental finding at autopsy
onic vascular network V. Hemorrhage
Some increase in size by incorporating A. Common causes
adjacent vesselsrecruitment Spontaneous intracerebral hemorrhage,
Clinical symptoms include seizures, steal often ganglionic (caudate, putamen) related
phenomenon, hemorrhage to hypertensionmost common cause of
In children, excessive shunting may lead to nontraumatic hemorrhage
cardiac decompensation (especially vein of Ruptured saccular aneurysm
Galen malformations). Vascular malformation, particularly AVM
True incidence of hemorrhage unknown, and cavernous angioma
estimates = 510% overall Coagulopathy associated, may have both
A subset of malformations is of mixed platelet and prothrombin time abnormal-
type. ities
B. Arteriovenous malformation (AVM) Congophilic (amyloid) angiopathy
Admixture of arteries, veins, and intermedi-
Lobar hemorrhage
ate size vessels
Vessels are separated by gliotic neural Elderly
parenchyma Amyloid deposition in vessel walls both
Foci of mineralization and hemosiderin in meninges and cortex
deposition common Most commonly -amyloid type (chro-
Typically superficial, wedge-shaped with mosome 21)
the apex directed toward the ventricle
Associated with Downs syndrome and
Commonly found in surgical series; the Alzheimers disease
most common vascular malformation asso-
ciated with hemorrhage; peak presentation Highlighted on Congo red (apple green
2nd4th decades birefringence with polarized light), thi-
C. Venous malformation (angioma) oflavin S or T, and crystal violet stains
Veins of varying sizes Neoplasms
Vessels separated by mostly normal Blood dyscrasias (i.e., sickle cell anemia)
parenchyma Vasculitis
CHAPTER 2 / VASCULAR LESIONS 11

B. Pathology Associated with polymyalgia rheumatica


Soft, gelatinous clot May be a focal process (skip lesions)
Small vessels with thrombosis may project Lymphoplasmacytic inflammation
into the hemorrhage granulomas, fibrous scarring with
Petechial hemorrhage may be seen around healing
the large hemorrhage Takayasus arteritis
Ischemic necrosis seen in associated brain Aortic arch and branches and descend-
tissue
ing aorta
Clot edge often has hemosiderin-laden mac-
Younger patients (1540 yr), Oriental
rophages
females at higher risk
Astrocytic proliferation often prominent at
the edge Lymphoplasmacytic inflammation of the
media with fibrosis, granulomas/giant
Amyloid may be seen in adjacent vessels cells
if congophilic angiopathy is the cause
Thromboangiitis obliterans
Venous hemorrhageusually the infarcts
Vasculitis associated with connective tis-
are larger and not confined to any arterial
sue disease, (i.e., systemic lupus erythema-
distribution, often over the convexity
tosus [SLE], Sjogrens syndrome, Behcets
VI. Vasculitis syndrome
A. Common causes Churg-Straussnecrotizing vasculitis with
Polyarteritis nodosa (PAN) and its variants eosinophilia
Systemic disease Primary angiitis of central nervous system
Male : female 2 : 1 Headaches, multifocal deficits, diffuse
encephalopathy
Any age, peak 4050 yr
Adults (ages 3050 yr)
Necrotizing vasculitis with fibrinoid
ESR normal or mildly elevated
necrosis of medium-sized vessels
Biopsy of nondominant temporal tip,
May represent immune-complex-
including leptomeninges (highest yield),
mediated vasculitis (subset of patients
cortex, and white matter recommended
are hepatitis B or C positive)
Hypersensitivity angiitis Arteries involved more than veins

Wegeners granulomatosis May be granulomatous or nongranulo-


matous
Systemic disease often with respiratory
tract component May be complicated by infarct or hem-
orrhage
Most patients 4060 yr
Other causes
ANCA (anti-neutrophil cytoplasmic anti- A variety of other conditions can result
bodies)directed against proteinase 3 in a vasculitic pattern of injury patho-
Lymphomatoid granulomatosis logically
Giant cell arteritis Drugs (e.g., cocaine, amphetamines)
The most common of the granuloma- Infection (e.g., Treponema, Borrelia,
tous vasculitides Herpes, HIV, Aspergillus, Mucor)
Older patients >50 yr Lymphoproliferative disorders
Primary target is extracranial arteries of Other (demyelinating disease, atheroscle-
head, but may involve cerebral vessels rosis, amyloid, etc.)
Headache, blindness, increased erythro- B. Pathology
cyte sedimentation rate (ESR) Segmental inflammation and necrosis of
12 NEUROPATHOLOGY REVIEW

vascular walls are the common features of C. Atherosclerosis/hypertensive angiopathy


the above-listed vasculitides. Risk factors: dyslipidemia, hypertension,
Vasculitis associated with SLE and lymph- smoking, diabetes
omatoid granulomatosis are the only two Initial atherosclerotic lesion is fatty streak
which typically involve brain parenchyma. marked by foam cells filled with low-
PAN, Wegeners granulomatosis, and giant density lipoprotein (LDL) cholesterol.
cell arteritis involve vessels in the sub- Fatty streaks may develop into fibrous
arachnoid space, peripheral nervous sys- plaques (especially develop at outer
tem, or the extracranial vessels. aspects of arterial bifurcation where lami-
nar flow is disturbed)connective tissue,
Neurologic dysfunction in all entities is
smooth muscle cells, foam cells, lympho-
related to ischemia.
cytes, necrotic cell debris, and extracellular
VII. Miscellaneous lipids/cholesterol
A. Binswangers disease Turbulence caused by plaque may cause
Rare condition generally presenting further disease progression.
between ages 5060 yr; evolves over 35 Complicated plaque results from disruption
yr of endothelium resulting in thrombus for-
mation (risk for occlusion and emboli).
Also called subcortical arteriosclerotic
encephalopathy Hypertensive angiopathy shifts the autoregu-
lation (maintains cerebral blood flow at a
Moderate intermittent hypertension and pro- constant level between mean arterial pres-
gressive, often profound dementia are fea- sures of 50150 mm Hg) curve to the right,
tures. raising the lower limit of regulation at which
Widespread vascular alterations and white adequate cerebral flow can be maintained.
matter changes are seen and readily demon- Malignant hypertensionacute hypertension
strated with neuroimaging.
Diffuse cerebral dysfunction, headache,
Ventricular dilatation, often secondary to nausea, vomiting, altered consciousness
hydrocephalus ex vacuo
May be the result of pheochromocytoma
Focal and diffuse myelin loss with associ- (release of catecholamine), disseminated
ated reactive astrocytosis in the deep hemi- vasculitis, eclampsia, rebound drug
spheric white matter effect
Subcortical arcuate fibers are spared. Brain edema, focal ischemia, and intra-
Changes most severe in the temporal and cerebral hemorrhage
occipital lobes. Chronic hypertension
Demyelination is thought to be secondary Further worsens atherosclerotic changes
to reduced perfusion due to arteriosclerosis in large arteries
of small penetrating vessels. Causes small vessel disease with disrup-
B. CADASIL (Cerebral autosomal dominant arte- tion of blood-brain barrier resulting in
riopathy with subcortical infarcts and leu- basal lamina thickening and reduplica-
koencephalopathy) tion, smooth-muscle degeneration, fibri-
Mutation of notch 3 gene on chromosome noid change (necrosis), and increased
19q12 collagen deposition (lipohyalinosiscol-
lagenous fibrosis)
Strokes and vascular dementia
May form Charcot-Bouchard microane-
Systemic disorder urysms (miliary aneurysms)dilata-
Thickened vessel walls (media and adventi- tions/outpouchings of vessel wall
tia) with basophilic, PAS-positive granules caused by fibrinoid change
within smooth muscle cells D. Moyamoya syndrome
Variable degrees of perivascular atrophy Defined angiographically by spontaneous
CHAPTER 2 / VASCULAR LESIONS 13

occlusion of the circle of Willis and the Hemiparesis and hemorrhage


presence of abnormal collateralization
Intimal fibroplasia usually without athero-
Two peaks of incidence1st and 4th sclerotic changes, no inflammation
decades
Most patients young, <20 yr, females >
males
3 Tumors

I. Astrocytic Neoplasms >90% derived from fibrillary astrocytes


A. Classification schemas (grading) for astrocy- Arise in white matter
tomas Frontal lobe>parietal>temporal>occipital
WHO classification Noncontrast enhancing, ill-defined region
of low density on CT scan
Grade I: pilocytic astrocytoma, subepen-
dymal giant cell astrocytoma Seizures, sensorimotor deficits, increased
intracranial pressure, altered mentation
Grade II: well-differentiated diffuse Grossly a solid tumor, infiltrative growth
astrocytoma, pleomorphic xanthoastro- pattern, obliterate graywhite junction, sub-
cytoma tle color change (yellow/tan or gray)
Grade III: anaplastic astrocytoma Microscopic
Grade IV: glioblastoma multiforme Uneven cellularity
Ringertz classification Rare mitoses
Low-grade astrocytoma Irregular nuclear contours, hyperchro-
Anaplastic astrocytoma matic nuclei
Glioblastoma multiforme Mild pleomorphism
Daumas-Duport classification (St. Anne- May see microcystic change (do not see
Mayo) typically with gliosis)
Calcification (15%)
Grade Criteria
Should not see vascular or endothelial pro-
1 0/4 liferation or necrosis
2 1/4 GFAP (glial fibrillary acidic protein) pos-
3 2/4 itive
4 34/4 Intermediate cytoplasmic filaments
(Criteria: mitoses, necrosis, nuclear atypia, Also stains ependymomas and choroid
endothelial proliferation) plexus tumors
B. Low-grade astrocytoma Decreased staining in higher-grade
20% of all gliomas, 1015% of all astro- tumors
cytic tumors Electron microscopy
Incidence: 1.4 new cases/1 million popula- Astrocytoma cells marked by cyto-
tion/yr plasmic intermediate filaments (711
4th-decade peak nm)

15
16 NEUROPATHOLOGY REVIEW

Generally not useful for diagnosis Hypertrophic cells (reactive astrocytes)


DNA/cell proliferation prominent

Most diploid Hypercellularity with even distribution

Ki-67 (MIB-1) proliferation indices Lack nuclear hyperchromasia and angu-


roughly correlate with tumor grade larity

Tumor heterogeneity in proliferation Absent calcification


different rates of cell proliferation and Rosenthal fibersintensely eosino-
varying histologic appearance in differ- philic, elongate, or corkscrew-shaped
ent regions of the tumor structures
Molecular genetics Collagen and hemosiderin deposition
TP53 mutations in >60% of diffuse Lack microcystic change
astrocytomas (chromosome 17)
No satellitosis
appear to progress more frequently
E. Anaplastic astrocytoma
Increased platelet-derived growth factor
receptor (PDGFR) alpha mRNA 4th and 5th decades
expression Can arise from low-grade astrocytoma
Other abnormalities which are variably Clinical presentation similar to low-grade
present include gain 7q, amplification astrocytoma
8q, LOH on 10p, and LOH on 22q. Same gross pathologic features as low-
May remain well differentiated or progress grade astrocytoma
to a higher-grade tumor over time Microscopic pathology
Average postoperative survival 68 yr Hypercellularity with more prominent
Young age and gross total resection do nuclear pleomorphism versus low-grade
better astrocytoma
C. Gemistocytic astrocytoma Mitotic figures more readily identifiable
Cells with plump, eosinophilic cell bodies Vascular or endothelial proliferation
and short processes may be present (not required)if prom-
Nuclei usually eccentric with small inently seen in the tumor, may warrant
nucleoli WHO grade IV designation
Perivascular lymphocytes common No necrosis
Lots of gemistocytes associated with more Secondary structures of Sherer (helpful,
aggressive behavior (20% or more gemisto- not always presentsatellitosis of
cytic component) tumor cells around pre-existing element
Gemistocytes usually with lower rate of such as neurons or vessels)
cell proliferation than background tumor Cell proliferation rate greater than low-
cells grade astrocytoma
D. Gliosis versus glioma P16 deletion (30%), RB alterations (25%),
Brain tissue adjacent to other lesions may P19 deletion (15%), CDK4 amplification
have gliosis (10%)
Limited biopsy makes it difficult to distin- Some with LOH 10q, 19q, and 22q
guish reactive gliosis from glioma Postoperative survival slightly > 2 yr
Biopsy for recurrent tumor will show reac- Treatment same as glioblastoma multi-
tive gliosis forme (radiation)
History of head traumamay see gliosis F. Glioblastoma multiforme (GBM)
Reactive gliosis Several possible origins
CHAPTER 3 / TUMORS 17

Astrocytic dedifferentiation (associated Polymorphous cell proliferation


with a loss of chromosome 10) smooth muscle cells, pericytes,
adventitia, endothelial cells
Nonastrocytic glioma differentiation
some use the term to denote high-grade Also seen in anaplastic oligodendro-
ependymomas, oligodendrogliomas and gliomas, anaplastic ependymoma,
mixed gliomas pilocytic astrocytoma, abscess
De novotwo genetic groups A number of histiologic variants
Loss of heterozygosity on chromo- existsmall cell, metaplastic, epitheli-
some 17p, mutations of p53 gene, no oid, granular cell, spongioblastomatous,
amplification of epidermal growth giant cell (monstrocellular sarcoma of
factor receptor (EGFR) gene Zulch)
Loss of heterozygosity on chromo- Most tumors focally GFAP positive,
some 10 and amplification of EGFR S-100 positive, vimentin positive; negative
gene; no loss of chromosome 17p for cytokeratin markers (note: some cross-
immunoreactivity with certain keratin
Tumor suppressor gene loci for astrocy-
immunostains such as AE1/3)
toma on chromosomes 10, 17p, 9p, 19q,
and 22 Differential diagnosis
Most common glioma, 5060% of all Radiation change
astrocytic neoplasms, 1215% of all intra- Lack of perinecrotic pseudopali-
cranial neoplasms sading
5th and 6th decades Microcalcifications in necrosis
Rare < age 30 yr
Vascular changes (hyalinized, scle-
Male : female 3 : 2 rotic vessel walls)
Increased risks with hydrocarbon exposure Bizarre cytologic atypia and cyto-
and radiation
plasmic vacuolization
Ring enhancing, central hypodense area
Macrophages associated with
(necrosis)also seen with abscesses, meta-
necrosis
stases, evolving infarcts
Short duration of pre-operative symptoms Metastatic carcinoma
Grossly, may appear somewhat discrete Discrete gross lesion
with variegated color but often widely No fibrillary background
infiltrative and frequently crosses midline Collagenous stroma
structures (butterfly glioma)
Discrete cell borders
Microscopic pathology
No perinecrotic pseudopalisading
Geographic necrosis
Generally no vascular proliferation
Perinecrotic pseudopalisading
Epithelial appearing cells
Usually areas of prominent cellularity
Epithelial membrane antigen (EMA)
and nuclear pleomorphism
and cytokeratin positive (some cyto-
Readily identifiable mitotic figures keratin markers such as AE1/3 cross-
Vascular or endothelial proliferation react with gliomas), most GFAP neg-
Proliferation and piling up of cells ative
around vascular lumina More favorable prognosis
Capillary and glomeruloid appear-
Young age (<45 yr)
ances
Some cells participating in the vascu-
Long duration of preoperative
symptoms
lar proliferation are Factor VIII neg-
ative Favorable neurologic condition
18 NEUROPATHOLOGY REVIEW

Presence of a substantial, better differ- Altered behavior and mentation, seizures,


entiated component headaches, focal deficits
Extent of resection Most diagnosed postmortem (difficult to
diagnose on biopsy alone)
Prominent giant-cell component (contro-
versial) Microscopic pathology

Lower rate of cell proliferation Undifferentiated cells with foci of


higher-grade tumor
Clinical course
Peripheral margin cells are elongated
Without therapy: 1416 wk
and slender
With therapy: about 1 yr
Mitotic activity variable
Chemotherapy generally not useful,
GFAP positive
treated with radiation
Metastases Secondary structures of Sherer
#1 lung
Cell of origin not known
#2 bone Poor prognosis
G. Multifocality in glial tumors J. Pilocytic astrocytoma
GBM is most common WHO grade I
25% Most common glioma in children, associ-
ated with neurofibromatosis type I
Increased recognition related to degree of
tissue sampling Locationscerebellum, wall third ventri-
No age difference cle, optic nerve, temporal lobe, thalamus,
basal ganglia
H. Gliosarcoma
Presentation most commonly includes
Feigin tumor focal neurological deficits or increased
Considered by many to be a GBM variant, intracranial pressure symptoms
genetic changes similar to GBM Discrete cystic mass; radiographically a
<2% of GBMs cystic tumor with enhancing mural
Similar presenting features, age, distribu- nodule(s)
tion and prognosis as GBM Microscopic pathology
Temporal lobe most common site Nuclear pleomorphisma degenerative
Sarcoma and GBM microscopically change
Increased metastasessarcoma part Mitoses rare
Sarcoma may be fibrosarcoma, malignant
Vascular proliferation and sclerosis
fibrous histiocytoma (MFH); rarely chon-
drosarcoma, rhabdomyosarcoma, or angio- Biphasic appearance
sarcoma Microcystic, loose and reticular areas
In some tumors, identical genetic abnor- Compact areas with more spindled
malities present in both components of the cells
tumor, supporting the notion that these
Granular eosinophilic bodies or cells
components arise from a single cell of
with protein drops, seen in loose
origin
areas
I. Gliomatosis cerebri
Rosenthal fibers more common in
Diffusely infiltrative glioma (WHO grade compact areas
III)
Microcysts
Peak incidence age 40-50 yr
Diffuse, symmetric enlargement of brain, Rarely lobular without cysts
loss of normal architectural landmarks Clinical course
CHAPTER 3 / TUMORS 19

Grow slowly, low rates of cell prolifer- Origin still debated, most classify as an
ation astrocytoma (GFAP positive); Scattered
cells in tumor stain with neuronal mark-
Better survival with gross total or radi-
ers, recently described association with cor-
cal subtotal excision
tical dysplasia implies possible maldevel-
5 yr survival: 85%; 10 yr survival: opmental origin
79% Temporal or parietal lobes
Malignant transformation rare Cystic with mural nodule(s) configuration
Infiltration of the meninges does not Nodulessuperficial, in contact with lepto-
adversely affect prognosis. meninges
K. Protoplasmic astrocytoma Abrupt interface with adjacent paren-
Young patients (males > females) chyma
Mixtures of protoplasmic and fibrillary Microscopic pathology
cells Densely cellular central areas
Superficial
Periphery less cellular
Cysts grossly and microscopically
Pleomorphic cells
Fairly well demarcated
Short processes, fibril poor Lipidization of astrocytes
GFAP immunostaining variable Perivascular lymphocytes
Favorable prognosis? Reticulin rich
L. Subependymal giant-cell astrocytoma Necrosis absent
Increased intracranial pressure (obstruction
Mitoses rare, generally low rates of cell
of foramina of Monro)
proliferation
Associated with tuberous sclerosis
(<20 yr), can occur later in life Rare anaplastic lesions with increased
Males = females mitoses and necrosis exist
WHO grade I GBM in differential diagnosisnecro-
sis, mitoses, older age, vascular prolifer-
Elevated nodule, wall of anterior lateral
ation, reticulin poor
ventricle
Often associated with smaller hamartomas, Generally favorable prognosis
which have been likened to candle drip- Rare anaplastic change (increased mitoses
pings and necrosis), malignant transformation or
Discrete grossly and microscopically recurrence in 1025%
Microscopic pathology N. Brain stem glioma
Giant cells with glassy eosinophilic Childhood, rarely adults
cytoplasm Generally unresectable
Smaller elongated cells in background Cranial nerve palsies, long tract signs, gait
disturbances, vomiting, and cerebellar
Coarsely fibrillar background
dysfunction
Rare mitoses Grossly may present as enlargement of
GFAP positive brain stem without a discrete mass
Good prognosis, low recurrence rate Bilateral or unilateral
M. Pleomorphic xanthoastrocytoma May be lobulated mass, may encircle
First two decades most commonly basilar artery
Seizures 1st decade of life Microscopic pathology
WHO grade II tumor Fasciculated and elongated cells
20 NEUROPATHOLOGY REVIEW

May see anaplastic astrocytoma or Disfigurement, obstruction, CSF rhinor-


GBM rhea, meningitis
Most fibrillary, diffuse astrocytomas Disorganized mixture of neuroglia and
fibrovascular tissue
Some pilocytic (1015%)better cir-
Treat by excision
cumscribed, less infiltrative
Not truly a glioma, more akin to hetero-
O. Optic nerve glioma topic or ectopic neuroglial tissue
1st decade Other sites of glial heterotopia include
Neurofibromatosis type I associated occipital bone, nasopharynx, scalp, subcu-
bilateral taneous tissue of temporal and parietal
Adultsanterior optic pathway, regions
aggressive Q. Astroblastoma
Visual loss Young adults
Grow within optic sheathenlarged optic Variable prognosis, tumors with high-
nerve grade histological features tend to behave
Fusiform enlargement more aggressively
Posterior lesionexophytic suprasellar chi- Cerebral hemispheres
asmal mass Discrete mass
Microscopic pathology Microscopic pathology
Two patterns Solid, elongated cells that radiate from

vasculature
Lacy, low cellularity, lacking polar-
ity, cystic foci with mucoid material Slight or moderate nuclear pleomor-
Fibrillated cells in bundles delineated phism
by nerve septae Rare mitoses
Rosenthal fibers (if pilocytic) GFAP and S-100 positive
Nuclei with little anaplasia Overlap with ordinary astrocytomas
Adultsanaplastic and GBM forms R. Desmoplastic cerebral astrocytoma of infancy
Rare mitoses, necrosis, endothelial pro- Superficial, frequently dural based
liferation <1 yr of age
Evaluate margin Desmoplasia
Clinical course Derived from subpial astrocytes
Frontal or parietal lobe
Slow growingpilocytic types in
children Cystic
No giant cells, no lipidization, no ganglion
Long survivalanterior to optic chiasm
cells (if ganglion cells presentdesmo-
Optic nerve and chiasmlocation, age, plastic infantile ganglioglioma)
extent of resection important factors GFAP positive
5% recur after total excision Favorable prognosis
Radiation therapy for incomplete S. Gliofibroma
excision Rare
P. Nasal glioma Tumors with malignant glial component
Congenital anterior displacement of non- and abundant collagen
neoplastic glial tissue Generally well-circumscribed, firm mass
No connection to brain (encephalocele if a Islands, cords, or single glial cells in a col-
connection is present) lagen stroma
CHAPTER 3 / TUMORS 21

Behavior seems to be dictated by grade of II. Oligodendroglial Neoplasms


glioma A. Oligodendroglioma
T. Chordoid glioma 515% of glial neoplasms
Rare, slow growing Long history of signs and symptoms
Most arise in 3rd ventricle in adults (mean 15 yr) prior to diagnosis
Solid tumors composed of clusters or Seizures, headaches most common symp-
cords of epithelioid cells with a variably toms; about 20% hemorrhage
mucinous stroma Adults > children; mean age 3545 yr
GFAP positive WHO grade II lesion
Mitoses rare White matter
Stromal lymphoplasmacytic infiltrate often Frontal lobe>parietal>temporal>occipital
with numerous Russell bodies Microscopic pathology
Tends to recur if incompletely excised Sheets of cells, distinct cytoplasmic
U. Ganglioglioma borders
WHO grade I tumor Vacuolated fried egg appearance
May be confused with astrocytoma formalin-fixation artifact
Most common in temporal lobe Central nucleus, minimal pleomorphism
Children > adults Arcuate or chicken wire vascular
Most patients present with chronic epi- pattern
lepsy Calcificationprominent at periphery,
Cystic tumor with calcified mural nodule seen in 90% of cases
A glioneuronal tumor; microscopically Cortical invasion
need to identify both atypical ganglionic
May see meningeal invasion with
(neuronal) cell component and a glioma
component desmoplasia

Atypical neurons may be binucleated Subpial and perineuronal arrangement


of tumor cells
Calcifications common
May see microcystic change
Granular bodies
Rare or absent mitoses Rare mitoses at most
Perivascular lymphocytes Occasional signet-ring cell variant
(degenerative phenomenon), no clinical
Associated with coexistent cortical dyspla-
significance
sia, possible maldevelopmental origin
GFAP negative or focally positive; S-100,
Generally good prognosis with complete
A2B5, and Leu-7 positive
excision; rare aggressive tumors reported
Cell proliferation marker labeling indices
Desmoplastic infantile ganglioglioma
low
very young patients, large size, superficial
location Molecular genetics
Gangliocytomatumor composed of Most frequent genetic alteration present
mature ganglion cells is LOH chromosome 19q
Dysplastic gangliocytoma of the cerebel- Other abnormalities include LOH 1p
lum (Lhermitte-Duclos)occurs in setting and 4q
of Cowden syndrome caused by phospha- TP53 mutations relatively rare
tase and tensin homolog deleted on chro-
mosome 10 (PTEN) germline mutation, Generally difficult to totally resect
hamartomatous lesion of ganglion cells Most do not survive beyond 10 yr
22 NEUROPATHOLOGY REVIEW

Radiation therapy of debated use, more Mitoses rare, low rates of cell prolifer-
likely to be chemoresponsive ation
May dedifferentiate with time Good prognosis, potentially curable with
B. Anaplastic oligodendroglioma complete excision
WHO grade III tumor E. Central neurocytoma
Pleomorphic nuclei WHO grade II tumor
Mitoses, often > 5 per high-power fields Another oligodendroglioma look-alike
Vascular proliferation Discrete, often partly calcified
Necrosis Intraventricular near septum pellucidum or
in 3rd ventricle
Increased cellularity
Rare examples of extraventricular neurocy-
Worse prognosis than low-grade tumor, toma have been reported including the pap-
median survival 34 yr illary glioneuronal tumor and cerebellar
Rarely can see glioblastoma multiforme liponeurocytoma
derived from oligodendroglial tumor Most frequent in young adults
1p and 19q chromosome deletions associ- Uniform nuclei with speckled chromatin
ated with increased likelihood of chemo-
responsiveness Perivascular pattern, pseudorosettes
C. Mixed glioma Fibrillary background
Clinically and radiographically similar to May be focally calcified
astrocytomas and oligodendrogliomas Mitoses generally rare
Most frequently a mixture of astrocytoma Generally GFAP negative, synaptophysin
and oligodendroglioma (oligoastrocytoma), positive
2030% minor component Dense core granules on electron micros-
May have anaplastic tumor components copy (evidence of neuronal differentiation)
present (malignant mixed glioma) Surgical cure may be possible with com-
A subset of tumors have genetic alter- plete excision
ations similar to oligodendroglioma III. Ependymal Neoplasms
Prognosis may be intermediate between A. Rosettes
pure oligodendroglioma and astrocytoma
True rosettes (tumor cells themselves form
Rarely see mixtures of other glioma types a lumen)
(oligoependymoma and ependymoma
astrocytoma) Flexner Wintersteiner
Small lumen
D. Dysembryoplastic neuroepithelial tumor
Cuboidal lining
WHO grade I tumor
Retinoblastoma
An oligodendroglioma look-alike
Young patients, childhood, chronic epi- Ependymal
lepsy history Ependymal lining

Multinodular, multicystic, cortical based Ciliary attachment (blepharoplast)

Associated with cortical dysplasia (malde- Cilia in 9 : 1 orientation


velopmental lesion) Canal/channel with long lumen
Mixture of predominantly oligodendroglial Pseudorosettes (tumor cells arrange them-
cells with fewer numbers of neurons and selves around something)
astrocytes arranged against a microcystic
background, no significant cytologic Perivascular
atypia Uniformly arranged cells
Temporal lobe most common site Vessel center
CHAPTER 3 / TUMORS 23

Homer Wright C. Anaplastic ependymoma


Fibrillary core WHO grade III tumor
Neuroblastic differentiation Rare
Primitive neuroectodermal tumor Precise histologic criteria to distinguish
(PNET) from ordinary ependymoma is not well
B. Ependymoma defined
Increased mitoses
WHO grade II tumor
Nuclear and cellular pleomorphism
510% of tumors, children
Hypercellularity
12% of brain tumors, adult
Endothelial proliferation
Any ventricle (especially 4th) and spinal
cord (intramedullary) Focal necrosis frequent
71% infratentorial D. Subependymoma
Obstruction of CSF-related signs and WHO grade I tumor
symptoms at presentation Adults; males > females
Grossly: soft lobular, discrete mass, well Asymptomatic, most cases are an inciden-
demarcated tal finding at autopsy
Microscopic pathology Small, usually <0.5 cm
Extreme fibrillarity Nodular mass; soft, tan, solid
Nuclei round to oval, carrot-shaped Septum pellucidum, floor or lateral recess
of the 4th ventricle
Dense nuclear chromatin
Microscopic pathology
Slight nuclear pleomorphism
Islands of blue (nuclei) in a sea of
Infrequent mitoses pink (fibrillary background)
Perivascular pseudorosettes Nuclei-like ependymoma
True ependymal rosettes Mitoses rare, low cell proliferation
Blepharoplastsintracytoplasmic, basal marker indices
bodies of cilia, best seen ultrastruc- Calcification common, microcystic
turally change common
Variants: papillary, clear cell, tanycytic, GFAP positive
melanotic, lipomatous, giant cell, sig- No rosettes or pseudorosettes
net-ring cell; no difference in prognosis
associated with these variants Microcystic change common
May see heavy calcification/ossifica- Excellent prognosis, surgical removal usu-
tion/cartilage ally curative
GFAP and EMA positive More likely to recur if larger, hard to
resect, mixed with true ependymoma
Low cell proliferation labeling indices
E. Myxopapillary ependymoma
Clinical course
WHO grade I tumor
Better prognosis in adults, supraten-
Males > females
torial tumors, gross total resection
Average age of diagnosis: 3040 yr
Radiation to CNS axis
Lower back pain history
Grade is not as important as location Slow growing
and extent of resection
May be locally infiltrative, hemorrhagic,
Better differentiated lesions do better or lobular
Spinal better than intracranial tumors 25% encapsulated
24 NEUROPATHOLOGY REVIEW

Filum terminale A variety of benign histologic subtypes


Rarely subcutaneous sacrococcygeal defined (WHO grade I tumors)
region Meningothelial (syncytial) cells
Microscopic pathology arranged in lobules divided by collage-
nous septae, whorling of cells around
Mucoid stroma, mucicarmine, and
vessels or psammoma bodies (calcifica-
alcian blue positive
tions), eosinophilic cytoplasmic protru-
Intracellular mucin sions into the nuclei prominent (nuclear
Recapitulate normal filum terminale pseudoinclusions)
Fibrous (fibroblastic)spindle cells
Papillary appearance with central vessel
with abundant collagen or reticulin
GFAP positive between cells
Some atypia and rare mitoses Transitional (mixed)features between
Clinical course meningothelial and fibrous types
Recurrence 1015% Psammomatousabundant calcifica-
tions
Good prognosis
Angiomatousmarked by prominent
Radiation therapy (if not completely vasculature, not to be confused with
excised) hemangiopericytoma or hemangio-
Mean survival 19 yr after total excision blastoma
in one large series Microcysticloose, mucinous back-
IV. Dural-Based Tumors ground
A. Meningioma Secretory (pseudopsammomatous)for-
1326% of primary intracranial tumors, mation of intracellular lumina contain-
incidence 6/100,000 ing PAS positive, eosinophilic material
Associated with neurofibromatosis type II Lymphoplasmacyte richmarked by
Any age, most common in middle-aged extensive chronic inflammation
and elderly Metaplasticdemonstrates mesenchy-
Females > males mal differentiation (bone, cartilage, fat)
Associated with breast carcinoma Marked by low rate of cell proliferation
May express progesterone > estrogen Immunohistochemistry: most EMA posi-
receptors tive and vimentin positive; variable positi-
vity with S-100 protein and cytokeratins
Most arise proximal to meninges from
arachnoidal cap cells; falx cerebri is single Electron microscopy shows prominent
most common site of origin interdigitated cell processes and desmoso-
mal junctions
Rarely arise in unusual sites such as intra-
ventricular region, head and neck region, May be induced by prior radiation
or lung Most common cytogenetic abnormality is
Most are slow growing and present with deletion of chromosome 22; other less fre-
signs and symptoms related to compres- quent abnormalities are deletion/loss on
sion of adjacent structures 1p, 6q, 9q, 10q, 14q, 17p, and 18q
Grossly, most are well circumscribed; com- Potentially curable with gross total resec-
press adjacent parenchyma tion, a subset recurs over time
May invade skull and produce hyperos- B. Atypical and malignant meningiomas
tosis Four histologic subtypes associated with
Gross appearance may be variable, depend- more aggressive behavior
ing on histologic type Chordoidareas similar to chordoma
CHAPTER 3 / TUMORS 25

with trabeculae of eosinophilic and vac- Derived from notochordal tissue rem-
uolated cells in a myxoid background, nants (ecchordosis physaliphora)
WHO grade II tumor
Most arise in spheno-occipital region or
Clear cellcells with clear, glycogen- clivus and sacrococcygeal region
rich cytoplasm, WHO grade II tumor Bone-based tumors, lobulated, may
Papillaryperivascular pseudopapillary appear grossly mucoid
pattern, WHO grade III tumor May contain cartilage (chondroid
Rhabdoidrhabdoid-like cells with chordoma)
eccentric nuclei, prominent nucleoli and Microscopically consists of epithelioid
cytoplasmic inclusions of intermediate cells arrayed against a mucoid matrix
filaments, WHO grade III
Some cells with marked vacuolation
Atypical meningioma marked by increased (physaliphorous or bubble cells)
mitoses (4 or more mitotic figures/10 high
power fields or 0.16 mm2) or 3 or more of Immunoreactive for vimentin, cytokera-
the following features: increased cellular- tin, EMA, S-100 protein
ity, small cells, prominent nucleoli, sheet- Locally recur, occasionally metastasize
like patternless growth pattern, and foci of Gross total resection and young age
necrosis; WHO grade II tumor, more com-
(<40 yr) do better
mon in males; 4.77.2% of meningiomas,
more likely to recur than grade I tumors. D. Hemangiopericytoma
Malignant (anaplastic meningioma) Relatively rare sarcoma of CNS
marked by histological features of frank Men > women
malignancy, 20 or more mitotic figures Mean age of diagnosis 4050 yr
per 10 high power fields (0.16 mm2), brain May cause lytic destruction of adjacent
invasion (rarely seen in lower-grade bone, no hyperostosis
lesions), or metastases; WHO grade III
Grossly a solid, well-demarcated mass,
tumor, median survival less than 2 yr
tends to bleed during removal
C. Mesenchymal, nonmeningothelial tumors
WHO grade II or III tumors
Includes a variety of benign and malignant Histologically, highly cellular
lesions, histologically resembling their
extra CNS counterparts Cells randomly oriented
Numerous staghorn contoured vessels
Benign lesions one can see include lipoma
(adipose), angiolipoma (adipose and promi- Reticulin rich
nent vasculature), hibernoma (brown fat), Calcification/psammoma bodies not seen
fibromatosis, solitary fibrous tumor (CD34 Variably see mitotic figures, necrosis
positive), benign fibrous histiocytoma
Immunohistochemistry: most stain with
(fibrous xanthoma), leiomyoma (smooth
vimentin, CD34; EMA negative
muscle, AIDS associated), rhabdomyoma
(striated muscle), chondroma (cartilage), May be pericytic in origin
osteoma (bone), osteochondroma (bone), Decreased survival has been associated
hemangioma (capillary or cavernous) and with increased mitosis (5 or more mitotic
epithelioid hemangioendothelioma figures/10 high-power fields), high cellular-
Malignant lesions one can encounter ity, nuclear pleomorphism, hemorrhage
include a variety of sarcomas (lipo- and necrosis
sarcoma, fibrosarcoma, malignant fibrous Treated by surgery and radiation, most
histiocytoma, leiomyosarcoma, rhabdomyo- eventually recur, 6070% eventually
sarcoma, osteosarcoma, chondrosarcoma, metastasize
angiosarcoma, and Kaposis sarcoma) E. Melanocytic lesions
Chordoma Melanocytoma
26 NEUROPATHOLOGY REVIEW

0.060.1% of brain tumors V. Choroid Plexus Tumors


Any age, females > males A. Choroid plexus papilloma
Most present in the 1st two decades of life
Monomorphic spindled or epithelioid
cells rich in melanin Lateral ventricle in children
4th ventricle more common in adults
Mitotic activity low, minimal necrosis
Presentation often related to CSF obstruc-
May recur locally tion symptoms
Diffuse melanocytosis Circumscribed, papillary mass
Children WHO grade I tumor
Leptomeningeal based proliferation of Pathologically marked by a hyperplasia of
nevoid cells bland epithelial-type cells on collagen
vascular cores
Associated with neurocutaneous melano-
Not much atypia, mitotic activity, necro-
sis syndrome
sis, or brain invasion
Poor prognosis May rarely contain metaplastic elements
Primary meningeal melanoma (bone, cartilage) or demonstrate oncocytic
change or mucinous degeneration
Very rare
Most stain with antibodies to cytokeratins,
Marked by cytologic atypia, high vimentin, S-100 protein; up to half stain
mitotic counts, necrosis, hemorrhage focally with GFAP
Poor prognosis Can seed cells in the CSF space
F. Hemangioblastoma Low rates of cell proliferation
May not necessarily be dural based Associated with SV40 DNA sequences
Young adults Associated with LiFraumeni syndrome
Cerebellum most common site, less com- Potentially curable by surgery with 5-yr
monly brain stem and spinal cord survival rate of up to 100%
Cyst with enhancing mural nodule appear- Occasional tumors with worrisome histol-
ance radiographically ogy (not enough to call carcinoma) desig-
nated as atypical choroid plexus papilloma
Associated with von Hippel-Lindau dis-
ease (chromosome 3p) B. Choroid plexus carcinoma
Associated with secondary polycythemia About 80% arise in children
vera (produces erythropoietin) Same locations as papilloma
Uncertain histogenesis WHO grade III tumor
WHO grade I tumor Grossly invasive, solid, and necrotic
Histologically marked by rich capillary net- Histologically demonstrates features of
work and intermixed vacuolated stromal frank malignancyatypia, increased mito-
cells ses, solid areas, necrosis, and brain
invasion
May show cystic changes and nuclear pleo-
morphism High rates of cell proliferation
Mitoses and necrosis not prominent 5-yr survival rate of about 40%
VI. Embryonal Tumors
EMA negative by immunohistochemistry
(to distinguish from EMA-positive meta- A. Myoepithelioma
static renal cell carcinoma in von Hippel- Rare, children 6 mo5 yr
Lindau setting) Most commonly arise in periventricular
Prognosis generally good region in cerebral hemispheres
CHAPTER 3 / TUMORS 27

Often large in size and well circumscribed Large cell variant marked by large, pleo-
at presentation morphic cells with prominent nucleoli
WHO grade IV tumor Medullomyoblastoma marked by focal
Mimics histologically the embryonic neu- myogenic differentiation
ral tube Melanotic medulloblastoma marked by
Papillary, tubular, or trabecular arrange- melanotic cells, often epithelial in appear-
ments of neuroepithelial cells which may ance and forming tubules
demonstrate neural, glial, or mesenchymal Many stain with synaptophysin antibody;
differentiation subset GFAP positive
Mitoses abundant, necrosis common Cytogenetics: about half with isochromo-
Generally poor prognosis, rapidly grow- some 17q, loss of part of 17p in 3045%,
ing, radiation may provide some benefit loss on 1q and 10q in 2040%
B. Ependymoblastoma 5-yr survival5070%
Rare, young children Poor prognosis associated with young age
(<3 yr), metastasis at presentation, subtotal
Most supratentorial, large in size
resection, large cell variant, melanotic
WHO grade IV tumor variant
Dense cellularity D. Cerebral neuroblastoma
Multilayered rosettes, outer cells of AKA: Supratentorial primitive neuroecto-
rosettes merge with adjacent undifferenti- dermal tumor (PNET)
ated embryonal tumor
Most arise in 1st decade
Prominent mitoses
Presentation with seizures, disturbances of
Grow rapidly, disseminate readily, poor consciousness, focal motor deficits
prognosis
WHO grade IV tumor
C. Medulloblastoma
Histologically similar in appearance to
Peak in 1st decade, 70% occur in children medulloblastoma
<16 yr More than half calcified
Males > females May contain areas with terminally differen-
Most arise in vermis and project into 4th tiated cells (ganglion cells)ganglioneuro-
ventricle blastoma
Clinical presentation most commonly 5-yr survival (3040%)
ataxia, gait disturbance, headache, vom- E. Atypical teratoid/rhabdoid tumor
iting
Most arise in infants or children
WHO grade IV tumor
Slightly more common in males
Typical histopathology marked by densely
packed cells with high nuclear-to-cyto- About half arise in posterior fossa
plasmic ratio About a third present with metastases
Homer Wright pseudorosettes <40% of throughout CSF axis
cases WHO grade IV tumor
Mitoses frequent, apoptosis frequent Histologically marked by rhabdoid cells
with eosinophilic cytoplasm, eccentric
Necrosis common
nucleus, prominent nucleolus
Desmoplastic variant marked by nodular,
Cytoplasmic pink bodycorresponds to
reticulin-free zones (pale islands) sur-
collections of intermediate filaments
rounded by more densely cellular areas
(reticulin rich) May contain a small cell embryonal com-
ponent or mesenchymal component
Nodular variant marked by intranodular
nuclear uniformity, cell streaming, neuro- Mitoses and necrosis common
cytic-like cells Rhabdoid cells by immunohistochemistry
28 NEUROPATHOLOGY REVIEW

express EMA and vimentin; variably Painful masses


express neurofilament protein, GFAP, Associated with neurofibromatosis type I
smooth-muscle actin and cytokeratin
immunoreactivity WHO grade I tumor
90% demonstrate monosomy or deletion Grossly may present as a cutaneous nod-
on chromosome 22 ule or as a diffuse lesion; plexiform
tumors are multinodular bag of worms
Most patients die within 1 yr of diagnosis
Histologically composed of an admixture
VII. Peripheral Nerve Tumors
of Schwann cells, perineurial-like cells
A. Schwannoma and fibroblasts in a mucoid/loose matrix
AKA: neurilemmoma, neurinoma, acoustic Mitoses rare, atypia usually not prominent,
neuroma cellularity typically low
Arises from peripheral nerve May contain tactile-like structures
8% of intracranial and 29% of primary spi- (Wagner-Meissner-like corpuscles)
nal cord tumors
Scattered S-100 positivity
Associated with neurofibromatosis type I
Plexiform tumors and those arising in
Any age, peak 4th6th decades large nerves may give rise to malignant
WHO grade I lesion peripheral nerve sheath tumors (MPNST)
Many are encapsulated, globoid masses AKA: neurogenic sarcoma, neurofibro-
Composed microscopically of spindled sarcoma, malignant schwannoma
cells with alternating areas of compact About half of MPNST arise in neurofi-
cells (Antoni A pattern) and less cellular
bromatosis I
looser regions (Antoni B pattern)
Cell nuclei with tapered ends Histologically marked by hypercellular-
ity, necrosis, increased mitoses
May demonstrate nuclear pleomorphism
(ancient change) Frequently invasive
May show nuclear palisading Verocay Variant types include epithelioid, glan-
bodies dular, and Triton tumor (rhabdomyo-
May contain lipid-like cells or undergo sarcomatous differentiation)
cystic degeneration Poor prognosis, 5-yr survival about
Frequently has thickened vessel walls 35%
Cellular schwannoma variant C. Perineurioma
hypercellular, mostly Antoni A pattern Rare benign tumor comprised of perineu-
Melanotic schwannomagrossly and micro- rial cells
scopically pigmented (melanin), about Presents in adolescence or young adults
50% of tumors with psammoma bodies
associated with Carney complex (autoso- Progressive muscle weakness (intraneural
mal dominant disorder marked by cardiac tumors) and mass effect (soft tissue
myxomas, endocrine abnormalities and pig- tumors)
mented skin lesions), about 10% follow a Intraneural tumors present with segmental
malignant course nerve enlargement
Plexiform schwannomaassociated with Microscopically see proliferating perineu-
neurofibromatosis type II rial cells in endoneurium around nerve
Immunohistochemistrystrong and dif- fibers (pseudo-onion bulbs)
fuse staining with S-100 protein Soft-tissue tumors comprised of spindled,
Slow growing, benign tumors wavy cells
B. Neurofibroma EMA positive by immunohistochemistry
Any age Benign clinical course
CHAPTER 3 / TUMORS 29

D. Neuromas Gene rearrangement studies by polymerase


Traumatic or amputation neuroma marked chain reaction may help in identifying
by fibrosis and disorderly arrangement of tumors where the number of cells present
nerve twigs/fibers is low
Mortons neuroma thickening and degener- Favorable prognosis in patients with single
ation of interdigital nerve of foot, presents lesion, absence of meningeal or periven-
with pain, histologically digital artery wall tricular tumor, immunocompetent, age <60
thickened occluded with fibrosis of yr, and preoperative Karnofsky score >70
nerve 85% response rate, median survival 1745
VIII. Hematopoietic Tumors mo
A. Lymphoma Very steroid-responsive initially
Increasing incidence in recent years B. Histiocytic Tumors
Associated with AIDS Langerhans cell histiocytosis
All ages, peak 6th7th decades in immuno- Children
competent persons
Males > females Multifocal involvement of bone and
brain (Hand-Schuller-Christian disease)
60% supratentorial or skin, nodes, viscera, and rarely CNS
2550% multiple (6085% in AIDS and (Letterer-Siwe disease)
posttransplant patients)
Grossly appear as yellow or white dural
Most present with focal neurological defi- nodules or granular parenchymal infil-
cits or neuropsychiatric symptoms trate
CSF cytology pleocytosis observed in
3560% of patients, cytology diagnostic in Granulomatous infiltrates marked by
530% of primary CNS lymphomas, and Langerhans cell histiocytes (eccentric
7095% of metastatic lymphomas (which nuclei, linear grooved nucleus, incon-
tend to be leptomeningeal based) spicuous nucleoli, CD1a positive,
Birbeck granules by electron micros-
Histologically marked by angiocentric, copy34-nm-wide rod-shaped or
atypical lymphoid cell infiltrate tennis-racket-shaped intracytoplasmic
Most primary CNS lymphomas are non pentalaminar structures), lymphocytes,
Hodgkins lymphomas, diffuse large cell plasma cells and eosinophils
type, B-cell immunophenotype (CD20,
CD79a positive) Overall survival 88% at 5 yr

Tumor cells may be admixed with vari- Non-Langerhans cell histiocytosis


able numbers of smaller tumor infiltrating Macrophage differentiation, absence of
lymphocytes (T-cells) features of dendritic Langerhans cell
About 2% are T-cell lymphomas
Rosai-Dorfman diseaseadults, dural
Other rare lymphoma types to involve based, sheets or nodules of histiocytes
CNS include angiotropic large cell lym- (CD1a negative, CD11c positive, CD68
phoma or intravascular lymphoma (cells positive)
are confined primarily to vascular lumens,
large B-cells), Hodgkins lymphoma (Reed Erdheim-Chester diseaseadults, lipid-
Sternberg cellsCD30 and CD15 posi- laden macrophages (CD1a negative,
tive) and mucosa-associated lymphoid tis- CD68 positive, S-100 protein negative)
sue (MALT) lymphoma of dura (low- and multinucleated Touton giant cells
grade B-cell lymphoma; CD20 and CD79a Hemophagocytic lymphohistiocytosis
positive; CD5, CD10, and CD23 negative) autosomal recessive, infants; infiltrates
Epstein-Barr virus genome present in most of lymphocytes and macrophages with
tumors in immunocompromised patients hemophagocytosis and variable necrosis
30 NEUROPATHOLOGY REVIEW

IX. Cysts Fibrous capsule containing gelatinous


A. Colloid Cyst fluid, no epithelial lining
Third ventricle H. Dermoid and epidermoid cysts
Young to middle-aged adults Arise from embryologically misplaced epi-
thelial cells either in meninges, ventricles,
Headaches, transient paralysis of lower
or brain parenchyma
extremities, incontinence, personality
changes Epidermoid: cerebellopontine angle most
common site, may occur in other sites
Histologically marked by a single layer of (i.e., suprasellar)
columnar epithelial cells, may contain gob-
let and ciliated cells Dermoid: posterior fossa, midline lesions,
4th ventricle
Amorphous, brightly eosinophilic cyst
fluid Macroscopic pathology
May see filamentous-like masses of nucleo- Epidermoid: smooth, translucent cyst,
protein when open has flaky contents
Positive for EMA and cytokeratin markers Dermoid: thicker capsule with piloseba-
by immunohistochemistry ceous contents
B. Rathkes cleft cyst Microscopic pathology
Intrasellar or suprasellar Epidermoid: keratin debris, keratohya-
Often incidental finding, larger ones may line granules, and delicate epithelial
be symptomatic causing visual distur- lining
bances or abnormalities of hypothalamic Dermoid: epithelial-lined cyst with
pituitary function sweat and sebaceous glands
Histologically lined by ciliated columnar Cysts can rupture and elicit a foreign-
epithelium, may see squamous metaplasia body giant-cell reaction to their con-
C. Endodermal cyst tents
Any age, many are intraspinal X. Miscellaneous Tumors
AKA: enterogenous, neurenteric, entero- A. Paraganglioma
genic cysts Most present as spinal intradural tumors in
Histologically consists of cuboidal to the cauda equina region
columnar epithelium lying on fibrovascu- Intracranial tumors usually represent exten-
lar capsule sions of jugulotympanic tumors
D. Ependymal cystintraparenchymal or peri- Mean age 4050 yr, males > females
ventricular cyst lined by ependymal type
cells Often present with lower back pain
E. Choroid plexus cystepithelial-lined cyst of WHO grade I tumor
the choroid plexus Composed of chief cells arranged in nests
F. Arachnoid cyst or lobules (zellballen) surrounded by a sin-
gle layer of sustentacular cells
Subarachnoid space in the temporal lobe
region is favored site May encounter mild pleomorphism, occa-
sional mitotic figures, and rarely foci of
Delicate, translucent wall filled with clear hemorrhagic necrosis
fluid
Chief cells stain with synaptophysin, chro-
EMA positive by immunohistochemistry mogranin, and neurofilament proteins
G. Synovial cysts of spine Sustentacular cells stain with S-100
AKA: ganglion cyst protein
Intradural, extramedullary lesions associ- Most in cauda equina are slow growing
ated with degenerative joint disease and potentially curable by total excision
CHAPTER 3 / TUMORS 31

Cannot predict behavior of tumor based Cut section: necrotic debris, calcium,
on histology cystic oil often with cholesterol
B. Pineocytoma Spillage of the oil may result in chemi-
Most common in adolescence and cal meningitis
adulthood Microscopic pathology
Less prone than pineoblastoma to seed the Cells form compact masses or line
CSF
cysts
Survival information scanty
Assume an adamantinomatous pattern:
Microscopic pathology peripheral palisading of epithelial cells
Lobular, small cells admixed with a loose stellate retic-
Largely hypocellular zones of fibrillar- ulum
ity that resemble Homer-Wright Keratin-containing cells
rosettes (pineocytomatous rosettes) Histiocytes, calcium, and cholesterol
Fibrillary zones have neural nature (syn- clefts seen
aptophysin, neuron-specific enolase, Surrounding brain has intense gliosis
and neurofilament protein positive) and Rosenthal fiber formation
C. Germ cell neoplasms E. Pituitary adenoma
Occur along the midline, most commonly About 1020% of all intracranial neo-
suprasellar region and pineal plasms
Suprasellar tumors equally divided among Most common in adults
males and females, pineal tumors with
Women most often affected
male predominance
Seen incidentally in approximately 25% of
Most occur in 1st three decades
autopsies
Common signs and symptoms: headache,
Microadenoma <1 cm in diameter
papilledema, mental status changes, gaze
palsies, precocious puberty Symptoms: related to endocrine dysfunc-
tion or visual disturbance
Diffuse dissemination occasionally seen
Most common secreting adenomapro-
Histologically the same as germ cell
lactinoma
tumors of the ovary or testis (germinoma,
teratoma, yolk sac tumor, embryonal carci- Microscopic pathology
noma, choriocarcinoma) Heterogeneous histologic appearance
Germinoma is the most common CNS Patterned sheets of uniform cells with a
germ cell tumor delicate vascular network
D. Craniopharyngioma Nuclei are round and have a delicate
Rare tumors of the sellar region chromatin pattern
Two peaks: 1st two decades and the Cells often have granular cytoplasm
6th7th decades
Lacks the normal acinar or lobular pat-
Local recurrence tern of the pituitary
Presents with disturbances in the hypo- Monotonous cell types
thalamicpituitary axis, visual symp-
Immunohistochemical staining for pitu-
toms, CSF symptoms (obstruction)
itary hormones will identify the secre-
May originate from squamous cells at tory products
the base of the infundibular stalk Rare examples of pituitary carcinoma
Macroscopic pathology (defined by noncontiguous spread)
Fill suprasellar region, displace or Apoplexysudden enlargement of ade-
attach to cranial nerves and vessels noma related to infarct/hemorrhage
32 NEUROPATHOLOGY REVIEW

F. Metastatic disease Ependymoma (near ventricle)


Common Meningioma/sarcoma (dural based)
Typically found in patients 5th7th Lymphoma
decades Neuroblastoma
Often multiple, may be ring-enhancing B. Corpus callosum
lesions radiographically Astrocytoma
Associated with prominent edema Oligodendroglioma
Most common in descending order of inci- Lipoma
dence: lung, breast, melanoma, renal cell,
gastrointestinal tumors C. Lateral ventricle
Ependymoma
Common in all areas of brain
Choroid plexus papilloma
70% of patients have more than one metas-
tasis Subependymoma
Distributed in arterial watershed zones Central neurocytoma
Associated with edema, which may be out D. Third ventricle
of proportion to tumor size Ependymoma
Frequently at the graywhite junction Colloid cyst
Discrete, sharply demarcated from adja- Pilocytic astrocytoma (adjacent)
cent brain grossly E. Fourth ventricle
Microscopic pathology Ependymoma
Distinctive nuclear characteristics, Choroid plexus papilloma
resembling the primary tumor Meningioma
Vascular proliferation uncommon F. Optic nerve/chiasm
Immunohistochemical staining with Astrocytoma
cytokeratins often positive and dis- Meningioma
tinctive G. Pituitary/sella
Cells often cohesive Adenoma
G. Lipoma Craniopharyngioma
Usually midline: corpus callosum, 3rd ven- Germ cell tumor
tricular region, cerebellopontine (CP) Meningioma
angle, quadrigeminal area
Rathkes cleft cyst
May be associated with developmental
H. Cerebellum
abnormalities (i.e., agenesis of the corpus
callosum) Hemangioblastoma
Usually incidental autopsy findings; rarely Medulloblastoma
associated with seizures Astrocytoma
Color (yellow) is essential feature and cal- Metastasis
cification may be seen peripherally Dermoid cyst
Composed of mature fat and blood vessels I. Pineal gland
XI. Common Tumors by Location Germ cell tumor (especially germinoma)
A. Cerebral hemisphere Pineocytoma
Astrocytoma (diffuse, fibrillary type) Pineoblastoma
Oligodendroglioma J. Cerebellopontine angle
Metastatic carcinoma Schwannoma
Glioneuronal tumors Meningioma
CHAPTER 3 / TUMORS 33

Epidermoid cyst Intradural, extramedullary


Ependymoma Schwannoma
Choroid plexus papilloma Meningioma
K. Brain stem Myxopapillary ependymoma
Astrocytomadiffuse, fibrillary type ver-
Paraganglioma
sus pilocytic
L. Spinal cord Chordoma
Intramedullary Extradural
Metastasis
Ependymoma
Lymphoma
Astrocytoma
Myeloma
4 Trauma

I. Herniation Syndromes Enlarged extracellular spaces with peri-


A. General information vascular astrocyte foot process swelling
Herniation is a significant cause of CNS Cytotoxic
morbidity and mortality. Result of impairment of cell Na+-K+ mem-
Alterations in intracranial dynamics causing brane pump
increased intracranial pressure (ICP) operate Intracellular swelling mainly in gray
by a variety of mechanisms matter
Rapid growth, rather than size of growth Hypoxia most common cause
often the more critical factor
No enlargement of extracellular spaces
Location of the mass relative to CSF and D. Lateral transtentorial herniation
venous outflow is important
AKA: parahippocampal/uncal herniation
Increased ICP also related to tissue
Usually occurs with lateral supratentorial
edema masses
B. Generalized brain swelling (edema)increase Earliest change is displacement of the uncus
in brain volume because of increased tissue and parahippocampal gyrus beneath the ten-
water content torial notch
Gyri flattened
Eventually hemorrhagic necrosis occurs
Narrow sulci
Herniating tissue alters the relationship of
Prominent pacchionian granulations structures to the tentorium
In chronic cases, bony erosion may result
Kernohans notchdamage to the contra-
Increased brain weight lateral cerebral peduncle
Obstruction of subarachnoid space and Compression of the ispilateral 3rd nerve
decreased ventricular size.
Posterior cerebral artery is trapped by the
May be associated with congestive brain
parahippocampal gyrus and the ipsilateral
swellingincrease in cerebral blood vol-
cerebral peduncle, leading to infarction in
ume in capillary and postcapillary beds
the calcarine cortex
C. Types of cerebral edema
Midline brain stem hemorrhage, Duret
Vasogenic hemorrhage
Result of blood-brain barrier breakdown E. Central herniation
Around tumors or trauma, diffuse in Occurs with parenchymal lesions in the fron-
white matter tal and parietal lobes

35
36 NEUROPATHOLOGY REVIEW

Extracerebral lesions near the vertex may geal artery; secondary to a fracture (in 80
also be responsible 85%)
Downward displacement of the hemispheres In children, may be the result of impact
results in herniation of the thalamus and alone without skull fracture
midbrain Amount of bleeding results from a variety
Compression of the anterior choroidal of factors
artery with subsequent infarction of the Adherence of the dura to the inner table
globus pallidus and optic tract of the skull
F. Subfalcial herniation Depth of the meningeal arterial groove
AKA: cingulate or supracallosal herniation Relationship of the fracture to the vessels
Usually associated with mass lesions in the One-third of patients with epidural hema-
anterior portion of the cerebrum (frontal or toma have other traumatic lesions
parietal lobe)
Epidural hematomas may deform the under-
Displacement of the cingulate gyrus under lying brain with subsequent mass effect
the falx cerebri
Majority are acute and potentially lethal if
Anterior cerebral artery may be displaced not evacuated
with resultant infarction
B. Subdural hematoma
G. Tonsillar herniation
More common than epidural hematoma;
Cerebellar tonsils are displaced downward may produce symptoms referable to mass
into the foramen magnum effect or may present with fluctuating neuro-
Tonsillar tips are elongated and flattened logic signs and headache
against the medulla Result of tearing of bridging veins which
Prolonged tonsillar compression leads to transverse through the subarachnoid space
hemorrhagic necrosis of the tonsils Acute subdural hemorrhage associated with
H. Upward herniation skull fracture (50%) and diffuse axonal
Result of an expanding posterior fossa injury; high mortality (4060%) if subdural
lesion bleed and diffuse axonal injury combined
Upward protrusion of the cerebellum Amount of blood that is significant depen-
through the tentorial notch with grooving of dent on age and rapidity of accumulation
the anterior vermis and the lateral lobes Infanta few milliliters
May cause compression of the superior Children (13 yr)30 mL
cerebellar artery with infarction Adult100150 mL
May displace the overlying vein of Galen May be categorized based on the interval
I. External herniation between the hematoma and the traumatic
Displacement of brain tissue through a event
defect in skull (often caused by trauma or Acute: within 3 d of trauma
surgery)
Subacute: between 3 d and 3 wk post-
Hemorrhagic infarct at edge of herniated trauma
tissue
Chronic: develops after 3 wk
II. Traumatic Hemorrhage
Microscopic dating of subdural hematoma
A. Epidural hematoma
1 wklysis of clot
Commonly seen post-trauma, occurs in
about 3% of significant head injuries 2 wkgrowth of fibroblasts from the
Most are frontal or temporal and associated dural surface into the hematoma
with a laceration of one of the meningeal 13 mosearly development of hyalin-
arteries, most commonly the middle menin- ized connective tissue
CHAPTER 4 / TRAUMA 37

C. Subarachnoid hemorrhage based on the presence or absence of an


May be related to trauma overlying scalp laceration
Nontraumatic causes most commonly the Most common in the frontal or parietal
result of saccular aneurysm and vascular region
malformation Rule of thumb: If the fracture is below the
May be secondary to contiguous rupture of hat brim, it is more likely to be secondary
intraparenchymal hemorrhage to a fall; a fracture above the hat brim level
is more likely secondary to traumatic force
Traumatic basilar subarachnoid hemorrhage
following a blow to the face, jaw, or Two special types of depressed skull frac-
neckresult of tear of vertebral artery at ture: orbital blow-in and blow-out fractures
C1 Occur in elderly people who have
III. Skull Fractures sustained a blow to the back or top of
the head
A. Classification
Will be present, by definition, in an open or May represent contrecoup movement of
penetrating head wound the frontal lobes over the thin bones in
the frontal fossa
May be present in up to 80% of fatal closed
head injuries D. Basilar skull fractures
Common, difficult to diagnose by x-ray
Fractures should be located accurately (i.e.,
calvarium, base of skull, facial bones) and Must strip dura from the skull base to visu-
defined as to the type alize, when basilar fracture is suspected at
autopsy
Linear
Result of significant force from falls, motor
Depressed (displaced) vehicle accidents, or assaults
Diastatic Subtypes include hinge, ring, and contra-
Open or closed coup fractures
Battle signsoft tissue swelling and discol-
Comminutedbone fragmented oration in the area of the mastoid, second-
Compound fracture with skull laceration ary to blood in the mastoid air cells
overlying the fracture Raccoon eyesrepresents periorbital ecchy-
B. Linear skull fractures moses, may be seen with basilar skull frac-
AKA bursting fractures tures
Make up the majority of skull fractures E. Hinge fractures
Secondary to the outbending of the skull Special type of fracture
at a point distant from the traumatic impact Occur with crushing injuries, such as com-
pression of the head between the ground
Frequently seen in falls and traffic accidents
and a heavy object (i.e., a car tire)
Diastatic fracturessubset of linear frac-
May be either longitudinal or transverse
tures occurring in young people, defined as
linear fractures that cause separation of or Transverse hinge fracture extends across the
across cranial sutures dorsum sellae of the skull
C. Depressed skull fractures IV. Contusions
Secondary to low-velocity local traumatic A. General characteristics
forces, also called bending fractures Most characteristic lesions of trauma
Subdivided into stellate and comminuted Contusions are parenchymal brain injuries
Depressed skull fracture present if the inner Typically wedge shaped and preferen-
table of the skull is depressed by a distance tially affect the crown of the gyri
equal to the thickness of the skull or greater Most common sites frontal and temporal
Further subdivided into open or closed poles
38 NEUROPATHOLOGY REVIEW

In many contusions, the pialglial mem- Cavitation


brane disrupted; by definition, the contu- Secondary missile productionpieces of
sion then accompanied by a laceration bone break off
A fracture contusion may develop at site of Gunshot wounds
fracture
Wounding capacity related to velocity of
Hernation contusion represents bruising of
missile
the uncus and/or parahippocampal gyrus as
it impacts on the dura of the tentorial notch; Tissue damage resulting from track of
produced by sudden, forceful herniation of bullet and temporary cavity that follows
brain tissue the bullet (damage from stretching in
Gliding contusionfocal hemorrhage in cor- adjacent tissue)
tex and underlying white matter at the supe- Entrance wound through skull will have
rior margins of the cerebral hemispheres, a broader inner table bevel; exit will
usually bilateral but asymmetrical have a broader outer table bevel
B. Coup and contrecoup contusions B. Blunt head injury
Coup lesionscontusions immediately Static loadingforces are applied to the
beneath and associated with direct trauma head gradually (more than 200 ms to
Stationary head impacted by object that develop) (e.g., earthquakes or landslides)
deforms the skull and impacts enough Dynamic loadingforces are applied < 200
energy to damage the underlying brain ms
surface Impulsive loadinghead is set in motion
Skull must be intact (i.e., no fractures) or moving head is stopped without being
struck (blows to face and thorax)
Contrecoup lesionscontusions at a dis-
tance from and frequently roughly opposite Impact loadingblunt object strikes
to the point of trauma head, injury often related to both contact
and inertia factors
Often represent rotational brain move-
ment and deceleration injury Strain caused by loading is the cause of
tissue injury as a result of compression,
Thought to be related to the irregularities
tension, or shear
of the skull opposite the point of impact
May be accompanied by skin lesions
Burst lobepulped frontal or temporal
pole resulting from massive contrecoup Abrasionsscraping away of skin sur-
contusion, associated with subdural hem- face (scratches, brush burns)
orrhage Contusionsbruise (extravasation of
V. Other Lesions blood into soft tissue)
A. Penetrating versus perforation wounds Ecchymosesextravasation of blood
Penetrating brain woundsthe object enters from one site to another
but does not completely transverse the brain Periorbital (raccoon eyes)fractures
(stab wounds, low-velocity gunshot) of orbital plates of the anterior cranial
Perforating brain woundswounds in fossa
which the object passes through the brain Mastoid (battle sign) fracture of
and exits petrous bone lacerating the internal
Factors to keep in mind related to brain carotid artery
wounds: Lacerationsplitting or tearing of the
Entrance wound (marginal abrasion, muz- tissue
zle imprint, soot, stippling) Crushing head injurystationary head
impacted by massive force that crushes the
Exit wound
skull, brain often lacerated by bone frag-
Internal ricochetlower-velocity bullets ments and fracture contusions
CHAPTER 4 / TRAUMA 39

Infant brains Punctate or streak-like hemorrhages


Smooth skull base and pliable skull Axonal injuryretraction balls or spheroids
Contusion tearsseparation of cortex (seen on silver stain), seen as early as 412
from subcortical white matter h after injury

Coup and contrecoup contusions seldom May get avulsion of the pontomedullary or
cervicomedullary junction with severe hyper-
seen in children before age 34 yr
extension of neck
C. Diffuse axonal injury
May result from angular acceleration
AKA: intermediary coup lesions, Strich
lesions D. Fat emboli
Shearing injury of nerve fibers Fracture of long bones
Associated with traffic accidents (frontal, Fat globules in circulation
occipital or vertex impacts) and shaken Often presents with sudden onset tachypnea,
infants dyspnea, and tachycardia 13 d after injury
Deformation of brain in the anterior
Petechial white matter hemorrhages
posterior direction
Need to process tissue fresh to stain with
Damage to corpus callosum, fornix, corona
oil-red-O or lipid stain
radiata, superior cerebellar peduncles
5 Congenital Malformations,
Perinatal Disease, and Phacomatoses

I. Congenital Malformations Protrusion of brain and meninges through


A. Disorders of closure congenital opening of skull
AKA: dysraphism Result of failure of overlying mesenchy-
Closure of neural tube (4th wk) mal tissue to develop
Craniorachischisis totalistotal failure of Posterior type (midoccipital, occipitocervi-
closure cal) most common in fatal cases
Brain and cord exposed to amniotic fluid Occipital encephalocele is part of
Meckel-Gruber syndromeautosomal
Necrosis and degeneration of brain and
recessive condition with sloping forehead,
cord polydactyly, polycystic kidneys, and
Anencephaly hepatic fibrosis
AKA: exencephalia acrania Spina bifida
Defective closure of anterior end of neu- Meningocelesubcutaneous herniation of
ral tube meninges through defective closure of ver-
Most aborted prematurely tebral arch

Females > males, high incidence in Ire- Meningomyeloceleherniation of both


land and Wales leptomeninges and cord through a verte-
bral defect
Associated cranial vault and scalp defects
Often associated with hydrocephalus,
Base of skull thickened, flattened; sphe- Chiari type II malformation, and other
noid wing abnormalities, shallow orbits spinal cord anomalies (syringomyelia,
Brain absent or rudimentary (cystic mass hydromyelia, diastematomyelia)
with vesselsarea cerebrovasculosa) Lesions above T12 level more likely
Eyes well formed (optic vesicles form associated with other malformations
very early) and more common in females

Cerebellum and brain stem may be pres- Myelocystocelemyelomeningocele with


ent, but usually absent central canal dilatation and thin posterior
cord
Absence or hypoplasia of pituitary (50%)
Myeloschisiscleft cord with interposed
Amyeliatotal absence of cord vascular tissue with remnants of neural
Cranium bifidum with encephalocele tissue

41
42 NEUROPATHOLOGY REVIEW

Diastematomyeliacord segments sepa- Associated craniofacial malformations


rated by collagen or bone, 50% associated Cyclopia
with myeloschisis and myelocystocele,
Nasal protuberance or proboscis above
rarely extends above the mid-thoracic
orbits
levels
Agnathia (absence of jaw)
Diplomyeliaduplication of the spinal
Fusion of ears
cord
Situs inversus
Aqueductal malformations
Cleft lip/palate
Stenosis without gliosis
Trigonocephaly
Forking
Three types
Septum (neuroglial membrane)obstruct- Alobar
ing caudal end
Most severe form
Gliosisnarrowing or subdivision of
Monoventricular small cerebrum
aqueduct
remains undivided into lobes
Acquired destructive process Thin membrane forms a portion of
Hydromyelia ventricle roof
Congenital dilatation of central canal Absent olfactory bulbs and tracts

Associated facionasal malformations


Associated with meningomyelocele and
Chiari malformation Brain stem and cerebellum generally
normal
Rarely symptomatic
Semilobar
Syringomyelia
Incomplete lobe formation
Slit or tubular cavitation of cord
Partial formation of a shallow inter-
(syrinx = tube)
hemispheric fissure
Most commonly cervical or thoracic cord Absent olfactory bulb tracts
Cavity may be multiple, irregular Associated facionasal malformation

May be filled with fluid associated with Lobar


compression and damage to surrounding Lobes and interhemispheric fissures
tissue well formed
Histologyvariable, cavity resembles Ocular abnormalities
tear, may see astrocytosis Usually normal karyotype, 1315
Associated with intramedullary tumors trisomy
Syringobulbia Chiari malformations
Cavity in medulla Type I (adult)
Downward herniation of cerebellar
Often associated with cervical syringo-
myelia tonsils
No elongation of medulla and 4th ven-
B. Disorders of diverticulation
tricle
Holoprosencephaly
No associated spina bifida
AKA: holotelencephaly (failure of ante-
Often asymptomatic, may cause late
rior telencephalon to divide [i.e. incom-
onset hydrocephalus
plete separation of the cerebral hemi-
Arachnoidal adhesions
spheres across the midline])
Associated with syringomyelia (50%)
Associated with trisomy 13 and increased
risk in diabetic mother Type II (infantile)
CHAPTER 5 / CONGENITAL MALFORMATIONS, PERINATAL DISEASE AND PHACOMATOSES 43

Malformation of basocranial bones Most develop by the 16th wk of gestation


Platybasia (flattened) Miller-Dieker syndromedel 17p13.3
Occipito-atlantal assimulation (LIS-1 gene, seizures, mental retardation,
(fusion) and agyria)
Small foramen magnum Gross patterns include agyria, pachygyria
Cervical spina bifida polymicrogyria, hemimegalencephaly, nod-
Klippel-Feil abnormality (vertebral ular and laminar heterotopias
fusion) All marked by an abnormal arrangement
Craniolacuniairregular patches of of neurons in the cortex
cranial vault thinning or erosion Several histologic types
Cerebellar malformations Laminar architecture abnormalities
Persistence of embryonic cervical Neuronal disorientation within cortex
flexure Hypercellular molecular layer
Foreshortening of hindbrain Balloon cells
Elongation and downward displace- Neuronal dysmorphism
ment of inferior vermis, medulla,
Pialarachnoidal glioneuronal hetero-
and cervical cord
topia
Associated features
Neuronal cytomegaly
Lumbar meningocele or meningomy-
elocele (95%) Association with tuberous sclerosis, neurofi-
bromatosis type I, epidermal nevus syn-
Hydrocephalus (95%)
drome
Microgyria (55%)
Association with neoplasia (ganglioglioma,
Tectal malformation (75%) dysembryoplastic neuroepithelial tumor
Aqueductal atresia (>35%) [DNT])
Type III Neuronal heterotopia
Very rare White matter neurons
Occipitocervical or high cervical bony Frequently seen in epilepsy patients
defect with herniation of cerebellum Significance uncertain
through the defect
D. Disorders of sulcation
Dandy-Walker syndrome
Normal development
Hypoplasia or aplasia of cerebellar
vermis 5th mofirst fissures form
Cystic dilatation of 4th ventricle 3rd trimestersecondary sulci form
Enlargement of posterior fossa 3rd trimester to 6 mo of agetertiary
Hydrocephalus symptoms early, promi- sulci form
nent occiput Lissencephaly (agyria) and pachygyria
Variable agenesis of corpus callosum, Agyriaabsence of convolution
neuronal heterotopia, spinal dysplasia,
Pachygyriafew broad gyri
renal defects, polydactylism, pachygy-
ria, polymicrogyria, microcephaly, Bilateral or unilateral
aqueduct stenosis, syringomyelia, men- Associated with neuronal migration abnor-
ingocele, spina bifida malities between 11th and 13th wk of ges-
Compatible with normal life tation
C. Disorders of neuronal migration Brain small, underweight (microcephalus
Cortical dysplasia vera)
44 NEUROPATHOLOGY REVIEW

Lateral ventricles larger (colpocephaly) Excessive weight (16002850 g), brain


Decrease in number of secondary gyri weight 2.5 standard deviations above the
mean for age and gender
Depth of gray matter underlying the
Heavy brain with associated anomalies
smooth part of cortex increased
Calvaria usually small, misshaped and Three types
thick Diffuse blastomatous glial over-
growthassociated with malignancy
Microscopic cortical dysplasia, four-layer
and dysplasia
cortex
Increased size without glial prolifer-
Often with heterotopias ation
Associated with small jaw, severe mental Increased size secondary to other dis-
and motor retardation, failure to thrive, ease (tuberous sclerosis, Tay-Sachs,
seizures leukodystrophies such as Alexanders
Walker-Warburg syndrome (HARD + E disease)
syndrome) Megalocephalyenlargement of head
Hydrocephalus, agyria, retinal dyspla- Microencephaly
sia, encephalocele
< 900 g weight in adults
Psychomotor retardation severe, death
in infancy Associated microcephaly (small head)

Cerebro-ocular dysplasiamuscular dys- Most the result of perinatal destructive


trophy syndrome processes
Similar to Walker-Warburg syndrome Familial type (500600 g), recessive
plus muscle disease defect
Muscular dystrophy Schizencephaly
Neu-Laxova syndrome Symmetrical clefts in line of primary fis-
Agyria with microcephaly
sures and involve whole depth of cerebral
wall
Autosomal recessive
Two types
May also see intrauterine growth retar-
Clefts intact
dation, skin dysplasia, organ hypopla-
sia, facial anomalies Internal hydrocephalus, clefts covered

Micropolygyria (polymicrogyria) by membrane


Miniature, incomplete convolution Clefts resulting from localized failure of
growth and differentiation in cerebral
Abnormal cortical architecture (four-layer wall before end of 2nd mo of gestation
cortex, absent molecular layer), thin corti-
cal ribbon F. Disorders of commissuration
Agenesis of corpus callosum
Associated with cerebellar microgyria
1222 wk of gestation
Associated with neuronal migration abnor-
malities between 20th and 24th wk of ges- Asymptomatic or associated with mental
tation. retardation, seizures, speech disturbances
Zellwegers syndrome Associated with holoprosencephaly,
Microgyria and polymicrogyria
pachygyria, schizencephaly
Peroxisome abnormality Associated with webbed toes, funnel
Chromosomes 1, 7, and 8
chest, spina bifida, persistent fontanelles,
facial asymmetry, high arched palate,
E. Disorders of proliferation agenesis of cerebellar vermis, ventricular
Megalencephaly septal defect
CHAPTER 5 / CONGENITAL MALFORMATIONS, PERINATAL DISEASE AND PHACOMATOSES 45

Abnormalities of septum pellucidum Cerebral hemispheres largely unaf-


Agenesis/hypoplasia fected
4th ventricle dilated
Associated with agenesis of corpus cal-
losum, holoprosencephaly, defects of Pontoneocerebellar hypoplasia
lateral ventricles and optic nerve Microcephaly
Cavum septi pellucidi Hypoplastic cerebellar hemispheres
Normal in newborn, separation of the
Dentate nuclei disorganized
leaflets of the septum pellucidum
Asymptomatic
Death by age 2 yr
Granular cell aplasia
G. Trisomies
Cerebellar heterotopiaectopic gray matter
Downs syndrome (Trisomy 21)
in cerebellar white matter, more common in
Brain more rounded (reduced frontal hemispheres
lobe) Cerebellar cortical dysplasiadisorganized
Usually decreased brain weight cerebellar cortical architecture
Narrowing of superior temporal gyrus; I. Brain stem malformations
bilateral in half of cases Olivary heterotopia
Brain stem and cerebellum small versus Olivary and dentate dysplasia
cerebrum J. Hippocampal sclerosis (mesial temporal scle-
Loss of neurons focally (cortical layer 3), rosis)
poor myelination Associated with chronic epilepsy
Alzheimers disease-like changes in mid- Neuronal cell loss and gliosis in Sommer
dle-aged patients sector region (CA1), dentate and endplate
(CA4) of hippocampus
May see pathology in brain (abscess,
embolic disease) related to congenital Etiology uncertain
heart abnormalities that can be seen in II. Perinatal Pathology
Downs syndrome A. Perinatal lesions of CNS
Patau syndrome (Trisomy 13) Birth injuriesmay be misnomer
Holoprosencephaly and arhinencephaly Uncertainty regarding time of insult
< 2/3 of cases Types
May see cyclopia, microcephaly, and cere- Anoxic/ischemic
bellar dysplasia
Trauma
Edwards syndrome (Trisomy 18)
Infection
Microcephaly, dysplasia, disorganization
of the lateral geniculate nucleus Toxic/metabolic
H. Cerebellar malformations B. Anoxia/ischemia
Total agenesis rare, often associated with Prenatal causes include maternal anemia,
hypoplastic/dysplastic pontine nuclei and heart disease, pneumonia, shock, placental
inferior olives; may see with large occipital abnormality (umbilical cord knots, infarcts,
encephaloceles infection), multiple gestations (twins, trip-
Aplasia of vermis lets, etc.)
Oversedation (improper anesthesia at
Seen in Dandy-Walker syndrome
delivery)
Joubert syndrome Postnatal causes include aspiration, unre-
Episodic hyperpnea, abnormal eye lieved airway obstruction, respiratory dis-
movements, ataxia, mental retardation tress syndrome, pneumonia
46 NEUROPATHOLOGY REVIEW

C. Ulegyria thyhypertrophic astrocytes, perivascular


Related to cortical ischemia globules in white matter, Gram-negative
endotoxin effect on myelin formation
Shrunken sclerotic gyri
H. Hydranencephaly
Loss of tissue at sulci depth
Related to ischemic change
Watershed areas
Cerebral hemispheres (carotid distribu-
Flanked by gyri with less neuronal loss and
tion)membranous sacs of leptomeninges
irregular patches of myelinated axons
and gliosis
D. Diffuse lobar sclerosis
Porencephalycontinuation with ventricular
Related to cortical ischemia system and subarachnoid space
Evenly distributed atrophy Spastic quadriplegia, failure to develop
Bilateral, symmetrical I. Germinal matrix hemorrhage
Walnut kernel brain Cause: perinatal anoxia
Secondary white matter atrophy Prematurity increased risk, <32 wk
E. Basal gray matter necrosis Rupture of thin-walled veins related to
Related to ischemia congestion
Cystic necrosis Intraventricular hemorrhage
Partial necrosis Cavitary lesions in healed cases
Irregular neuronal loss and astrocytosis May see choroid plexus hemorrhage
(full term)
Overgrowth of perivascular myelinated
nerve fibers (status marmoratus, etat mar- J. Subarachnoid hemorrhage
ble), ferruginization of residual neurons Origin
F. Multilocular cystic encephalomalacia Related to germinal matrix bleed into
Related to ischemia ventricle
Infancy De novo related to anoxia
Cystic cavities with gliotic white matter rem- Traumatic venous tear
nants
Progressive hydrocephalus
G. Periventricular leukomalacia
May see in cerebellum
Related to ischemia
K. Perinatal vascular lesions
Multifocal, sharply circumscribed foci of
Arterial
necrosis, typically adjacent to lateral ven-
tricle Thrombotic occlusions rare (middle cere-
Coagulative necrosis starting 38 hr after bral artery most common site)
insult, marked by nuclear pyknosis, edema, Emboliccongenital heart disease
eosinophilia, swollen axons (spheroids) Venous
After 1215 hr, marginal astrocytic prolifera- Circulatory failure and leptomeningitis
tion and capillary hyperplasia seen
stasis thrombosis
Microglial proliferation and macrophages
Prematuredeep internal venous system
within 1 wk
Later astrocytosis, mineralized axons, cavi- Termsuperficial cerebral veins and
tation sinuses
Associated with prematurity (greatest risk Arteriovenous malformation
2832 wk), respiratory distress syndrome, L. Subdural hemorrhage
apnea, cardiac arrest Trauma related, full term with prolonged
Petechiae, hemorrhage in white matter labor
Perinatal telencephalic leukoencephalopa- Laceration of falx or tentorium
CHAPTER 5 / CONGENITAL MALFORMATIONS, PERINATAL DISEASE AND PHACOMATOSES 47

Damage to bridging veins, straight sinus, Cortical dysplasia


vein of Galen III. Phacomatoses
Posterior fossa rare, more serious than con- A. Neurofibromatosis type I (classical type NF-I,
vexity von Recklinghausens disease)
Subdural hygromahematoma encased by Autosomal dominant, 1 : 30001 : 5000
neomembrane
50% of cases are sporadic
M. Ponto-subicular necrosis
Variable expressivity and penetrance
Neuronal necrosis in basis pontis
subiculum Gene located chromosome 17q11.2
Pathogenesis may be function of disordered Gene product: neurofibromin, a GTP-ase
flow through vertebrobasilar circulation activating protein
N. Mobius syndrome Diagnostic criteria for NF-I (need at least
two of the following)
Necrosis of selected cranial nerve nuclei
(especially VI and VII) Six or more cafe-au-lait macules >5 mm
Destruction may occur in utero or in neona- in diameter in children and >15 mm in
tal period teenagers and adults (usually present at
birth or early childhood, flat and hyperpig-
Masked facies (facial diplegia) and stra-
mented macules)
bismus
Skeletal abnormalities and mental retar- At least two neurofibromas of any type or
dation one plexiform neurofibroma
O. Bilirubin encephalopathy Axillary or inguinal freckling (intertrigi-
AKA: kernicterus, nuclear jaundice nous hyperpigmentation)
Associated with erythroblastosis fetalis, Optic nerve glioma (12% of NF-I
hemolytic anemias patients, 50% of childhood tumors are in
Two patterns patients with NF-I)

Symmetrical; pallidum, substantia nigra, Two or more hamartomas of the iris


Ammons horn, cranial nerve nuclei (III, (Lisch nodules) (mostly bilateral, appear
VIII, XII), inferior olive, dentate by age 25 yr by slit-lamp exam)

Diffuse, associated with periventricular A distinct osseous lesion (sphenoid dys-


leukomalacia plasia, thinning of long-bone cortex with
or without pseudoarthrosis).
Damage to blood-brain barrier with necrosis
Discoloration of neurons (lost over time), A first-degree relative (parent, sibling or
pyknosis, eosinophilic neuronal degenera- child) with NF-I
tion, glial scarring, atrophy Other findings may include malignant periph-
Increased risk with low albumin, respiratory eral nerve sheath tumor, meningioangio-
distress, acidosis matosis, pheochromocytoma, cranial and
renal artery stenosis, skull defects, rhabdo-
P. Alcohol (fetal alcohol syndrome)
myosarcoma, Wilms tumor and nonlympho-
Dysraphism cytic leukemia, glioma, dural ectasia, macro-
Holoprosencephaly cephaly, cortical dysplasia/hamartomas,
Agenesis of corpus callosum scoliosis/kyphosis
Porencephaly B. Neurofibromatosis type II (central type, NF-II)
Microcephaly Autosomal dominant
Hydrocephalus 1 : 50,000
Cerebellar malformations Mutation rare
Optic nerve hypoplasia Gene located chromosome 22q
Retinal ganglion cell loss Gene product: merlin (a cytoskeletal protein
48 NEUROPATHOLOGY REVIEW

and tumor suppressor)moesinezrin Shagreen patcheslumbar and intertrigi-


radixin-like protein nous areas of leathery macular patches,
Diagnostic criteria (need one of following) 20% of patients
Bilateral cranial nerve VIII masses seen Ash-leaf patcheshypopigmented
with appropriate imaging (schwannoma) patches
A first-degree relative with NF-II and Periungual fibromas550% of patients
either a unilateral cranial nerve VIII mass Angiomyolipoma
or two of the following:
Polycystic kidneys in children
Neurofibroma

Meningioma Cardiac rhabdomyomas


Glioma Lymphangioleiomyomatosis
Schwannoma Retinal hamartoma
Juvenile posterior subcapsular lenticu- D. Sturge-Weber disease
lar opacity AKA: encephalotrigeminal angiomatosis
C. Tuberous sclerosis Absence of familial predisposition
Tuber-potato-like growth, lesions often Facial port wine stain (nevus flammeus)
calcify with agesclerotic especially trigeminal region
Autosomal dominant with incomplete pene- Seizures, retardation
trance; 6 : 100,000 incidence
Leptomeningeal venous angioma
Frequently sporadic (5080%)
Cortical calcification (tram-line type of calci-
Forme-fruste cases common fication radiologically) and atrophy
Decreased life expectancy Hemihypertrophy of skullreaction to brain
Chromosome 9q34 (TSC1 gene) and 16 atrophy, elevation of sphenoid wing and
p13.3 (TSC2 gene) petrous ridges, enlarged paranasal sinuses
Classic triad: adenoma sebaceum, seizures, Congenital buphthalmus and glaucoma
mental retardation Hemiatrophy of bodycontralateral to
Tuberous sclerosis findings in central ner- facial nevus, result of hemiparesis
vous system E. von Hippel-Lindau disease
Subependymal giant-cell astrocytoma, usu- 1/31,0001/53,000 incidence
ally near foramen of Monro
20% of cases with familial incidence, males
Astrocytic hamartomassmall nodules, > females
candle guttering, lateral ventricles Autosomal dominant with incomplete pene-
Tubersfirm, ill-defined, widened gyri, trance, chromosome 3p
disorganized cortical architecture, can Cerebellar hemangioblastoma (Lindaus
calcify, sheaf-like bundles of neuroglial tumor) and retinal hemangioblastoma (von
fibers in superficial part of tuber Hippels disease)
Idiopathic ventricular dilatation Renal cell carcinoma and renal cysts
Cortical dysplasia Hepatic adenomas and cysts
Seizures in 90% of patients Pancreatic adenoma
Mental retardation in a minority of Epididymal cystadenoma
patients Pheochromocytoma
Tuberous sclerosis findings in other organ Polycythemia associated with hemangio-
systems blastoma
Adenoma sebaceum (angiofibroma) Paragangliomas
6090% of patients over 4 yr of age Aggressive papillary tumor of middle ear
CHAPTER 5 / CONGENITAL MALFORMATIONS, PERINATAL DISEASE AND PHACOMATOSES 49

F. Ataxiatelangiectasia benign tumors of the skin and other organs,


1/40,000 incidence thyroid carcinoma
Autosomal recessive I. Retinoblastoma syndrome
Prone to developing osteosarcomas,
Chromosome 11q2223
pineoblastomas, malignant gliomas
Progressive cerebellar ataxia (usually pres- Chromosome 13q14, RB1 gene
ents in infancy), conjunctival and facial tel-
angiectasias, immunoglobulin abnormalities Gene product causes cell cycle arrest
[especially IgA], hypersensitivity to radia- Both normal alleles of Rb locus must be
tion, predisposition to lymphoma, thymus inactivated (two hits) for development of
absent or rudimentary, hypoplastic gonads) tumor. In familial cases, the child is born
with one normal and one defective copy of
Abnormality of DNA processing and repair
the Rb gene. The intact gene copy is lost
Cerebellar cortical atrophy as a result of through a somatic mutation.
defective maturation of Purkinje cells and J. Li-Fraumeni syndrome
granular neurons, posterior column degenera-
Malignant gliomas of CNS
tion, nuclear pleomorphism and cytoplasmic
enlargement in Schwann cells and gangli- Also predisposed to breast carcinoma, soft
onic satellite cells tissue sarcoma, osteosarcoma, leukemia,
lymphoma, and adrenocortical carcinoma
G. Neurocutaneous melanosis (Touraine syn-
drome) p53 gene on chromosome 17p13
K. Turcot syndrome
Autosomal dominant
Medulloblastoma and malignant glioma
Melanocytic proliferation of leptomeninges,
skin, and eye Also develop polyposis coli
Chromosome 5q21, APC gene
Some cases sporadic
L. Gorlen syndrome
Rarely associated with neurofibromatosis
Medulloblastoma
H. Cowden syndrome AKA: basal cell nevus syndrome (basal cell
Dysplastic gangliocytoma of cerebellum carcinoma), keratocysts of the jaw, ovarian
(Lhermitte-Duclos) fibromas
Also develops trichilemmomas, hamartomas, Chromosome 9q31
6 Demyelinating and Dysmyelinating
Diseases

I. Demyelinating Disease Two-thirds with continuous deterioration


A. Definition Subgroups are either rapidly fatal or
benign
AKA: myelinoclastic disease
Genetic predisposition
Disease of central and peripheral nervous sys-
tem characterized by a selective loss of 1520 risk in immediate relatives
myelin with relative sparing of axons 533 risk for identical twins
To be distinguished from dysmyelinating dis-
259 risk for fraternal twins
ease (i.e., leukodystrophies), which are char-
acterized by loss of myelin with an accumula- Northern European HLA-DR15
tion of abnormal myelin breakdown products Association with M3 antitrypsin allele
and axonal involvement
Geographical distribution
B. Myelin sheath
Formed by modification of oligodendroglial High-frequency zones: northern Europe
cell membrane and United States, Southern Canada, New
Zealand, southeast Australia; infrequent in
75% lipid, 25% protein Orientals
Myelin basic protein
Multiple sclerosis factor operates during
Membrane lipidscholesterol (43%), phos- adolescence
pholipids (40%), sphingolipids (17%)
Classic clinical presentation
C. Causes/mechanisms of demyelination
Peak age 2040 yr, females > males
Oligodendroglial cell affected
Clinical course highly variable
Viral
Paresthesias, limb weakness, ataxia, blad-
Chemical
der dysfunction, nystagmus, optic neuritis,
Myelin affected rarely clinically silent
Immune Recovery from acute relapse may be the
Viral result of loss of edema, decreased astro-
cytic swelling, disappearance of locally
Chemical
produced substances that interfere with
Physical nerve conduction
D. Multiple sclerosis CSF fluid with elevated protein; increased
Relapses and remissions -globulin and oligoclonal bands

51
52 NEUROPATHOLOGY REVIEW

Gross pathology Blindness with paraplegia (demyelination


May see atrophy of optic nerve and spinal cord)

Plaques firm, deep white matter, often mul- Most with acute onset, 50% die within
tiple months of clinical onset
Diffuse distribution especially periven- E. Tumor-like demyelinating lesion
tricular region, optic nerve, spinal cord Most present with a solitary lesion, radio-

graphically suggesting tumor
Cervical cord most common spinal site
Subset of patients are older (i.e., >55 yr)
Lesions often of different histologic age
Most improve with steroid therapy, most do
Acute lesionmyelin fragmentation, relative not develop additional lesions
axonal preservation, microglial proliferation
Pathology marked by demyelination (white
Subacute plaqueLoss of myelin and oligo- matter macrophages, reactive astrocytosis,
dendrocytes, macrophages, free neutral fat, perivascular chronic inflammation)
astrocytosis, perivascular lymphocyte
inflammation (T-cells) F. Acute disseminated encephalomyelitis
Postvaccination, postinfectious (measles,
Remote/inactive lesionloss of myelin,
mumps, chicken pox, rubella, whooping
some axons and astrocytic processes remain,
cough, herpes simplex)
meningeal inflammation, iron deposits at
edge, adjacent white matter abnormal Sudden onset 514 d after viral infection or
immunization
Shadow plaquesborder between normal
and affected white matter where there are Generally good prognosis, death 10%
abnormally thin myelin sheaths, may repre- Immunological mechanism (resembles experi-
sent either partial/incomplete myelin loss or mental allergic encephalitis).
remyelination by surviving oligodendrocytes Pathology marked by perivascular demyelin-
May occasionally see plaques in the cerebral ation, perivascular inflammation (acute and
cortex chronic), microglial proliferation
Marburg type In acute phase, considerable edema (with her-
niation) and vascular congestion possible
Acute, fulminant multiple sclerosis
G. Acute hemorrhagic leukoencephalitis
Death, often between 1 and 6 mo after AKA: Weston-Hurst disease
onset of symptoms
Less than age 30 yr, males > females
Most plaques histologically acute/subacute
Incidence highest in spring and fall
(hypercellular)
Abrupt onset, pyrexia, neck stiffness, hemi-
Balo type plegia, seizure, coma, death in a few days
Concentric sclerosis Preceded by upper respiratory infection in
Acute onset, typically 2nd and 3rd decades about 50% of cases
Increased CSF protein, neutrophils
Most patients die in less than 2 yr from
presentation Etiology: ? immune complex mediated, peri-
vascular immunoglobulin and complement
Concentric demyelination
Pathology marked by swollen, soft brain,
Schilder type symmetrical white matter hemorrhages, necro-
Acute/subacute form with one or more sis of vessel wall, perivascular edema, neutro-
large plaques involving cerebral hemi- phils, perivascular demyelination, ball and
sphere and measuring >2 3 cm ring hemorrhages (thrombosed capillary in
center)
Seen predominantly in childhood
H. Central pontine myelinolysis
Devic type
Pons (central part of upper and middle pons),
Neuromyelitis optica other sites may be affected
CHAPTER 6 / DEMYELINATING AND DYSMYELINATING DISEASES 53

Loss of myelin, preservation of neurons tain dyes there is a shift in the absorbance
Oligodendrocytes lost spectrum of the dye
Reactive astrocytosis Electron microscopy
Associated with abnormalities of sodium Cytoplasmic bodies in oligodendrocytes,
(rapid increase from hyponatremic state) astrocytes, Schwann cells, 3 types
I. Marchiafava-Bignami disease Concentrically lamellar structures (tuff-

Corpus callosum and genu demyelination, stone inclusions)


may see lesions elsewhere Herringbone pattern or hexagonal

Drinkers of crude red wine, associated with arrangement


alcohol use Laminated structures or zebra-like
Degeneration of anterior commissure and bodies
middle cerebellar peduncles Also seen in skin, kidney, liver, gall-
Total loss of oligodendrocytes, macrophage bladder, and urine
and astrocyte response, vessels in involved B. Globoid cell leukodystrophy
regions may proliferate and show hyalinized AKA: Krabbes disease
walls
Autosomal recessive
Pathogenesis not known, toxic?
Deficiency of galactocerebroside-B galacto-
J. Toxic sidase
Chronic cyanide Diagnose with enzyme assay using serum,
Chronic methyl alcohol white blood cells, cultured fibroblasts
Amphotericin B Onset 35 mo
Many others Seizures, feeding problems, hyperirritability,
II. Dysmyelinating Diseases psychomotor retardation, cortical blindness
A. Metachromatic leukodystrophy Death by age 2 yr
Autosomal recessive Cerebral atrophy
Arylsulfatase A deficiency, chromosome 22q Gray discoloration of centrum ovale and
corona radiata
Sulfatide accumulation
Spares arcuate fibers
Most common form lateinfantile
Globoid cell proliferation along vessels,
Diagnosis: amniotic fluid fibroblast assay, macrophage origin
leukocytes, urine assay
Cells PAS positive, weakly sudan positive
Electromyography (EMG) of peripheral nerve
abnormalloss of myelinated axons and thin Pyramidal tracts most severely affected
myelin sheaths Reactive astrocytosis
Onset age 12 yr with flaccid weakness, psy- Loss of large myelinated axons with segmen-
chomotor retardation, quadriplegia, blindness tal demyelination
Death in 2 yr Electron microscopy: inclusionsstraight or
Adult type presents with psychosis and curved tubular profiles with longitudinal stria-
dementia, may die of venous thrombosis tions within globoid cells
Chalky discoloration of white matter, macro- C. Adrenoleukodystrophy
phages containing cerebroside sulfate Adrenomyeloneuropathy (spastic paraparesis,
Preservation of arcuate fibers distal symmetrical polyneuropathy)
Severe axonal damage and loss Neonatal
Metachromatic granular bodies in macro- Symptomatic heterozygote
phages, neurons, Schwann cells Most X-linked (Xq28)
Metachromasiawhen sulfatide binds to cer- Classical form with onset in 1st decade
54 NEUROPATHOLOGY REVIEW

Visual symptoms early, motor signs later White cortical ribbon, discolored, loosened
Steady deterioration with death in 5 yr white matter
One-third with adrenocortical failure Diffuse demyelination
Arcuate fibers not spared
Cutaneous pigmentation
Rosenthal fibers
Some juvenile forms with long survival
E. Canavans disease
Adults may present with psychiatric
symptoms AKA: van Bogart and Bertrand spongy
degeneration
Defect in long-chain fatty acid metabolism
Deficiency in aspartoacylase
(carbon chain length >22)
Subcortical arcuate fibers involved
Firm, gray colored white matter; occipital,
parietal, temporal lobes; atrophy symmetric Spongy change (spongy degeneration)
European Jews, autosomal recessive
Relative sparing of subcortical arcuate fibers,
band of Gennari, frontal lobe, brain stem, Most die in infancy, onset first 6 mo
and spinal cord Severe mental retardation, head enlargement,
Microscopically three zones hypotonia followed by spastic paralysis
Demyelination, Alzheimer II astrocytes
Destruction of myelin, axonal sparing,
occasional PAS-positive sudanophilic mac- Occipital lobe more involved
rophages F. Pelizaeus-Merzbacher disease
X-linkedmost cases (Xq 21)
Myelinated and demyelinated axons,
inflammation, and macrophages Defect in gene for phospholipid protein
Classic (type I)
Dense gliosis, loss of oligodendrocytes,
myelin, and axons Onset infantile
Dementia
Degeneration of geniculate bodies, hippo-
campal dentate nucleus Death within 5 yr
Electron microscopy: curvilinear inclusions Atrophic brain
(macrophages) Brain stem and cerebellum severely affected
Adrenals may show Tigroid demyelinationirregular islands of
myelin (perivascular) preserved (some degen-
Cortical atrophy, medulla spared erating fibers)
Relative sparing of zona glomerulosa Astrocytosis
Ballooned epithelial cells, lymphs G. Cockaynes disease
Inclusionsseen in testis (Leydig cells), Onset late infancy, autosomal recessive
Schwann cells, nodes, liver, spleen Sunken orbits, dwarfism, deafness, mental
deficiency, prominent straight nose
D. Alexanders disease
Completely shrunken white matter
Presents in infancy or childhood
Calcification
Progressive dementia, paralysis, epilepsy
Small brain, atrophic optic nerves, fibrosed
Megalencephalic meninges
Sporadic
7 Metabolic and Toxic Diseases

I. Metabolic Disorders Death age 23 yr


A. Lysosomal disorders Late infantile form (onset after 18 mo) and
rare adult forms exist
Lysosomal enzyme deficiency resulting in
accumulation of material in secondary lyso- C. GM1 gangliosidosis
somes. Infantile form (type I)failure to thrive, hep-
Includes glycogenosistype II, sphingolipi- atosplenomegaly, facial dysmorphisms, car-
doses, mucopolysaccharidoses, mucoli- diomyopathy, psychomotor retardation, sei-
pidoses, gangliosidoses zures, cherry red spots
Others Late infantile/juvenile (type II)progressive
mental retardation and motor retardation, sei-
B. Tay-Sachs disease zures, death age 310 yr, organs normal size
GM2 gangliosidosis Rare adult form (type III)
Autosomal recessive Defect in -galactosidase (chromosome 3)
Ashkenazi Jewish population Brains generally appear grossly normal or
Hexosaminidase A deficiency (lack of syn- atrophic
thesis of -subunit of enzyme)type B Balloon neuronal cytoplasm with foamy qual-
Hexosaminidase A and B deficiency ity and white matter gliosis
(Sandhoff)type O, infantile onset D. Niemann-Pick disease
Activator protein deficiency, hexosaminidase Autosomal recessive, chromosome 11p
levels generally normalType AB, infantile
Mutations common in Ashkenazi Jewish pop-
onset
ulation
Accumulation of GM2 ganglioside Sphingomyelinase deficiency (a subset of
Normal at birth, psychomotor retardation patients do not have a sphingomyelinase
starts after 56 mo deficiency)
Later hypotonia and spasticity develop, Accumulation of sphingomyelin and choles-
seizures terol
Brain weight normal or decreased/increased Five phenotypes (AE)
Balloon neurons with cytoplasmic vacuoles Type A7580% of cases (severe infan-
Visceral organs generally do not show much tile neurovisceral form)
evidence of ganglionic accumulation Type Bvisceral disease only, normal
Electron microscopy: whorled membranes or CNS function
zebra bodies Electron microscopy: membranous cyto-
Cherry red spot in macula, blindness plasmic bodies, zebra bodies

55
56 NEUROPATHOLOGY REVIEW

Brain weight decreased, balloon neurons and age 30 yr with myoclonic epilepsy,
glial cells dementia, ataxia
Multiorgan involvement with type A, includ- Finnish typelate infantile onset (47
ing hepatosplenomegaly, cherry red spots of yr); chromosome 13q
macula, lung infiltrates Accumulation of autofluorescent lipopig-
May see ballooned ganglion cells in the gas- ments in neurons
trointestinal tract (Auerbach and Meissner Brain usually atrophic, skull thickened
plexuses)
Neuronal loss, gliosis, loss of myelin includ-
Nieman-Pick cellfoamy phagocyte storage ing subcortical fibers
cell seen in extra-CNS organs (sea blue his-
tiocytes) Electron microscopy: granular, curvilinear,
fingerprint patterns
Death age 12 yr
Biopsy: skin, rectum, or brain to diagnose;
E. Gauchers disease can evaluate lymphocytes in some cases
Autosomal recessive G. Mucopolysaccharidoses
Glucocerebrosidase deficiency, chromosome Classification
1q
Type I Hurler-Scheie; -l-iduronidase
Accumulation of glucocerebroside in reticulo-
deficiency; chromosome 4p
endothelial cells and neurons
Three clinical types (I: adult, noncerebral; II: Type II Hunter; iduronate sulfatase defi-
infantile, acute cerebral pattern; III: interme- ciency; chromosome Xq
diate) Type III Sanfilippo; heparan N-sulfatase,
Brain gross appearance generally normal -N-acetylglucosamindase, acetyl CoA :
Hepatosplenomegaly and lymphadenopathy -glucosaminide acetyl transferase defi-
ciency; subset with chromosome 12q
Gaucher cells (RE cells) rarely vacuolated, abnormalities
fibrillary cytoplasm, crumpled tissue
paper, cells often perivascular in distri- Type IV Morquio; -galactosidase defi-
bution ciency; chromosome 16q in subset
Electron microscopy: distended, elongated Type VI Maroteaux-Lamy; N-acetylgalac-
lysosomes with stacks of lipid bilayers tosaminase-4-sulfatase deficiency; chromo-
Neurons shriveled and destroyed, especially some 5q
cortical layers III and V, Purkinje cells, den- Type VII Sly; -glucuronidase deficiency;
tate nucleus chromosome 7q
F. Ceroid-lipofuscinoses (Battens disease) Accumulation of dermatan sulfate or hep-
Most autosomal recessive aran sulfates
Five main clinical types Storage of mucopolysaccharides in lyso-
somes in multiple organ systems
Infantile typepsychomotor regression
starting at about 8 mo, visual loss, hypoto- Brains usually grossly normal
nia, ataxia, microcephaly, death age 310 Perivascular collections of foamy cells, peri-
yr, chromosome 1p vascular pitting in white matter
Late infantile typeseizures start Electron microscopy: zebra bodies, mem-
between 18 mo and 4 yr, death between 4 brane-bound lamellar stacks
and 10 yr H. Sialidosis
Juvenile typeretinopathy presentation Neuraminidase deficiency; chromosome 10p
between ages 4 and 9 yr, seizures, gait Cherry red spot of the macula, myoclonus,
disturbance, hallucinations, death late coarse facial features, skeletal dysplasia,
teens; chromosome 16p deafness, mental retardation
Adult type (Kufs disease)onset around Variable degrees of ballooned neurons, vacu-
CHAPTER 7 / METABOLIC AND TOXIC DISEASES 57

olation of neuronal cytoplasm, and lipo- M. Cystinosis


fuscin accumulation Associated with renal disease, Fanconis syn-
May see extra-CNS pathologyGI tract, kid- drome, photophobia, hypothyroidism
ney, liver Widespread deposition of cystine including
A group of disorders (galactosialidoses) brain (especially choroid plexus)
marked by a combined neuraminidase/- Crystals are soluble, should process tissue
galactosidase deficiency fresh/frozen or in alcohol to see crystals on
I. Mucolipidoses sectioning
Defect in N-acetylglucosamine-1-phospho- N. Lafora body disease
transferase; chromosome 4q Abnormality of carbohydrate metabolism
Skull/skeletal deformities, short stature, psy- resulting in an accumulation of polyglucosan
chomotor retardation, joint contractures, hep- (glucose polymers)
atosplenomegaly (type IV) Autosomal recessive; chromosome 6q
Vacuolated lymphocytes in peripheral blood Seizures with myoclonus
Brain usually normal grossly; may see some Microscopically, cytoplasmic neuronal inclu-
neuronal loss and gliosis in type IV sion (Lafora body) with basophilic core and
concentric target-like appearance and pale
J. Mannosidoses
outer zone (PAS positive)
Defects in lysosomal - or -mannosidase
O. Mitochondrial encephalomyopathies
Defect in -mannosidase in autosomal reces- Leighs disease
sive condition, chromosome 19
AKA: subacute necrotizing encephalomye-
Psychomotor retardation, coarse facies, gingi-
lopathy
val hyperplasia, deafness, hepatosplenomeg-
aly, corneal opacities Autosomal recessive disorder
Vacuolation of nerve cells in brain, PAS pos- Lactic acidemia, arrest of psychomotor
itive development, feeding problems, seizures,
K. Fucosidosis extraocular palsies, weakness
Defect in -fucosidase; chromosome 1 Associated with a point mutation at posi-
tion 8993 of mitochondrial DNA
Brain may be large or small
Most die within 12 yr
Neuronal ballooning and some neuronal
loss/gliosis Result of biochemical defects in mitochon-
drial pathway
May see many Rosenthal fibers
L. Fabrys disease Multifocal, asymmetric destruction of
brain tissue
Defect in -galactosidase A; chromosome
Xq Vessel proliferation in periventricular gray
matter of midbrain, tegmentum of pons,
Skin lesionstelangiectasias (angiokeratoma
thalamus, and hypothalamus
corporis diffusum)
MERRF
Renal insufficiency
Myoclonic epilepsy with ragged red fibers
Retinal abnormalities, autonomic dysfunc-
tion, hypertrophic cardiomyopathy Maternally transmitted mitochondrial
defect (DNA mutation)
PAS-positive deposits in certain neurons
(amygdala, hypothalamus, brain stem) MELAS
Vascular changes in endothelial cells in the Mitochondrial encephalopathy, lactic aci-
brain, infarcts relatively common dosis, and stroke-like episodes
Electron microscopy: lipid lamellae inclu- Mitochondrial abnormalities in cerebral
sions vessels and basal ganglia calcification
58 NEUROPATHOLOGY REVIEW

P. Phenylketonuria Swelling of myelin with vacuolations of tho-


Impaired cognitive development racic cord with degeneration of posterior col-
umns and descending pyramidal tracts
Can identify patients with neonatal urine
screening U. Hepatic encephalopathy
Microcephaly if untreated Alzheimer II astrocytes in cortex and basal
ganglia
White matter spongiosis and gliosis
Alzheimer II astrocytesenlarged, pale cen-
Q. Maple syrup urine disease tral chromatin, prominent nuclear membrane
Vomiting, lethargy, convulsions, urine and nucleus
smells like maple syrup In Wilsons disease (autosomal recessive
Caused by a variety of defects in -ketoacid condition related to copper accumulation),
dehydrogenase neuronal degeneration in the striatum with
In untreated cases, spongiosis and gliosis of Alzheimer II astrocytes
white matter develop II. Toxic disorders
R. Menkes disease A. Carbon monoxide
X-linked recessive disease of copper metab- Selective injury of neurons of layers III and
olism V of cortex, Sommer sector and Purkinje
Mental retardation, hypotonia, hypothermia, cells
visual disturbance, seizures, failure to thrive Bilateral globus pallidus necrosis
Atrophic brain (especially cerebellum), B. Methanol
cerebral arteries thin walled and tortuous Edema and petechial hemorrhages
(because of altered elastic fibers) Degeneration of retinal ganglion cells caus-
Neuronal loss and gliosis, hypomyelination, ing blindness
abnormal Purkinje cell dendritic arborization Bilateral putamenal necrosis and focal white
S. Thiamine deficiency matter necrosis
Beriberiassociated with a symmetrical C. Ethanol
peripheral neuropathy (axonal degeneration) Anterior vermal atrophy associated with
Wernicke encephalopathyabrupt onset of ataxia and nystagmus
psychotic symptoms, if untreated may At risk for developing expressive Wernicke
develop into Korsakoff syndrome marked by encephalopathy, coagulopathies related to
memory disturbances and confabulation liver disease, head trauma, fetal alcohol syn-
May be caused by any disorder that results drome, myelopathy, peripheral neuropathy,
in thiamine deficiency (e.g., alcoholism, gas- central pontine myelinolysis, Marchiafava-
tric abnormalites) Bignami disease
D. Metallic intoxications
Wernicke encephalopathy marked pathologi-
cally by hemorrhage and necrosis of mamil- Aluminumabnormal cytoskeletal derange-
lary bodies, periventricular regions, hypothal- ments, neurofibrillary tangles (structurally
amic nuclei and thalamus; eventually see different than those of Alzheimers disease)
gliosis with hemosiderin deposition and mac- Arsenicused in weed killers (pesticides),
rophages peripheral neuropathy with segmental demye-
lination, microvascular hemorrhagic encepha-
T. Vitamin B12 deficiency
lopathy in pons and midbrain
Subacute combined degeneration of the spi- Leadcerebral edema and vascular conges-
nal cord tion, petechial white matter hemorrhages
Ataxia, numbness and tingling of extremi- Manganesebasal ganglia degeneration
ties, spastic weakness/paraplegia of extrem- (especially medial globus pallidus and sub-
ities stantia nigra)
8 Degenerative Disease

I. Brain Changes in Normal Aging B. Proposed causative mechanisms


Loss of mass (1020%) with neuronal loss Geneticchromosome 4 (presenilin-1), 21
Ventricular dilatation (amyloid), 19 (apolipoprotein E), 1 (preseni-
lin-2)
Thickened leptomeninges
Vascular (amyloidosis)
Vascular changes
Neurofibrillary tangles, neuritic plaques, Toxic (aluminum)
Lewy bodies, granulovacuolar degeneration, Traumatic (head traumarisk factor)
Hirano bodies in small numbers Inflammatory (acute-phase reactants)
Corpora amylacea Metabolic (mitochondrial abnormality)
Lipofuscin accumulation Infection (atypical agent)
Increase in astrocyte number and size Degenerative (age as risk factor)
Decrease in neurotransmitters
C. Gross pathology
II. Normal Pressure Hydrocephalus
Normal size or atrophic brain
Normal CSF pressure
Atrophy fronto-temporal predisposition (nar-
Elderly rowing of gyri and widening of sulci)
Triad: dementia, gait disturbance, urinary
Ventricular dilatation
incontinence
D. Neuronal loss
Ventricular dilatation, no cortical atrophy
? impaired CSF absorption Acetylcholine deficiency
At autopsy, ventricular size usually Nucleus basalis of Meynertneurons lost,
decreased (terminal edema, formalin) tangles
III. Alzheimers Disease Locus ceruleus, nucleus raphe dorsalis,
A. General information pontine tegmentum, hippocampus and
amygdaloid nucleus with variable neuronal
Familial/hereditary cases (510%)begin loss
early, progress rapidly
E. Neurofibrillary tangles
Atrophy (temporal, frontal) variable finding
Non-membrane-bound masses of filaments
Impaired higher intellectual function, affect
disturbances Silver positive
Death usually from infection, inanition, dehy- Neuronal cytoplasm
dration Phosphorylated tau protein (i.e., A68 [tau is
Identical pathologic changes in trisomy 21 a microtubule protein that promotes tubule
patients stability and assembly])

59
60 NEUROPATHOLOGY REVIEW

1020-nm filaments in paired helix configu- Amorphous/acellular eosinophilic appear-


ration, crossover every 80 nm ance by light microscopy
Tangles may be seen in a whole variety of -Amyloid encoded on chromosome 21
other pathologies, including normal aging, J. There are several published pathologic criteria
subacute sclerosing panencephalitis, Nie- for the diagnosis of Alzheimers disease
mann Pick disease, Downs syndrome, myo- Khachaturian criteria based on age-related
tonic dystrophy, dementia pugilistica, pro- minimum number of senile plaques of neo-
gressive supranuclear palsy, Kufs disease, cortex per 1 mm2 at 200 magnification
and Cockaynes syndrome
CERAD (Consortium to Establish a Registry
F. Neuritic plaques for Alzheimers Disease) criteriasemi-
Spherical structures, 5200 m in diameter quantitative assessment of plaque frequency
Swollen neurites (dystrophic), contain degen- in three neocortical regions (superior tempo-
erating clusters of mitochondria, lysosomes, ral gyrus, prefrontal cortex, inferior parietal
and filaments lobule) correlated with age and clinical his-
tory of dementia
Difficult to see with routine hematoxylin
and eosin staining Braak and Braak stagingexpansion of the
CERAD to also evaluate entorhinal cortex
The neurites highlighted by silver staining
and hippocampus, also looks at neurofibril-
-amyloid core (Congo red, thioflavin S, lary tangles
crystal violet positive), -amyloid encoded K. Mixed dementia
on chromosome 21
One may encounter Alzheimers disease
Microglial cells and astrocytosis with amy- changes along with other dementia causing
loid nearby pathologies (e.g., diffuse Lewy body dis-
Three types ease, Parkinsons disease, infarcts)
Classic/typical Anywhere between 7% and 25% of Alzhei-
mers disease patients will have evidence of
Prematuresmall cerebrovascular disease
Burnt outstellate amyloid with few neu- Multi-infarct dementia: >100 mL of infarct,
rites clinically need to be able to temporally
G. Granulovacuolar degeneration relate onset of dementia with stroke
Basophilic granules with halo (35 m) IV. Other Degenerative Disorders
Pyramidal neurons of hippocampus A. Picks disease
May represent degenerating autophagocytic AKA: lobar atrophy
process involving cytoskeletal proteins Peak age of onset 4565 yr, uncommon >75
H. Hirano bodies yr
Less than 2% of dementia cases
Rod-like or oval eosinophilic cytoplasmic
inclusions (1030 m in length) within Subset of patients (minority) have autosomal
neurons dominant inheritance pattern
May represent abnormal microfilament con- Frontal and temporal lobes preferentially
figurations, by electron microscopy they con- atrophic
sist of parallel filaments and sheet-like May see atrophy in basal gray nuclei, sub-
material stantia nigra, outer three layers of cortex
I. Amyloid angiopathy Neuronal loss, astrocytosis, spongiosis
Presenilin mutations can cause increased Knife-blade atrophy or walnut brain
-amyloid production Posterior two-thirds of superior temporal
-Amyloid deposits in cerebral vessel wall gyrusspared
(especially in small arteries/arterioles in the Characteristic pathology marked by balloon
leptomeninges and cortex) cell neurons (Pick cells, globose cells) and
CHAPTER 8 / DEGENERATIVE DISEASE 61

circumscribed argentophilic cytoplasmic nesia, disturbance of posture and equilib-


inclusions called Pick bodies (silver posi- rium, autonomic dysfunction, dementia
tive) within neurons consisting of collections (30%); juvenile cases frequently present
of neurofilaments and microtubules. Both with dystonia
Pick cells and Pick bodies stain with anti- Age 4565 yr
bodies to ubiquitin and tau
MPTP-A (contaminant in the illicit synthesis
B. Huntingtons chorea of meperidine analogs); dopamine antago-
Autosomal dominant, gene on short arm of nists cause similar clinical picture
chromosome 4p16.3 (codes for huntingtin); Most cases idiopathic, some cases with
paternal transmissionsymptomatic earlier autosomal dominant transmission (linked
Trinucleotide (CAG) repeat disorder with -synuclein gene)
Choreiform movement, dementia Mitochondrial dysfunction and free-radical
Death in 15 yr injury may play a role in pathogenesis
Dopamine activity > acetylcholine activity Pathology
Decrease in choline acetyl-transferase, glu- Substantia nigra (zona compacta) and
tamic acid decarboxylase, and -aminobu- locus ceruleus marked by loss of pig-
tyric acid mented neurons
Pathology Phagocytic cells with pigment
Atrophy with loss of small ganglion cells Astrocytosis
in caudate (tail > body), putamen (ante- Lewy bodyeosinophilic, cytoplasmic
rior > posterior), larger neurons spared inclusion (intermediate filaments), 830
Astrocytosis m in diameter, may have a clear halo
Dilated frontal horns of lateral ventricles, around it, present in 20% of normal peo-
ple by age 80 yr, may be seen in Alzhei-
frontal lobe atrophy
mers disease, stain with ubiquitin and
Nucleus basalis of Meynert spared -synuclein
Vonsattel grading schema Diffuse Lewy body disease
Grade 0grossly normal, 3040% neu- May be on a continuum with Parkinsons
ronal loss in striatum without gliosis disease
Grade 1grossly normal, 50% neu- Numerous widespread neocortical Lewy
ronal loss in striatum, gliosis in medial bodies (especially superior temporal lobe,
caudate and dorsal putamen cingulate gyrus, insular cortex layers V
Grade 2neuronal loss evident in cau- and VI)
date and dorsal putamen with gliosis, Often the neocortical Lewy bodies have
gross atrophy of caudate head and atypical morphologiesubiquitin stain
putamen maybe useful in highlighting their
Grade 3gross atrophy of caudate and presence
putamen and mild atrophy of globus
Many also have features of Alzheimers
pallidus, neuronal loss as in grade 2,
disease
mild astrocytosis of globus pallidus
Grade 4severe gross atrophy of cau-
More acute onset of symptoms/dementia,
date, putamen, and globus pallidus, younger age of onset
microscopic findings worse than grade Postencephalitic Parkinsons disease
3, astrocytosis of nucleus accumbens Months or years after influenza pandemic
C. Parkinsons disease of 19141918
AKA: paralysis agitans Loss of pigmented neurons, astrocytosis
Loss of dopaminergic neurons Neurofibrillary tangles, binucleated
Clinical: pill rolling tremor, rigidity, bradyki- neurons
62 NEUROPATHOLOGY REVIEW

Few, if any, Lewy bodies Neuronal loss, axonal swelling


D. Parkinsonismdementia complex of Guam Demyelination
Chamorro natives of Guam and Marina Astrocytosis
Islands
G. Striatonigral degeneration
Rigidity, bradykinesia, dementia
Clinically similar to Parkinsons disease
Onset age 5060 yr
Resistant to levodopa
Death in 5 yr
Atrophy of caudate, putamen, zona com-
Generalized cerebral atrophy pacta of substantia nigra
Tangles, neuron loss; no plaques, Lewy bod- Neuron loss, astrocytosis
ies, or inflammatory reaction
No Lewy bodies, no tangles
Hippocampus, amygdaloid nucleus, temporal
and frontal cortex, basal ganglia, substantia Cytoplasmic inclusions (tubular structures)
nigra in oligodendroglia and some neurons,
nuclear inclusions in neurons and glial cells,
Anterior horn cell disease in some patients neuropil threadsthese findings also noted
(amyotrophic lateral sclerosis-like picture) in Shy-Drager syndrome and olivopontocere-
longer clinical course bellar atrophy
E. Progressive supranuclear palsy Part of multisystem atrophy syndrome
AKA: Steele-Richardson-Olszewski syn-
H. Shy-Drager syndrome
drome
Autonomic system failure with parkinsonism
Ophthalmoplegia (loss of vertical gaze),
ataxia, expressionless facies, rigidity (hyper- Loss of intermediolateral column neurons
extension of headposterior cervical mus- Pathology variablebetween Parkinsons
cle), mental disorders/dementia disease and striatonigral degeneration
Onset 5th7th decades Part of multisystem atrophy syndrome
Pathology I. Olivopontocerebellar atrophy
Neuronal loss with tau positive tangles Ataxia, eye and somatic movement abnor-
(straight filaments with a diameter of 15 malities, dysarthria, rigidity
nm), gliosis, and neuropil threads Most autosomal dominant
Basal ganglia and brain stem (substantia Gross atrophy of basis pontis
nigra, subthalamic nucleus, superior colli- Depletion of Purkinje cells, loss of neurons
culi, periaqueductal gray, pontine tegmen- in inferior olives
tum, dentate) the most severely involved
areas, may see atrophy in these regions Variable degeneration of spinal cord long
tracts and neuronal loss/gliosis in putamen
Astrocytes may also stain with tau protein and substantia nigra
F. Hallervorden-Spatz disease Part of multisystem atrophy syndrome
Autosomal recessive J. Corticobasal degeneration
Early childhood onset 6th7th decade
Dystonic posturing, choreoathetotic or tremu- Apraxia, limb dystonia, akinesia, rigidity,
lous movements, muscular rigidity and spas- impaired ocular movement, dementia
ticity, dementia
Frontoparietal atrophy
Death in 1012 yr
Ballooned neurons
Pathology
Degeneration of subcortical nuclei, espe-
Rust brown discoloration of globus pal- cially substantia nigra
lidus and substantia nigra
Neuronal inclusions (neurofibrillary tangle-
Mineralization (iron) of vessel walls like)tau positive; ubiquitin positive
CHAPTER 8 / DEGENERATIVE DISEASE 63

K. Friedreichs ataxia Asymmetrical symmetrical extremity


Familial form, onset 1st two decades involvement
Autosomal recessive Muscle wasting, fasciculation, sensation
intact
Chromosome 9q13 (frataxin gene)
CSF normal
GAA trinucleotide repeat expansion
Ataxia, sensory disturbances (lower part of 23 yr to death
bodyloss of joint position sense, vibration Cause of deathrespiratory failure and
sense, two-point discrimination) pneumonia
Other neurologic disturbancesdysarthria, Atrophy of anterior spinal nerve roots and
optic atrophy, deafness, dementia, epilepsy affected muscles
Skeletal deformitiespes cavus, kyphosco- Precentral gyrus atrophic
liosis Loss of anterior horn cells (lumbar, cervical
Heart abnormalenlarged, pericardial adhe- cord)
sions, myocarditis Several types of inclusions may be seen in
Peak age of onset 10 yr motor neurons
Duration of disease 550 yr Bunina bodiessmall eosinophilic inclu-
Death most commonly in 3rd and 4th sions (25 m)
decades (heart disease) Hyaline inclusions
Degeneration of sensory fibers, posterior Basophilic large inclusions
roots, and ganglion cells in lower segment
of cord Aggregates of thread-like structures called
skeins (seen on ubiquitin staining)
Posterior column degeneration, gracilis >
cuneatus Neuronal loss of hypoglossal nuclei and
other cranial nerve nuclei (facial, trigeminal)
Degeneration of dorsal nuclei (Clarkes col-
umn) and posterior > anterior spinocerebel- Pyramidal tract degeneration
lar tract Neuronophagia and gliosis
Cranial nerve tracts variably involved N. Spinal muscular atrophies
Cerebellar afferents (accessory cuneate Variety of types
nucleus) and outflow from dentate nucleus Werdnig-Hoffmannprototypical
also involved
Kugelberg-Welander
L. Dentorubropallidoluysial atrophy
CAG trinucleotide repeat expansion Adult onset forms
Chromosome 12p Later onsetmore favorable prognosis
Ataxia, chorea, myoclonic epilepsy, Werdnig-Hoffman disease
dementia Autosomal recessive
Severe neuronal loss and gliosis in dentate Chromosome 5q1113
nucleus, globus pallidus, red nucleus, and
subthalamic nucleus 1 : 20,000
Atrophy of superior cerebellar peduncles Death before 2 yr
M. Amyotrophic lateral sclerosis Proximal > distal onset of weakness
Male : Female 1.7 : 1 Hypotonia, floppy baby
Middle-aged and elderly Decreased fetal movement3rd trimester
5-10% familial autosomal dominant, genetic Facial muscles spared
locus for some of this group on SOD1 gene
on chromosome 21q Fasciculations of tongue
Weakness and painin extremities or bul- Loss of anterior horn cell neurons of spi-
bar musculature nal cord
64 NEUROPATHOLOGY REVIEW

Atrophy of anterior spinal roots Loss of neurons in nuclei ambigui, hypo-


glossal and facial nuclei
Neurogenic muscle diseaserounded
fiber atrophy Onufs nucleus spared
9 CNS Infection

I. General Information Associated with prior surgery or trauma


A. CNS protection Most common organisms are Streptococcus
followed by Staphylococcus
CSF lacks host defense
Intracranial
Meninges
Blood-brain barrier Most often related to extension from a
frontal or mastoid sinus infection or focus
B. Routes of infection
of osteomyelitis
Direct implantation
Circumscribed, flattened
Shunt
Adhesion of dura to bone confines
Local spread
abscess
Hematogenous
May cause subdural abscess, leptomen-
Septic emboli
ingitis
C. Factors in infection
Spinal
Number of organisms
Thoracic and lumbar regions
Host immune system
Type of organism More common than intracranial epidural
abscesses
D. Pathophysiology of infection
Compression of cord and roots
Injury to blood-brain barrier
Separation of tight junctions Staphylococcus aureus
Increased albumin in CSF, increased ICP B. Subdural abscess
Edema (vasogenic and inflammatory medi- Secondary to bone/sinus infection (espe-
ators) cially paranasal sinuses, middle ear, cra-
nium) via thrombophlebitis
Spinal nerve root and nerve (neck stiffness)
Dorsolateral aspect of cerebrum, frontal
Vasculitis; infarction
polar region, interhemispheric fissure
Direct cytopathic/toxic effect
Adhesions, loculation
II. Suppurative Lesions
Compression of brain, increased ICP
A. Epidural abscess
Streptococcus (#1) and Staphylococcus (#2)
Extension from bones (trauma, osteomyeli-
tis, otitis) Spinal subdural rare
Less often extension from soft tissue C. Leptomeningitis
(venous) Neutrophils, fibrin in the leptomeninges

65
66 NEUROPATHOLOGY REVIEW

With treatment, the inflammatory infiltrate Early: centralpus; middlegranulation tis-


may switch to more lymphocytes, macro- sue; outerastrocytosis/edema
phages, and plasma cells Later: centralliquefaction; middlecollag-
Arterial subintimal infiltration enous wall; outerastrocytosis
Thrombosis, vasculitis, infarct, hemorrhage, Granulation tissuecontrast enhances on
cerebral edema, vascular congestion CT scan as a result of hypervascularity
Virchow-Robin space extension Hematogenous (metastatic) abscessesoften
May extend to cause ependymitis, ventri- multiple
culitis Damage a result of local tissue destruction,
Subarachnoid fibrosis, CSF obstruction, sube- edema (mass effect)
pendymal gliosis with healing Consequences
Neonatal (maternal genital flora)commu- Organization
nity acquired Formation of daughter abscess
Strep. agalactiae (group B)exudate Ependymitis
over vertex
Leptomeningitis
E. colihemorrhage, necrotizing arteritis,
E. Septic embolism
necrosis, roughly same incidence in this
age group as Strep. agalactiae Occlusioninfarction
Proteus Infectionarteritis, septic aneurysm, hemor-
rhage, septic infarcts
Listeriamicroabscesses
Small parenchymal arteriesmicroabscess,
Childhoodcommunity acquired petechiae
Neisseria meningitidisbasal, young Septicemiacapillaries, microabscess
adults, army recruits, most epidemic out- Leptomeningesseptic infarct, hemorrhage,
breaks related to group A, associated with meningitis
Waterhouse-Friderichsen syndrome [dis-
Large arteryinfarct
soninated intravascular coagulopathy
(DIC) picture with hemorrhagic adrenal Trunk artery base of brainsubarachnoid
glands] bleed
H. influenzavertex, cortical softening, F. Tuberculosis (TB)
venous thrombosis Epidural
Strep. pneumoniaevertex Extension from vertebral infection (Potts
disease)
Adults/elderlycommunity acquired
Compression of cord
Strep. pneumoniae
Rarely causes meningitis
Listeria
Subdural
Any organism is possible with hospital-
acquired infections Miliary or tuberculoma-en-plaque
D. Abscess Tuberculoma = abscess
Localized suppuration within brain sub- Granulomas with caseous necrosis
stance Usually small and multiple
Streptococcus (#1), Gram-negative bacilli May act as mass
(#2) Staphylococcus (#3), may see anaer- Enhancing on CT scan
obes and aerobes TB meningitisbase of brain; neutrophils,
Frequently multiple organisms fibrin, endarteritis, hemorrhage, caseous
Focal septic encephalitis (cerebritis), periph- necrosis, granulomatous inflammation.
eral edema CSF studies: lymphocyte predominance with
CHAPTER 9 / CNS INFECTION 67

pleocytosis, elevated protein, decreased ening of the intima and thinning of the
glucose media), which may lead to infarct
Culture still remains gold standard for diag- Pachymeningitis cervicalis hypertrophi-
nosis, molecular biologic techniques exist to canscervical fibrosis of dura to pia
identify the organisms General paresis
Use of special stains (such as Ziehl-Neelsen,
Brain invasionsubacute encephalitis
fite) on tissue sections has a relatively low
yield (<50%) Prolonged secondary stagefailure to pro-
Other nontuberculous mycobacteria may duce neutralizing/destructive antibody or
infect CNS failure of antibody to reach spirochete
Mycobacterium avium intracellulare typi- Earlybrain grossly normal
cally marked by perivascular macro- Laterfrontal atrophy
phages with organism; rarely presents as
Loss of neurons, astrocytosis, capillary
a tumor-like nodule with fibrosis
proliferation, perivascular inflammation,
G. Sarcoidosis microglia with iron
Noncaseating granulomas
Spirochetes may be hard to identify, 3-yr
Base of brain, posterior fossa course
Near vessels in meninges Lissauers dementia
Optic nerve and chiasm involvement Variant of general paresis
common
Epileptic or apoplectiform attacks
Course usually slow, may be rapidly fatal
Hydrocephalus, cranial nerve defects, hypo- Focal signs
thalamic symptoms Average 7 to 8-yr course
May have no apparent systemic disease Temporal atrophy with pseudolaminar
H. Whipples disesase degeneration
Caused by Tropheryma whippelii, a Gram- Tabes dorsalis
positive actinomycete Degeneration of spinal dorsal roots and
PAS-positive organisms in macrophages that columns
are often arranged in perivascular distri-
Lumbosacral and lower thoracic cord
bution
I. Syphilis Dorsal root ganglia unaffected
Caused by Treponema pallidum May see meningovascular disease
Highlighted with special stains (e.g., Steiner, Optic atrophy
Warthin-Starry, Dieterle) Chronic pial inflammation, degeneration
Subacute secondary lymphocytic meningitis of nerve
Meningovascular Peripheral > central
Lymphs, plasma cells gummata (central Associated with tabes dorsalis
necrosis surrounded by epithelioid histio-
Congenital
cytes, fibroblasts, and multinucleated
giant cells) Meningovascular with obstruction, hydro-
cephalus
Do not usually see organisms with
gumma because it represents a hyperim- General paresisespecially cerebellum
mune form of tissue necrosis Rarely tabes dorsalis
Perivascular parenchymal inflammation J. Lyme disease
Arteritis, intimal proliferation, Heubners Caused by Borrelia burgdorferi (a spiro-
endarteritis obliterans (collagenous thick- chete transmitted by ioxodid ticks)
68 NEUROPATHOLOGY REVIEW

Associated skin lesions (erythema chron- or rarely direct extension from an orbital or
icum migrans) paranasal sinus infection
Lymphoplasmacytic leptomeningitis Pathology
III. Fungal Infection Granulomaabscess basal meningitis
A. General Septic infarcts secondary to vascular infil-
Immunocompromised persons at risk tration, hemorrhage
Secondary spread common Focal cerebritis, especially white matter
Hematogenous spread > contiguous E. Mucormycosis
spread Broad branching, nonseptate hyphae (1015
Granulomatous inflammation and abscess m)
are common patterns of injury Genera of Mucoraceae family, including Rhi-
Septic emboli, may result in mycotic zopus, Absidia, Mucor, Rhizomucor, Apo-
aneurysm physomyces, and Chlamydoabsidia
B. Cryptococcosis Entrynasal sinus, extension into orbit, aspi-
Budding yeast, thick mucopolysaccharide ration into lung
wall Pathology
Pigeon excrement Acute meningeal inflammation
Entrypulmonary > skin Hemorrhagic coagulative necrosis of
Pathology brain (ventral)
Leptomeningitis, gelatinous bubbly Vascular luminal and mural invasion
exudate F. Coccidioidomycosis
Macrophages Semiarid areas (South America and south-
Perivascular extension, may be quite west United States)
extensive and give the brain a honeycomb Double-contoured, refractile capsules with
look grossly endospores
Granulomas, torulomas (abscesses) Self-limited respiratory infection
Organisms highlighted with PAS, muci- 0.05% cases disseminate, CNS disease in
carmine stains 1025% of these cases
C. Candidiasis Pathology
Budding cells, branching pseudohyphae Basilar meningitisneutrophils, fibrinous
Entryskin, GI tract exudate
CNS disease as a result of systemic infec- Latergranulomas, fibrosis
tion or endocarditis Cranial osteomyelitis
Pathology Parenchymal granulomas rare
Chronic granulomas G. Blastomycosis
Microabscesses Males > females, agricultural workers
Rarely arteritis, basal leptomeningitis Acquired from contaminated soil, southeast-
ern United States
Organisms highlighted on Gomori
methenamine silver (GMS) or PAS stains Broad-based budding
D. Aspergillosis Protoplasmic body (1015 m in diameter)
Dichotomously branching septate hyphae No hyphae in tissue sections
Entryairborne lung infection followed by Lung and skin entry portals
hematogenous spread, may also get dissemi- Leptomeningitis and granulomas rare, often
nated from other organ system involvement presents as abscess
CHAPTER 9 / CNS INFECTION 69

H. Histoplasmosis Degenerationchromatolysis, eosino-


Entryinhalation of dust containing spores philia, nuclear changes, neuronophagia
Histoplasma capsulatum consists of small Eosinophilic nuclear inclusions (Cowdry)
ovoid and budding bodies, PAS and GMS Type Aintranuclear, large spherical,
positive halo (e.g., CMV, herpes, PML)
Only rarely disseminates to involve the CNS Type Bsmall, multiple, no nucleolar

Necrosis with macrophages (containing displacement (e.g., polio)


organisms), lymphocytes, and plasma cells Intracytoplasmic

I. Actinomycosis Reactive endothelial changes


Bacteria with fungal-like morphology Tissue necrosis or demyelination with cer-
Thin (<1 m pseudohyphae) filaments tain viruses
Gram-positive, acid-fast negative Inflammatory cell exudate
Sulfur granules Microgliadiffuse or focal (neurono-

Entrycervicofacial phagia, glial nodule, glial shrub, glial


scar)
Pathology
Chronic inflammatory cells (lympho-
Granulomaabscess, multiple cytes, plasma cells and macrophages),
Basilar meningitis often perivascular distribution
J. Nocardiosis B. Polio
Bacteria with fungal-like morphology Acute febrile disease, asymmetric paralysis
(legs > arms > trunk)
Thin filaments
5% mortality
Gram-positive, partial acid-fast positive
Pathology
No sulfur granules
Large motor neurons lost (anterior horn
Entrylung cells, motor neurons of cranial nerves)
Pathology Lymphocytic perivascular and meningeal
Granulomas, small and mulitple inflammation
IV. Viral Infection Gliosis and microgliosis with recovery
A. General C. Arbovirus
Routes of spread Arthropod (RNA viruses)mosquito borne
Hematogenous (polio, arbovirus) Serologic exam to make diagnosis
In United StatesSt Louis, California virus,
Peripheral nerve (rabies)
Eastern and Western Equine encephalitis
Olfactory mucosa, trigeminal nerve Pathology
(herpes)
Brain congestion and edema
Morphologic features
Widespread, gray matter
Little (edema, congestion) or no distinc-
tive gross alterations (except herpes, pro- Most severe changes with Eastern equine
gressive multifocal leukoencephalopathy Neuronal degeneration, microglial nod-
[PML]) ules, perivascular and meningeal chronic
Diffuse distribution (except polio, herpes inflammation; abscess and neutrophils in
severe cases
zoster)
D. Aseptic Meningitis
Primary neuronal involvement common
Refers to any culture-negative meningitis
Common histopathologic features
Many virusesCoxsackie and Echovirus
Nerve cell alterations most cases
70 NEUROPATHOLOGY REVIEW

Low mortality lum)eosinophilic cytoplasmic neuronal


Meningeal and choroid plexus lymphs inclusion with basophilic inner formation
E. Influenza Lyssa bodylack inner formation
Onset respiratory J. Herpes simplex
Abnormal behavior, disoriented, seizures, 70% fatal
coma No seasonal incidence
Neuronal degeneration, inflammation May see no obvious skin or visceral disease,
F. Measles the virus often replicates at a peripheral site
Mostly < age 10 yr before reaching the CNS
Low fatality Virus often later peripherally located and
may be reactivated by a variety of factors
Perivascular inflammation, demyelination
(stress, fever, sunlight, trauma, radiation)
Multinucleated giant cells, intranuclear and
intracytoplasmic inclusions Virus spreads from periphery to ganglion
via retrograde axoplasmic transport
G. Subacute sclerosing panencephalitis (SSPE)
Pathology
Fatal, measles virus
Congenitalacute encephalitis with vis-
Under age 12 yr
ceral lesion, herpes type I
Course: months to a few years
Adultsherpes type II, acute hemor-
Progressive dementia, myoclonus, seizures,
rhagic necrotizing encephalitis (basal fron-
ataxia, dystonia
tal, cingulate, temporal gyri, insula)
Pathology
Eosinophilic nuclear inclusions in glial
Gray matter subacute encephalitis cells (oligodendrocytes > astrocytes)
Eosinophilic nuclear and cytoplasmic K. Herpes zoster
inclusions, neuronal loss, microglial prolif- Skin lesions (shingles)vesicles in a sen-
eration, perivascular inflammation sory nerve dermatome distribution
Demyelination, oligodendroglial inclu- Acute cerebellar diseasenystagmus, dysar-
sions, reactive astrocytosis thria, ataxia
Cerebral atrophyold cases Pathology
May see occasional neurofibrillary tangles Dorsal gangliainflammation, petechiae,
H. Mumps degenerating neurons
210 d after onset Eosinophilic intranuclear inclusions
Low mortality Meningeal inflammation, granulomatous
Perivenous demyelination angiitis
1020% aseptic meningitis L. Cytomegalovirus
I. Rabies Infanttransplacental transmission, still-
Variable incubation (13 mo) birth, prematurity
Bite of infected animalskunk, racoon, etc. Adultrare, less extensive, immunocom-
Anxiety, psychomotor sensitivity, hydropho- promised states (1833% of AIDS patients
bia, dysphagia, seizures have evidence of CMV in brain)
Death 27 d Pathology
Pathology Infantsnecrosis, inclusions (neurons,
glial cells, endothelial cells), chorioretini-
Encephalomyelitis (especially brain stem) tis; endstagegliosis, calcification, hydro-
Glial nodules cephalus, microcephaly
Negri bodies (hippocampus, cerebel- Adultsmicroglial aggregates
CHAPTER 9 / CNS INFECTION 71

Large cells with intranuclear (Cowdry A) O. Human T lymphotropic virus type 1 (HTLV-1)
and cytoplasmic inclusions Retrovirus causing tropical spastic parapa-
May involve ependyma and subependy- resis
mal regions (hemorrhagic necrotizing ven- Myelin loss and axonal degeneration in spi-
triculoencephalitis and choroid plexitis) nal cord white matter, leptomeningeal
chronic inflammation and fibrosis, gliosis
M. Progressive multifocal leukoencephalopathy
(PML) Thoracic and lumbar regions particularly sus-
ceptible
Immunocompromised states
May see demyelination of optic and audi-
Progressive disturbances in motor and men- tory nerves
tal function and vision
V. Parasitic Infections
Papova virus (JC, SV40)DNA virus
A. Toxoplasmosis
Demyelinationmultifocal, asymmetric
Caused by Toxoplasma gondii infection
Astrocytosis Congenital form
Alzheimer I astrocytesbizarre, atypical
Occurs only if the maternal infection is
astrocytes; probable viral cytopathic effect
acquired during pregnancy
Intranuclear inclusionsoligodendrocytes
Miliary granulomata
(Cowdry A)
Paucity of lymphocytes Necrosis
N. HIV Calcification
RNA retrovirus Meningitis, chorioretinitis
10% develop aseptic meningitis within Adult
12 wk of seroconversion May rarely encounter cysts in an incompe-
Meningoencephalitis often presents with tent individual
dementia, seizures, myoclonus and focal Space-occupying lesion(s)abscesses,
motor signs may demonstrate the same architectural
Chronic inflammation with microglial features of a bacterial abscess
nodules Scattered necrotic foci
Microglial nodules often contain macro- Rarely microglial nodules/encephalitis
phage-derived multinucleated giant cells picture
Virus can be detected in macrophages, CD4- B. Amebiasis
positive lymphs and microglia
Entamoeba histolytica or lodamoeba buet-
Areas most severely affected include deep schlii
cerebral white matter, basal ganglia, cerebral
Naegleriavia nasal cavity, often acquired
cortex, and brain stem
by swimming in contaminated water, puru-
Vacuolar myelopathy involving degeneration lent meningitis, rapidly progressive, severe
of spinal cord (worst in thoracic cord), espe- involvement of the olfactory area (often the
cially posterior columns; presents as point of entry)
progressive paraparesis, sensory ataxia, and Acanthamoebadiffuse necrotizing granulo-
incontinence; swelling between myelin matous encephalitis and meningitis
sheaths, reactive astrocytosis, macrophage
infiltrate and occasional multinucleated giant Often see trophozoites in tissue section
cells. C. Malaria
AIDS in children frequently marked by calci- 2% of cases of Plasmodium falciparum
fication of parenchymal vessels within basal Presents with headache, drowsiness, confu-
ganglia and deep cerebral white matter sion, coma, fevers
At risk for opportunistic infections Often see the organisms in peripheral blood
72 NEUROPATHOLOGY REVIEW

Grossly, see opacity of meninges, dusky col- Conformational change promotes further pro-
oration of the gray matter, edema, con- duction of the abnormal prion protein
gestion material
Capillaries filled with organism Prion protein inactivated by autoclaving or
Small foci of ischemic necrosis rimmed by bleach
microglia (Durcks nodes) B. Creutzfeldt-Jakob disease (CJD)
White matter petechiae 1/106 incidence
Can see granules of dark malarial pigment Subacute dementia
in red cells Myoclonus
D. Cysticercosis EEGbiphasic, triphasic spikes
Taenia solium (pork tapeworm) Onset > 30 yr; female : male 2 : 1
Larvae encysts in brain Duration < 1 yr
Inflammation, granuloma, calcification Can evaluate CSF for 14-3-3 protein to sup-
E. Schistosomiasis port diagnosis
S. japonicumcerebrum lesions Pathology
S. mansoni, S. hematobiumspinal cord Spongiform change
lesions Neuronal atrophy/loss
Necrotizing inflammation with neutrophils, Gliosis
eosinophils, giant cells
Granular layer degeneration
Fibrosis and calcification with healing
Kuru plaque 10% (extracellular deposits
F. Trypanosomiasis
of aggregated abnormal proteinCongo
African disease caused by T. brucei; red and PAS positive), prominent in vari-
acquired by bite of tsetse fly; may present ant CJD, which may be related to inges-
with fever, rash, edema, cardiac involve- tion of contaminated beef (mad cow
ment, anemia; meningoencephalitis occurs disease)
during late stages of the disease
Iatrogenic transmission documented (e.g.,
South American disease (Chagas disease)
pooled pituitary hormone preparations,
caused by T. cruzi; acquired from feces of the
dura mater and corneal grafts/transplants)
reduviid bug; brain with edema, congestion,
hemorrhages, microglial nodules, granulomas C. Gerstmann-Straussler-Scheinker disease
G. A variety of other parasites have been Ataxia
described to cause CNS pathology, including Behavioral change
Echinococcus, Spirometria (Spanganosis), Para- Late dementia
gonimus (lung fluke), Trichinella spiralis, Loa Autosomal dominant, onset > 30 yr
Loa, Onchocerca, and Strongyloides
Duration5 yr
VI. Transmissible Spongiform Encephalopathies
1/10 million incidence
A. General
Corticospinal tract degeneration
Includes Creutzfeldt-Jakob disease,
No/mild spongiform change and astrocytosis
Gerstmann-Straussler-Scheinker, Kuru, famil-
ial insomia Cerebellar plaques
Related to prion protein abnormalties D. Kuru
Prion protein is a normal 30 Kd cellular pro- Cannibalism: Papua New Guinea; Fore
tein present in neurons (chromosome 20) tribewomen and children
Tremor, ataxia, dysphagia, euphoria
In disease, protein undergoes conformational
change from normal -helix isoform to an Duration of disease < 1 yr
abnormal -pleated sheet isoform (PrPsc or Severe cerebellum diseasekuru plaques,
PrPres) axon torpedoes, granular layer degeneration
CHAPTER 9 / CNS INFECTION 73

E. Familial insomnia Duration of disease < 2 yr


Sleep disorder, bizarre behavior, autonomic Severe anterior and medial thalamus gliosis,
dysfunction inferior olive degeneration
Adult onset, autosomal dominant
10 Skeletal Muscle

I. General Information Trichrome (Gomori)


A. Muscle biopsy Nuclei red
Do not biopsy muscle with needle in it Ragged red fibers (mitochondria)
(EMG), injection site (gluteus, deltoid),
Inclusion body myositis inclusion
severely diseased muscle
(rimmed vacuoles)
Commonly biopsied muscles: quadriceps
(vastus lateralis), triceps, biceps, and gas- Fibrosis
trocnemius PAS
B. Clinical information useful when interpreting Normal muscle with glycogen
a muscle biopsy
Type I > type II
Age
Glycogen storage diseases
Gender
Drugs/medications Oil-Red-O
Medical/family history Normally evenly distributed
EMG findings Type I > type II
C. A variety of enzyme histochemical stains and Lipid disorders
special stains are used in the evaluation of
(SAB) Sulfonated alcian blue/Congo red
biopsy specimens. Many of these stains
require fresh or frozen tissue to work. Amyloid
Improper freezing of the specimen may result Mast cells (SAB positiveinternal
in a vacuolar artifact. control)
D. Staining of the muscle biopsy specimen Nonspecific esterase
Routine hematoxylin and eosin
Angular atrophic fibers (neurogenic)
Acid phosphatase
Acetylcholinesterase
Lysosomal enzyme
Type I > type II
Increased with degeneration
(red staining) Acute denervation (6 mo)
Inflammatory cells Some inflammatory cells
Alkaline phosphatase NADH
Normal capillary basement stains Oxidative enzyme
Regenerating fibers (black staining) Type I > type II

75
76 NEUROPATHOLOGY REVIEW

Myofibrillary network inside muscle tem and result in secondary changes in the
fiber muscle
Cytochrome oxidase D. Neurogenic changes
Mitochondrial enzyme Angular atrophic fibers (esterase positive)
Type I > type II Group atrophy
Fiber type grouping
Focal deficiencies exist
Target and targetoid fibers
ATPase (adenosine triphosphatase)
Nuclear bags
pH 4.6 (reverse) or pH 4.3
No necrotic or basophilic fibers
Type I dark
Minimal interstitial fibrosis
Type IIA light
No inflammation
Type IIB intermediate
No centralized nuclei
Type IIC light (pH 4.2)
III. Inflammatory Myopathies
pH 9.8 (standard)
A. Polymyositis
Type I light
3060% of inflammatory myopathies
Type II dark
Nonhereditary
Succinate dehydrogenase (SDH)
Adults (age 4060 yr), second peak in
Mitochondrial enzyme childhood
Type I > type II Male : female 1 : 2
A variety of other stains may be selec- Idiopathic versus connective tissue disease
tively performed including myophosphory- (SLE, rheumatoid arthritis associated)
lase, myoadenylate deaminase, phospho- An association with neoplasms in adults
fructokinase, etc.
Proximal muscle weakness, often painful
E. Tissue is often processed in glutaraldehyde or
Fatigue, fever, weight loss
other fixative for potential electron micro-
scopic evaluation CPK and aldolase elevated
F. In cases of suspected metabolic disease, extra Most respond to steroids or other immuno-
frozen tissue for assay purposes is warranted. suppressives
II. Myopathic Versus Neurogenic Changes EMGfibrillation potential in acute stage,
polyphasic short duration potentials
A. Myopathic processes include such disease
entities as inflammatory myopathies, muscu- Histopathology
lar dystrophies, congenital myopathies and Increased variation of myofiber size,
metabolic diseases central nuclei
B. Myopathic changes Inflammation (lymphocytes and macro-
Irregularity of fiber size phages), endomysial (surrounding indi-
Rounded fibers vidual muscle fibers)
Centralized nuclei CD4-positive lymphocytes more com-
Necrotic and basophilic fibers mon around vessels
Cytoplasmic alterations CD8-positive lymphocytes more com-
Interstitial fibrosis mon in endomysium
Absence of enzyme staining Degeneration
Inflammation Regenerationbasophilic staining cyto-
Storage of glycogen and lipid plasm, nuclear enlargement with promi-
C. Neurogenic diseases include those processes nent nucleolus
that affect the peripheral/central nervous sys- Vasculitis (lymphocytic, nonnecrotizing)
CHAPTER 10 / SKELETAL MUSCLE 77

Endomysial fibrosis Rimmed vacuolesautophagic vacu-


Motheaten fibers oles, granular basophilic material on tri-
chrome stain
Ill-defined zones of enzyme loss
Rare intranuclear inclusions
Type I > type II
Frequent neurogenic atrophy-like com-
Nonspecific findings in myopathies
ponent
B. Dermatomyositis
D. Rheumatoid arthritis
Proximal muscle weakness
Type II atrophy
Heliotropic rash, Gottrons papules Moth-eaten fibers common
Dysphagia, arthralgias, constitutional Severe diseasetype I and type II atro-
symptoms phy, ring fibers, increased central nuclei
Childhood disease often associated with Chronic inflammation, either polymyositis
vasculitis and calcinosis, not associated or focal nodular myositis pattern of injury
with malignancy
Vasculitisusually non-necrotizing lymph-
Associated with connective tissue disease ocytic
Associated with malignancy in 730% of E. Bacterial myositis
cases, elderly patients (>5560 yr); cancers
Often related to skin injury
of the breast, ovary, lung, prostate, colon
Gas gangrene (Clostridium)
Histopathology
Pyomyositis (Staph)
Inflammationperivascular, may be
Neutrophils frequently present
scant, mixture of T- and B-lymphocytes
(vasculopathic disease) F. Viral myositis
May see non-necrotizing lymphocytic Muscle pain and muscle weakness usually
vasculitis, thrombosis not salient features
Influenza A and B
Degeneration
Enterovirus
Regeneration
HIV (AZT treatment)AZT may also
Perifascicular atrophyischemia related result in mitochondrial abnormality
C. Inclusion body myositis Coxsackie B (epidemic pleurodynia,
Most > 50 yr, males > females Bornholms disease)
Progressive, painless muscle weakness G. Parasitic myositis
Steroid resistant Trichinosis (Trichinella spiralis)
Enzymes sometimes normal Encysted larvae
EMGneurogenic picture Granulomatous inflammation with eosin-
Etiology not known ophils

Tubulofilaments (1518 nm) on electron Inadequately cooked meat


microscopy, most readily found in fibers Toxocariasis, Cysticercosis, Hydatidosis,
with rimmed vacuoles Toxoplasmosis, Sarcosporidiosis, Trypano-
May see amyloid protein material somiasis
Mitochondrial abnormalities frequent con- H. Fungal myositis
commitant finding Very rare
Histopathology Candida and cryptococcus the most
common
Inflammation
I. Vasculitis
Degeneration
May see vasculitic component to another
Regeneration inflammatory myopathic process (generally
78 NEUROPATHOLOGY REVIEW

non-necrotizing lymphocytic vasculitis) or Type grouping10 fibers of the


may be part of systemic vasculitis process same type contiguous along 1 edge
such as polyarteritis nodosa Target fibers
Polyarteritis nodosa (PAN) affects medium Type I > type II fibers
to small size arterial vessels causing Three zones of staining: central
fibrinoid necrosis of vessel walls
pale zone devoid of oxidative activ-
With PAN, will often see vessels at vari- ity; dark annular intermediate zone
ous stages of the disease process in the rich in oxidative enzymes; normal
same biopsy peripheral zone
May be associated with denervation atro- Targetoid fibers (core fibers)no
phy changes (presumably related to intermediate zone, type II predilec-
involvement of peripheral nerves) tion, a less specific finding
IV. Denervation Atrophy B. Spinal muscular atrophies
A. General information Includes those disorders that cause anterior
Group of disorders that affect anterior horn horn cell degeneration such as Werdnig-
cells, nerve roots, or peripheral nerves Hoffman disease, Kugelberg-Welander dis-
causing secondary changes in the skeletal ease, etc. (see Neurodegenerative diseases)
muscle Chromosome 5 (survival motor neuron
Types of neurogenic disease gene)
Motor neuron EMG findings confirm denervation
Amyotropic lateral sclerosis (ALS) CPK levels often normal or slightly ele-
Spinal muscular atrophy (Werdnig-
vated
Hoffmann, Kugelberg-Welander) Fascicular atrophy, rounded fiber atrophy
in infantile variant
Motor endplate
In later onset varients, see angular atrophic
Myasthenia gravis
esterase positive fibers instead of rounded
Peripheral (motor) neuropathy fiber atrophy
Metabolic Scattered type I hypertrophic fibers
Toxic Increased endomysial fibrofatty tissue
Vascular/connective tissue disease Increase in muscle spindle number
C. Myasthenia gravis
Systemic disease (diabetes, uremia)
Immune-mediated loss of acetylcholine
Infection receptors with circulating antibodies to ace-
Malignancies tylcholine receptor
Histopathology Prevalence of about 3/100,000 persons
Angulated fiber atrophy (esterase pos- In young patients (<40 yr), women > men;
itive) in older patients, equal gender distribution
Type I and type II fibers both involved Associated with thymic hyperplasia (65%)
and thymoma (15%)
Increased NADH staining in angular
Normal nerve conduction studies
atrophic fibers
Decreasing motor response with repeated
Nest-like fascicular atrophytwo con- stimulation
tiguous motor units involved
Presents with weakness of extraocular mus-
Nuclear bagsendstage cle, fluctuating generalized weakness
May see evidence of reinnervation Improved strength in response to acetylcho-
Collateral sprouting of spared motor linesterase agents
neurons Histopathology
CHAPTER 10 / SKELETAL MUSCLE 79

Type II atrophy Both type I and type II fibers affected


Ultrastructurally, may see simplification Nemaline rods not specific for nemaline
of the postsynaptic membrane myopathyreported in a variety of other
muscle disorders
Lymphorrhages (lymphoid aggregates)
Rods derived from Z-band material
in about 25% of cases
(-actinin)
May see evidence of mild denervation C. Central core disease
atrophy
Early-onset hypotonia and nonprogressive
D. Lambert-Eaton myasthenic syndrome muscle weakness
Usually develops as a paraneoplastic pro- Joint deformities such as hip dislocations
cess (especially with small cell carcinoma common
of lung)
May develop malignant hyperthermia
Proximal muscle weakness with autonomic
dysfunction Autosomal dominant condition related to
defect of ryanodine receptor-1 gene (chro-
Enhanced neurotransmission with repeated mosome 19q13.1)
stimulation
CPK generally normal or slightly elevated,
Decrease in the number of vesicles especially in later-onset cases
released in response to each presynaptic
action potential Cytoplasmic cores devoid of enzyme activ-
ity (NADH negative) in many fibers
Some patients with antibodies to presynap-
tic calcium channels Cores may be eccentrically placed and mul-
tiple
V. Congential Myopathies
Cores generally confined to type I fibers
A. General information
Ultrastructurally, cores are zones in which
Disorders of muscle fiber maturation and myofilaments are densely packed with
reorganization absence of mitochondria, glycogen, and
Nonprogressive/slowly progressive lipids
Retarded motor development, muscle Minicore or multicore disease is a distinct
weakness autosomal recessive condition
Frequently associated with skeletal dysm- D. Centronuclear (myotubular) myopathy
orphism Presents in infancy or childhood with
May present as floppy baby due to hypo- involvement of extraocular and facial mus-
tonia or with joint contractures (arthro- cles, hypotonia, and slowly progressive
gryposis) limb muscle weakness
Familial incidence CPK may be normal or slightly elevated
Specific structural abnormality of muscle X-linked autosomal recessive or sporadic
fibers variants
B. Nemaline myopathy Increased numbers of centrally placed
Weakness, hypotonia, and delayed motor nuclei (involving most myofibers)
development in childhood, may present in Most fibers containing central nuclei are
adulthood type I and tend to be somewhat atrophic
Autosomal recessive, autosomal dominant Small zones surrounding the nuclei that
and sporadic variants recognized are devoid of myofibrils and filled with
CPK often normal or mildly elevated mitochondrial aggregates, lipid, and gly-
cogen
Intracytoplasmic nemaline rodlets (red on
trichrome) E. Congenital fiber type disproportion
Number of affected fibers has no correla- Autosomal recessive condition
tion with disease severity Variable presentation clinically; hypotonia,
80 NEUROPATHOLOGY REVIEW

long thin face, contractures, congenital hip Retinitis pigmentosa


dislocation, respiratory problems
Ophthalmoplegia
Mean fiber diameter of type I fibers at
least 12% smaller than type II fibers Atrial-ventricular (AV) block
May also see increased numbers of central- Cerebellar ataxia
ized nuclei, moth-eaten fibers and nema- Deafness
line rods
Peripheral neuropathy
VI. Metabolic Myopathies
Dwarfism
A. Mitochondrial myopathy
Mitochondrial abnormalities in other
Many related to mutations in either nuclear
organs (liver, skin, brain)
or mitochondrial genes
Brain with calcification
Three categories of genetic alterations
B. Glycogenoses
Class I mutations: involve nuclear DNA
(e.g., Leighs syndrome) Abnormalities of glycogen metabolism
The most common ones to affect skeletal
Class II mutations: point mutations of
muscle are type II (acid maltase defi-
mitochondrial DNA (e.g., MERFF and
ciency) and type V (myophosphorylase
MELAS)
deficiency, McArdles disease).
Class III mutations: deletions or duplica- Less commonly, other types may affect
tions of mitochondrial DNA (e.g., muscle, including type III (amylo 1,6-glu-
Kearns-Sayre syndrome) cosidase deficiency, debrancher enzyme),
May be related to abnormalities of mito- type IV (branching enzyme) and type VII.
chondrial transport, substrate utilization, (phosphofructokinase deficiency)
Krebs cycle, oxidativephosphorylation Type II glycogenosis (Pompes disease)
coupling, or the respiratory chain (the most
Acid maltase deficiency
common group of mitochondrial myopa-
thies) Chromosome 17q autosomal recessive
Myopathy only presentation in some condition
patients Neonatal form with hypotonia and cardi-
Cerebral dysfunction with myopathy in omegaly, rapidly fatal
some patients (mitochondrial encephalomy- Childhood and adult form with limb
opathies) girdle weakness and cardiac disease
Progressive external ophthalmoplegia, prox- Elevated CPK in all forms
imal muscle weakness
Type I fibers with vacuoles
Ragged red fibers (Gomori trichrome
stain)related to aggregates of often mor- Vacuoles PAS and acid phosphatase
phologically abnormal mitochondria situ- (acid maltase is a lysosomal enzyme)
ated in a subsarcolemmal location) positive
Not all myofibers equally affected Glycogen accumulation on electron
Often nonspecific increase in lipid and microscopy
glycogen Type V glycogenosis (McArdles disease)
Sporadic and inherited forms Myophosphorylase deficiency
Abnormal mitochondrial structure on elec- Autosomal recessive condition, chromo-
tron microscopic studies, paracrystalline or some 11q
parking lot inclusions
Males > females
Example: oculocraniosomatic syndrome
Muscle cramps and stiffness on exertion
(Kearns-Sayre syndrome, ophthalmoplegia
plus) Abnormal ischemic exercise testdo
CHAPTER 10 / SKELETAL MUSCLE 81

not get the expected elevation of blood Most are autosomal dominant, males >
lactate levels following exercise females
Vacuolar myopathytypically subsarco- Myotonia, relapsing episodes of hypotonic
lemmal blebs paralysis following exercise, exposure to
cold or high-carbohydrate meal
Vacuolar PAS positive, acid phospha-
tase negative Variants recognized associated with varied
serum potassium levels (normokalemic,
C. Lipid storage myopathies
hyperkalemic, or hypokalemic periodic
Resulting from deficiencies in the carnitine paralysis)
transport system or deficiencies in the mito-
Myotonia congenita form linked to the
chondrial dehydrogenase enzyme system
chloride channel (chromosome 7)
Related to deficiencies of carnitine, acyl-
Hyperkalemic forms and paramyotonia con-
CoA dehydrogenase, or carnitine palmitoyl-
genita form (childhood disorder marked by
transferase (CPT)
myotonia and hypotonia that increases
Carnitine deficiency may be either myo- with continued exercise, especially in the
pathic (limited to muscle) or systemic cold) linked to the sodium channel (chro-
Myopathic carnitine deficiency presents mosome 17)
with generalized weakness, usually starting PAS-positive vacuoles resulting from dila-
in childhood tation of the sarcoplasmic reticulum
CPT deficiency often presents with myoglo- Tubular aggregates, highlighted by NADH
binuria (rhabdomyolysis following stain
exercise)
VII. Toxic Myopathies
Histologically marked by an accumulation
of lipid within predominantly type I myo- A. Rhabdomyolysis
cytes (highlighted with oil-red-O or Sudan Necrosis (degeneration)
black stain) Regeneration
D. Myoadenylate deaminase deficiency Scant inflammation
Intolerance to exercise with cramps, stiff- Rarely calcification
ness, or pain
Seen with drugs, metabolic myopathy,
Autosomal recessive condition viral myositis, malignant hyperthermia,
CPK levels high with exertion potassium deficiency, trauma/ischemia,
Type II fibers normally contain more of idiopathic
the enzyme than type I fibers Caused by drugs and toxic agents, include
Can stain for myoadenylate deaminase to alcohol, heroin, carbon monoxide, barbitu-
demonstrate deficiency rates, amphetamines, methadone, amphoter-
icin B
E. Malignant hyperthermia
Rapid and sustained rise in temperature B. Subacute necrotizing myopathyalcohol,
during general anesthesia AZT, clofibrate
Generalized muscle rigidity, tachycardia, Ethanol ingestion
cyanosis May also cause rhabdomyolysis
Extensive myonecrosis with subsequent Acute muscle pain that may be generalized
renal failure may develop or localized
Autosomal dominant condition C. Inflammatorycimetidine, procainamide,
Caffeine or anesthetic stimulation test on D-penicillamine
muscle strips in vitro D. Hypokalemicdiuretics, laxatives, licorice
Muscle pathology nonspecific E. Type IIb fiber atrophysteroids
F. Channelopathies F. Vacuolar chloroquine myopathy
Ion channel myopathies Proximal muscle weakness
82 NEUROPATHOLOGY REVIEW

Autophagic membrane bound vacuoles con- Deathearly twenties in Duchenne dys-


taining membranous debris trophy
Curvilinear bodies Becker dystrophy less severe with later
Vacuoles more common in type I fibers age of onset and slower rate of progression
with normal life expectancy
Variable degrees of myocyte necrosis
G. Hyperthyroid myopathy Dystrophin absent in Duchenne dystrophy;
decreased or abnormal dystrophin with
Thyrotoxic myopathyproximal muscle Becker dystrophy
weakness, biopsies often normal
Dystrophin gene (chromosome Xq) abnor-
Hypokalemic periodic paralysis rarely malites may be result of deletions, point
develops with vacuoles mutations, or frameshift changes
Associated with myasthenia gravis and
Dystrophina 427 KD protein associated
exophthalmic ophthalmoplegia
with the sarcolemmal membrane and local-
VIII. Muscular Dystrophies ized to I and M bands
A. Traditionally classified by pattern of inheri- Histopathology overlapping between
tance Duchenne and Becker dystrophy
X-linked recessive Variable muscle fiber sizeatrophy and
Duchenne hypertrophy with central nuclei
Becker Large ballooned fiber and hypercon-
Emery-Dreifuss tracted fibers
Autosomal dominant Nests of regeneration
Facio-scapulo-humeral Muscle fiber degeneration
Myotonic Endomysial fibrosis
Oculopharyngeal Type I predominance
Distal dystrophic myopathy D. Emery-Dreyfuss dystrophy
Autosomal recessive Onset in childhood
Limbgirdle (some autosomal dom- Difficulty in walking, rigidity of neck or
inant) spine, elbow deformity
Congenital Slowly progressive
B. Many of them are being reclassified based on Cardiac involvement common
underlying defects Histopathology marked by myopathic
C. Duchenne dystrophy/Becker dystrophy changes such as variation in fiber size,
Males increased central nuclei, degenerating
fibers, fibrosis
Rarely spontaneous mutation (about one-
third of cases) Absent emerin by immunohistochemistry
Early-onset pelvic limbgirdle (proximal), E. Facio-scapulo-humeral dystrophy
wheelchair dependence by age 1012 yr in Variable age of onset (usually between
Duchenne dystrophy ages 10 and 30 yr)
Pseudohypertrophic calvesrelated to Weakness of muscles of face, neck, and
increased fat and connective tissue shoulder girdle
CPK increased A subgroup related to defect on chromo-
Cardiac involvement (heart failure or some 4q35
arrhythmias) Histopathology characterized by fiber size
Subset of patients have structural abnormal- variation, chronic inflammation, moth-
ities in the CNS with mental retardation eaten and lobulated fibers
CHAPTER 10 / SKELETAL MUSCLE 83

F. Myotonic dystrophy Difficulty swallowing


Most patients young and middle-aged, Gene defect on chromosome 14q
most common of adult-onset dystrophies Histopathology characterized by vacuoles
Facial, distal amyotrophy of limbs, myo- with tubulofilamentous structures by elec-
tonic (sustained involuntary contractions of tron microscopy, variability in fiber size,
muscles) syndrome increased central nuclei
Systemic involvement: frontoparietal bald- H. Limbgirdle dystrophy
ing, cataracts, cardiac disease, hypoplasia Wide variety of subtypes recognized as
of genital organs related to different gene defects (caveolin
An inherited neonatal form exists 3, calpain 3, sarcoglycan)
Gene located on chromosome 19q13 Onset of different subtypes at different
encodes myotonin protein kinase ages
Trinucleotide (CTG) repeat disorder Weakness of pelvi-femoral muscles and
Histopathology shouldergirdle muscles
Type I atrophy and type II hypertrophy Pathology marked by fiber splitting,
increased central nuclei, variability in fiber
Chains of nuclei with increased internal size, degenerating fibers, fibrosis, regenerat-
nuclei ing fibers, moth-eaten fibers, whorled
Acid phosphatase hyperactivity fibers, and ring fibers
Variable fibrosis, degeneration, regener- I. Congenital muscular dystrophy
ation Neonatal hypotonia, respiratory insuffi-
ciency, delayed motor milestones, con-
Ring fibers commonrim of fiber con-
tractures
tains myofibrils that are oriented circum-
ferentially around the longitudinally ori- A subgroup is merosin negative (related to
ented fibrils in the rest of the fiber mutation in the merosin laminin II gene)
G. Oculopharyngeal dystrophy Pathology marked by variability in fiber
size, fibrosis, degenerating and regenerat-
Onset in mid-adult life
ing myofibrers, increased central nuclei
Ptosis and extraocular muscle weakness
11 Peripheral Nerve

I. General Background Crush traumacauses axonal swelling, gaps


A. Incidence of peripheral neuropathy-40/100,000 in myelin
B. Clinical patterns include polyneuropathy, mono- Fixation artifactsformalin fixation may
neuropathy, mononeuropathy multiplex, and cause neurokeratin or herringbone appear-
plexopathy ance of myelinated fibers, fixation in a hyper-
osmolar solution may cause shrinkage and
C. Etiologies quite variable
loss of circular profile
Acquired toxic or metabolic (50%), include
II. Major Pathologic Processes
such entities as diabetes, alcoholism, vitamin
B12 deficiency, drug toxicities A. Axonal (Wallerian) degeneration
Inflammatory/infectious (1020%), includes Result of primary destruction of the axon
such entities as Guillian-Barre syndrome, with secondary degeneration of myelin
chronic inflammatory demyelinating poly- Damage to axon may be focal (trauma, isch-
radiculopathy, vasculitis, leprosy emia) or more generalized because of abnor-
Neoplastic or paraneoplastic (510%) malities affecting the neuron cell body (neu-
Genetic/inherited abnormalities (1020%), ronopathy) or axon (axonopathy)
includes the hereditary motor and sensory Wallerian degeneration occurs as result of
neuropathies focal lesiontraumatic transection of nerve
Unknown (1020%) Distal to site of transectionacute nerve
D. Evaluation of the patient with suspected periph- ischemia with inflammation (macrophages to
eral nerve diseasehistory (onset, course, clean up myelin debris)
other diseases, medications, family history, Axons undergo alterations of the cytoskele-
exposures), physical examination, electrodiag- ton and organelles, watery axoplasm
nostic testing with EMG/nerve conduction stud- Myelin undergoes fragmentation forming
ies, therapeutic trial, biopsy myelin ovoids with paranodal retraction
E. Procedures involved in handling of the biopsy With axonal regeneration, may see a prolifer-
specimen ation of Schwann cells within pre-existing
Fixation in formalin (routine light micros- basal laminabands of Bungner
copy) and glutaraldehyde (electron A regenerative cluster a result of the growth
microscopy) of one or more growth cones into a band of
Teased nerve preparation Bungner
Frozen tissue Eventually, remyelination occursnew
F. Artifacts associated with biopsy specimen pro- myelin thinner caliber, shortened internodal
cessing distances
Stretch traumacauses myelin splitting Regenerative process slow, limited by the

85
86 NEUROPATHOLOGY REVIEW

slowest component of axonal transport nerve sheath and the misalignment of fas-
(2 mm/d) cicles
B. Distal axonopathy Regrowth may result in a disorganized mass
Dying back axonopathy of axonstraumatic or amputation neuroma
Entire neuron diseased Compression neuropathy related to carpal
tunnel syndrome (compression of the median
Initial changes seen in portion farthest away nerve at the level of the wrist)
from the cell body
In entrapment neuropathies, the compressed
Marked by axonal swelling, filamentous region narrowed with nerve expansion proxi-
inclusions, axonal atrophy, organelle alter- mal to and distal to the region, myelinated
ations, and Schwann cell networks fiber size decreased and may see evidence of
C. Demyelination (segmental) axonal degeneration and/or demyelination
Primary demyelination related to Schwann Other common sites of compression neuropa-
cell dysfunction (e.g. diphtheritic neuropa- thy the ulnar nerve at the elbow, peroneal
thy) or myelin sheath damage nerve at the knee, radial nerve in the upper
Degenerating myelin engulfed by Schwann arm, and interdigital nerve of the foot at
cells and macrophages intermetatarsal sites (Mortons neuroma)
Injured Schwann cells may be replaced by Radiation may cause fiber loss and fibrosis
cells in the endoneurium B. Guillain-Barre syndrome
Axons surrounded by degenerating myelin AKA: acute inflammatory demyelinating
or denuded myelin polyradiculopathy
Denuded axon provides stimulation for remy- Overall annual incidence of 13/100,000 in
elination United States
Newly formed myelin of thinner caliber Distal limb weakness, rapidly progressing to
more proximal weakness (ascending
Shortened internodal distances with remyelin-
paralysis)
ation
Majority of cases with antecedent viral
With repeated episodes of demyelination and
illness
remyelination, a layering of Schwann cell
processes (concentric layers of Schwann cell Deep tendon reflexes diminish
cytoplasm and basement membrane) around Decreased nerve conduction velocity
a thinly myelinated axon forms onion bulb Elevated CSF protein
With chronicity, some degree of axonal Mortality rate 25%, most commonly related
injury may occur to respiratory problem or autonomic insta-
May get secondary demyelination related to bility
axonal injury Pathology
Rare defects of hypomyelination (myelin Endoneurial and perivascular chronic
sheaths thin in all fibers [e.g., hereditary inflammation (lymphocytes, plasma cells
motor and sensory neuropathy type III]) and and macrophages)
hypermyelination (abnormal numbers of
myelin lamellae often related to folding and Segmental demyelination
reversal of the myelin spiral) C. Chronic inflammatory demyelinating polyradicu-
III. Neuropathy Types lopathy (CIDP)
A subacute or chronic course (at least 8 wk)
A. Traumatic
Association with HLA-B8
May be caused by lacerations (cutting
injury), avulsions (pulling apart), or traction Typically marked by relapses and remissions
Regeneration of the transected nerve may be Most commonly a symmetric, mixed sensori-
slow, problems related to the discontinuity motor polyneuropathy
of the proximal and distal portions of the May respond to steroids or plasmapheresis
CHAPTER 11 / PERIPHERAL NERVE 87

Pathology marked by demyelination pattern HIV


of injury with some onion bulb formation May encounter acute and chronic polyneu-
developing over time ropathies
D. Infectious neuropathies
May resemble Guillian-Barre disease or
Leprosy CIDP
Infection by Mycobacterium leprae Most common HIV-related neuropathy:
Also affects primarily skin and mucous painful symmetrical sensorimotor neuropa-
membranes thy (mixture of axonal degeneration and
Symmetric polyneuropathy especially demyelination)
involving pain fibers May encounter CMV infection
Lepromatous leprosy marked by Schwann E. Malignancyassociated neuropathies
cell invasion by organisms, segmental Direct infiltration or compression of nerve
demyelination and remyelination, and loss by tumor (mononeuropathy)
of axons Paraneoplastic neuropathy
With chronicity, fibrosis and perineurial Most often encountered with bronchial
thickening occurs carcinoma (small cell) associated with
Tuberculoid leprosy marked by nodular IgG anti-Hu antibody
granulomatous inflammation, particularly Subacute sensory neuronopathy or sensori-
affecting cutaneous nerves, sparse num- motor neuropathy
bers of organisms
Depletion of ganglion cells in dorsal root
Diphtheria
ganglion with a proliferation of capsule
Neuropathy related to the effects of an cells and perivascular chronic inflam-
exotoxin mation
Often presents with weakness and pares- Dysproteinemic neuropathy
thesias, loss of proprioception and vibra- Monoclonal gammopathies of undeter-
tory sensation mined significance
Earliest changes observed in the sensory Most commonly associated with IgM para-
ganglia proteins, usually with light chain
Demyelination changes
Males > females
Varicella-zoster
Chronic distal sensorimotor neuropathy
Affected ganglia marked by neuronal with tremor and ataxia
destruction and loss
Demyelinative neuropathy with hypertro-
Prominent chronic inflammation phic changes and widely spaced myelin
May encounter focal necrosis and hemor- Anti-myelin-associated protein (MAG)
rhage antibodies in 5090% of cases
Axonal degeneration F. Hereditary motor and sensory neuropathies
Lyme disease (HMSN)
Caused by infection by Borrelia burgdorf- Charcot-Marie-Tooth disease, hypertrophic
eri spirochete type (HMSN I)
Acquired from a tick bite Presents in childhood/early adulthood,
Rash (erythema chronicum migrans), car- slowly progressive
ditis, arthritis Autosomal dominant
Frequently affects peripheral nerves Peroneal muscular atrophy, pes cavus
Perivascular chronic inflammation with Three subtypes: IAduplication on chro-
axonal degeneration mosome 17p11.2p12 peripheral myelin
88 NEUROPATHOLOGY REVIEW

protein 22; IBchromosome 1 myelin Segmental demyelination with onion


protein zero; ICnot known bulbs
X-linked variety associated with mutation Linear inclusions in Schwann cells
of gap junction protein gene connexin-32
Metachromatic leukodystrophy
Demyelinating neuropathy with prominent
Reduction in myelinated fiber number
onion bulb formation and hypertrophic
changes Segmental demyelination and remyelin-
HMSN II ation
Neuronal form of Charcot-Marie-Tooth Perinuclear granules in Schwann cells,
disease which stain metachromaticallymem-
brane bound, lamellated inclusions (prism
No nerve enlargement as in HMSN I stacks, tuff stone bodies, and zebra
Presents at a slightly later age than bodies)
HMSN I Globoid cell leukodystrophy
Chromosome 1p abnormality Moderate reduction in myelinated axons
Loss of axons Segmental demyelination and remyelin-
Dejerine-Sottas disease (HMSN III) ation
Slowly progressive, autosomal recessive Schwann cells contain straight or curved
Presents in early childhood with delayed prism/tubular inclusions
developmental milestones Refsums disease
Trunk and limb muscle involved Phytanoyl CoA -hydroxylase abnormal-
Nerve enlargement ity (peroxisomal enzyme)
Demyelination with onion bulb formation, Autosomal recessive
some axonal loss Onset before age 20 yr
G. Other hereditary neuropathies Ataxia, night-blindness, retinitis pigmen-
Hereditary sensory and autonomic neuropa- tosa, ichthyosis
thies
Hypertrophic nerves, mixed motor and
Type Iautosomal dominant, axonal sensory neuropathy
degeneration, sensory neuropathy present-
Prominent onion bulb formation
ing in adults
Porphyria
Type IIautosomal recessive, sensory
neuropathy presenting in infancy, axonal Peripheral nerve involvement with acute
degeneration intermittent and variegate coproporphyria
types
Type IIIRiley-Day syndrome, autoso-
mal recessive chromosome 9q, autonomic Autosomal dominant
neuropathy of infancy, axonal degenera- Acute episodes of neurologic dysfunction,
tion of unmyelinated axons, atrophy and psychiatric abnormalities, abdominal pain,
loss of sensory and autonomic ganglia seizures, autonomic dysfunction, proximal
Adrenoleukodystrophy weakness
X-linked Distal axonopathy of dying back type
Mixed motor and sensory neuropathy H. Nutritional deficiency/toxic neuropathies
Adrenal insufficiency Beriberi (thiamine deficiency)
Spastic paraplegia Distal, symmetrical sensorimotor neu-
Axonal degeneration (both myelinated ropathy
and unmyelinated axons) Axonal loss and degeneration distally
CHAPTER 11 / PERIPHERAL NERVE 89

Secondary segmental demyelination more thy accompanied by some segmental


proximally demyelination
Alcoholic neuropathy has a similar patho- Arteriolar thickening
logic appearance Uremic neuropathy
Vitamin B12 deficiency Distal, symmetric neuropathy
Symmetrical sensory polyneuropathy May be asymptomatic or associated with
muscle cramps, dysesthesias, and
May not necessarily be associated with
decreased deep tendon reflexes
subacute combined degeneration of the
cord Axonal degeneration
Axonal degeneration Regeneration and some recovery after
dialysis
Vitamin E deficiency
Amyloid neuropathy
Spinocerebellar degeneration and sensory
May encounter with myeloma, Walde-
neuropathy
nstroms macroglobulinemia
Related to chronic intestinal fat malab- Hereditary syndromes existmost com-
sorption monly related to mutations in transthy-
Loss of myelinated axons retin (prealbumin) gene
Toxic neuropathies Hereditary forms most commonly autoso-
mal dominant, often marked by sensory
A wide spectrum of toxic neuropathies
and autonomic dysfunction
Diagnoses depend on excluding other Amyloid deposits primarily in vessel
causes of neuropathy walls with axonal degeneration
I. Miscellaneous neuropathies
Neuropathy usually not associated with
Diabetic neuropathy secondary amyloidosis
Wide variety of diseases may be encoun- Vasculitic neuropathy
tered: distal symmetric or sensorimotor Axonal degeneration
neuropathy, autonomic neuropathy (20
May be related to a variety of different
40% of diabetics), focal/multifocal asym-
metric neuropathy vasculitic processes: polyarteritis nodosa
(necrotizing), Churg-Strauss, collagen
Decreased sensation in distal extremities vascular disease
Prevalence of peripheral neuropathy Sarcoidosis
related to duration of disease May be multifocal
Almost all patients with conduction abnor- Perivascular or endoneurial non-necrotiz-
malities ing granulomas
Predominant pathology: axonal neuropa- Nerve findings nonspecific
12 Figures with Questions

Question 1

1. The lesion shown here is almost exclusively seen


in the setting of
A. Neurofibromatosis type I
B. Neurofibromatosis type II
C. Tuberous sclerosis
D. Neurocutaneous melanosis
E. Li-Fraumeni syndrome

91
92 NEUROPATHOLOGY REVIEW

Question 2

2. The most common cytogenetic abnormality associ- 4. This lesion presented as a lateral ventricular mass
ated with this lesion is related to chromosome in an 18-yr-old male with seizures and adenoma
A. 1 sebaceum. The lesion represents a
B. 6 A. Cortical tuber
C. 9 B. Subependymal giant-cell astrocytoma
D. 17 C. Hemangioma
E. 22 D. Neurofibroma
3. The most likely diagnosis is E. Hemangioblastoma
A. Metastatic carcinoma
B. Lymphoma
C. Glioblastoma multiforme
D. Meningioma
E. Medulloblastoma
CHAPTER 12 / FIGURES WITH QUESTIONS 93

Question 3

Question 4
94 NEUROPATHOLOGY REVIEW

Question 5

Question 6

5. These brightly eosinophilic staining, irregularly 6. The cells in the intravascular lesion stain posi-
shaped structures in a pilocytic astrocytoma are tively with CD20. The diagnosis
referred to as A. Metastatic carcinoma
A. Rosenthal fibers B. Metastatic pituitary adenoma
B. Granular bodies C. Vasculitis
C. Marinesco bodies D. Lymphoma
D. Tangles E. Histiocytosis X
E. None of the above
CHAPTER 12 / FIGURES WITH QUESTIONS 95

Question 7

Question 8

7. This lesion represents a pineal gland mass in a 8. This right arm lesion in an area of previous
17-yr-old male. trauma represents a
A. Lymphoma A. Neurofibroma
B. Pineocytoma B. Schwannoma
C. Germinoma C. Mortons neuroma
D. Pineoblastoma D. Traumatic neuroma
E. Epithelioid glioma E. Paraganglioma
96 NEUROPATHOLOGY REVIEW

Question 9

Question 10

9. The malformation illustrated is best denoted as 10. This lesion is best classified as a(n)
A. Lissencephaly A. Astrocytoma
B. Polymicrogyria B. Ependymoma
C. Schizencephaly C. Meningioma
D. Periventricular leukomalacia D. Metastatic carcinoma
E. Hydranencephaly E. Schwannoma
CHAPTER 12 / FIGURES WITH QUESTIONS 97

Question 11

Question 12

11. The numerous round to oval calcified structures 12. Collars of perivascular meningothelial cells mark
in this tumor are termed this lesion, which is most commonly seen associ-
A. Calcified parasite eggs ated with
B. Marinesco bodies A. Neurofibromatosis type I
C. Hyaline bodies B. Neurofibromatosis type II
D. Psammoma bodies C. Tuberous sclerosis
E. Corpora amylacea D. von Hippel-Lindau disease
E. Sturge-Weber disease
98 NEUROPATHOLOGY REVIEW

Question 13

Question 14

13. This lesion is extending into the dura from an 14. This lateral ventricular mass in a 32-yr-old male
adjacent sinus. The diagnosis is stains positively with synaptophysin and nega-
A. Mucor infection tively with GFAP. The diagnosis is
B. Aspergillus infection A. Oligodendroglioma
C. Squamous cell carcinoma B. Clear cell ependymoma
D. Meningioma C. Neurocytoma
E. Adenocarcinoma D. Subependymal giant-cell astrocytoma
E. Subependymoma
CHAPTER 12 / FIGURES WITH QUESTIONS 99

Question 15

Question 16

15. The diagnosis is 16. This bone-based, sacrococcygeal mass microscopi-


A. Anaplastic astrocytoma cally consists of vacuolated epithelioid cells
arranged against a mucoid matrix. The most
B. Schwannoma
likely diagnosis is
C. Ependymoma
A. Chordoma
D. Neurofibroma
B. Paraganglioma
E. Leiomyoma
C. Syncytial meningioma
D. Myxopapillary ependymoma
E. Teratoma
100 NEUROPATHOLOGY REVIEW

Question 17

Question 18

17. The structure marked by an arrow in this cerebel- 18. Cerebrospinal fluid cytology shows atypical cells
lar mass in a 6-yr-old is a from a 4-yr-old males cerebellar mass (synapto-
A. Fleurette physin positive). The diagnosis is
B. Homer Wright rosette A. Ependymoma
C. Flexner Wintersteiner rosette B. Pilocytic astrocytoma
D. Perivascular pseudorosette C. Medulloblastoma
E. None of the above D. Choroid plexus papilloma
E. Neurocytoma
CHAPTER 12 / FIGURES WITH QUESTIONS 101

Question 19

Question 20

19. The lesion illustrated in this 4-yr-old with cere- 20. The ultrastructural finding here is from a Tay-
bral palsy is most likely the result of a Sachs disease patient and is known as
A. Storage disease A. Whorled membrane bodies
B. Vascular insult B. Granular bodies
C. Tumor C. Fingerprint bodies
D. Hemorrhage D. Curvilinear bodies
E. Inherited syndrome E. Zebra bodies
102 NEUROPATHOLOGY REVIEW

Question 21

Question 22

21. The defect seen here is best classified as 22. The most likely diagnosis for this well-circum-
A. Agenesis of the corpus callosum scribed intramedullary spinal cord mass is
B. Partial agenesis of the corpus callosum A. Schwannoma
C. Polymicrogyria B. Meningioma
D. Dandy-Walker syndrome C. Astrocytoma
E. Schizencephaly D. Ependymoma
E. Oligodendroglioma
CHAPTER 12 / FIGURES WITH QUESTIONS 103

Question 23

Question 24

23. The most likely diagnosis in this 8-yr-old female 24. This intradural, chromogranin-positive mass in
with tuberous sclerosis is the cauda equina region represents a
A. Cortical tuber A. Paraganglioma
B. Cortical dysplasia B. Schwannoma
C. Subependymal giant-cell astrocytoma C. Myxopapillary ependymoma
D. Metastatic germ cell tumor D. Meningioma
E. Angiomyolipoma E. Chordoma
104 NEUROPATHOLOGY REVIEW

Question 25

25. The most common location for this lesion is 27. What is the diagnosis?
A. Frontal lobe A. Metastatic carcinoma
B. Parietal lobe B. Craniopharyngioma
C. Occipital lobe C. Pituitary adenoma
D. Temporal lobe D. Meningioma
E. Cerebellum E. Astrocytoma
26. The best diagnosis for this lesion is
A. Herniation
B. Duret hemorrhage
C. Chiari malformation
D. Brain stem glioma
E. Dandy-Walker syndrome
CHAPTER 12 / FIGURES WITH QUESTIONS 105

Question 26

Question 27
106 NEUROPATHOLOGY REVIEW

Question 28

28. The most likely diagnosis is 30. The most likely diagnosis for this tumor that is
cytokeratin AE1/3 negative, S-100 protein posi-
A. Colloid cyst
tive is
B. Rathkes cleft cyst
A. Metastatic lung carcinoma
C. Neurenteric cyst
B. Glioblastoma multiforme
D. Arachnoid cyst
C. Melanoma
E. Endodermal cyst
D. Lymphoma
29. The most likely location for this tumor in a
E. None of the above
12-yr-old is
A. Frontal lobe
B. 4th ventricle
C. Cerebellum
D. Lateral ventricle
E. Temporal lobe
CHAPTER 12 / FIGURES WITH QUESTIONS 107

Question 29

Question 30
108 NEUROPATHOLOGY REVIEW

Question 31

31. Which chromosome is affected in this disorder? 33. The abnormality seen is most likely associated
A. 17 with which of the following?
B. 9 A. Neu-Laxova syndrome
C. 16 B. Zellwegers syndrome
D. 3 C. Walker-Warburg syndrome
E. 22 D. Cerebro-ocular dysplasia
32. The most likely hormone secreted by this lesion E. Lissencephaly
would be
A. Prolactin
B. Growth hormone
C. ACTH
D. TSH
E. FSH
CHAPTER 12 / FIGURES WITH QUESTIONS 109

Question 32

Question 33
110 NEUROPATHOLOGY REVIEW

Question 34

34. The hemorrhage seen here in the periventricular 36. The spinal cord tumor seen here ultrastructurally
region is most likely to occur in a(n) is best classified as
A. <32-wk-gestation premature infant A. Astrocytoma
B. Full-term infant B. Ependymoma
C. 1-mo-old infant C. Meningioma
D. 1-yr-old infant D. Schwannoma
E. An adult E. Paraganglioma
35. The lesion seen here represents
A. Multiple sclerosis
B. Huntingtons chorea
C. Multisystem atrophy
D. Agenesis of the septum pellucidum
E. Cavum septi pellucidi
CHAPTER 12 / FIGURES WITH QUESTIONS 111

Question 35

Question 36
112 NEUROPATHOLOGY REVIEW

Question 37

37. This temporal lobe tumor arose in a 14-yr-old 39. The changes seen here are most characteristic of
with a history of chronic epilepsy. The MIB-1 A. Dandy-Walker syndrome
labeling index was <1%. The most likely diagno- B. Chiari malformation
sis is
C. Brain stem glioma
A. Pleomorphic xanthoastrocytoma
D. Dentate dysplasia
B. Glioblastoma multiforme
E. Downs syndrome
C. Dysembryoplastic neuroepithelial tumor
D. Pilocytic astrocytoma
E. Ependymoma
38. A reticulin stain highlights spindled cell areas of
this tumor. Nonspindled areas stain with GFAP.
The tumor demonstrates focal vascular prolifera-
tion and necrosis. The best diagnosis is
A. Malignant oligodendroglioma
B. Malignant ependymoma
C. Gliosarcoma
D. Metastatic carcinoma
E. Lymphoma
CHAPTER 12 / FIGURES WITH QUESTIONS 113

Question 38

Question 39
114 NEUROPATHOLOGY REVIEW

Question 40

Question 41

40. The ependymal-lined cavity in this section of spi- 41. This patient is likely to have all the following
nal cord is best termed except
A. Syrinx A. Lisch nodules
B. Syringobulbia B. Pheochromocytoma
C. Aqueductal malformation C. Sphenoid dysplasia
D. Hydromyelia D. Neurofibroma
E. Diplomyelia E. Subependymal giant cell astrocytoma
CHAPTER 12 / FIGURES WITH QUESTIONS 115

Question 42

Question 43

42. The gross appearance is most typical for a(n) 43. Which tumor is most likely to present like this?
A. Infiltrating low-grade glioma A. Low-grade astrocytoma
B. Encephalitis B. Low-grade oligodendroglioma
C. Demyelinating disease C. Ependymoma
D. Picks disease D. Ganglioglioma
E. Infarct E. Pilocytic astrocytoma
116 NEUROPATHOLOGY REVIEW

Question 44

Question 45

44. The best diagnosis for this lesion in a 77-yr-old 45. This GFAP-positive filum terminale mass repre-
male is sents a(n)
A. Remote infarct A. Subependymoma
B. Demyelinating disease B. Ependymoma
C. Glioblastoma multiforme C. Myxopapillary ependymoma
D. Hemorrhage D. Astrocytoma
E. Fat emboli E. Schwannoma
CHAPTER 12 / FIGURES WITH QUESTIONS 117

Question 46

Question 47

46. This 32-yr-old female has multiple white matter 47. The infant has megalencephaly with diffuse white
lesions that look like this. Her diagnosis is matter abnormalities. The most likely diagnosis is
A. Infarct A. Alexanders disease
B. Multiple sclerosis B. Adrenoleukodystrophy
C. Anaplastic astrocytoma C. Krabbes disease
D. Vasculitis D. Metachromatic leukodystrophy
E. Storage disease E. Cockaynes disease
118 NEUROPATHOLOGY REVIEW

Question 48

48. This mass arose in the floor of the 4th ventricle 50. The region denoted by the arrow represents
and was an incidental findings at autopsy. The A. Dentate
best diagnosis is B. CA4
A. Neurocytoma C. CA3
B. Ependymoma D. Sommer sector
C. Choroid plexus papilloma E. Subiculum
D. Subependymoma
E. Meningioma
49. The most likely diagnosis in this 28-yr-old immu-
nocompetent female is
A. Multiple infarcts
B. Vasculitis
C. Astrocytoma
D. Multiple sclerosis
E. Progressive multifocal leukoencephalopathy
CHAPTER 12 / FIGURES WITH QUESTIONS 119

Question 49

Question 50
120 NEUROPATHOLOGY REVIEW

Questions 51, 52, and 53

51. The patient is an 82-yr-old female with dementia. 54. The cell denoted by the arrow represents a(n)
The changes illustrated in this figure are character- A. Neuron
istic of B. Oligodendrocyte
A. Malformation C. Ependymal cell
B. Atrophy D. Endothelial cell
C. Tumor E. Microglial cell
D. Demyelinating disease 55. The organism denoted by the smaller arrow
E. Hemorrhage represents
52. (See Figure 51) The lesion denoted by the A. CMV
smaller arrows most likely represents a(n) B. PML
A. Infarct C. Toxoplasmosis
B. Malformation D. Amebiasis
C. Multiple sclerosis E. Cysticercosis
D. Low-grade astrocytoma 56. (See Figure 55) The larger arrow denotes an
E. Microglial nodule aggregate of cells forming what structure?
53. (See Figure 51) The lesion denoted by the larger A. Granuloma
arrow is B. Durcks node
A. Germinoma C. Microglial nodule
B. Part of a malformation D. Abscess
C. Colloid cyst E. Sulfur granule
D. Pineal gland
E. Pituitary gland
CHAPTER 12 / FIGURES WITH QUESTIONS 121

Question 54

Questions 55 and 56
122 NEUROPATHOLOGY REVIEW

Question 57

57. This patient is a 62-yr-old male with a history of 60. The structures seen here represent infection from
non-Hodgkins lymphoma who developed multi- A. Cryptococcus
ple white matter lesions in both the brain and B. Candida
brain stem. The most likely diagnosis is
C. Aspergillus
A. SSPE
D. Mucormycosis
B. PML
E. Actinomyces
C. Multiple sclerosis
61. (See Figure 60) All of the following organisms
D. CMV may have hyphae which look like this, except
E. Toxoplasmosis A. Rhizopus
58. The infectious agent represented in this photo- B. Absidia
micrograph is
C. Mucor
A. Cryptococcus
D. Apophysomyces
B. Candida
E. Fusarium
C. Blastomycosis
D. CMV
E. Papova virus
59. (See Figure 58) The process shown represents
A. Abscess
B. Ventriculitis
C. Meningitis
D. Infarct
E. Encephalitis
CHAPTER 12 / FIGURES WITH QUESTIONS 123

Questions 58 and 59

Questions 60 and 61
124 NEUROPATHOLOGY REVIEW

Questions 62 and 63

62. The best diagnosis for this 48-yr-old male with 65. The structures located in the cytoplasm of the neu-
rapidly progressive dementia and myoclonus is ron marked by the arrow represent
A. Alzheimers disease A. Pick bodies
B. Picks disease B. Granulovacuolar degeneration
C. Parkinsons disease C. Neurofibrillary tangles
D. Progressive supranuclear palsy D. Hirano bodies
E. None of the above E. Lewy bodies
63. (See Figure 62) The structure denoted by the 66. (See Figure 65) The lesion depicted here is most
arrow represents a(n) likely to be encountered in the setting of
A. Neurofibrillary tangle A. Alzheimers disease
B. Neuritic plaque B. Picks disease
C. Kuru plaque C. Parkinsons disease
D. Axon torpedo D. Corticobasal degeneration
E. Granular body E. Amyotrophic lateral sclerosis
64. The strucuture denoted by the arrow represents
A. Marinesco body
B. Hyaline body
C. Rosenthal fiber
D. Corpora amylacea
E. Alzheimer type I astrocyte
CHAPTER 12 / FIGURES WITH QUESTIONS 125

Question 64

Questions 65 and 66
126 NEUROPATHOLOGY REVIEW

Question 67

Question 68

67. This 29-yr-old female most likely has 68. The organism seen here represents
A. Multiple sclerosis A. Candida
B. Glioblastoma multiforme B. Aspergillus
C. Cortical dysplasia C. Mucormycosis
D. Leukodystrophy D. Blastomycosis
E. Cavum septum pellucidum E. Cryptococcus
CHAPTER 12 / FIGURES WITH QUESTIONS 127

Question 69

Questions 70 and 71

69. This GFAP-positive intramedullary spinal cord 71. (See Figure 70) The stains most useful to high-
mass represents a(n) light these structures include
A. Astrocytoma A. Bodian and Congo red
B. Ependymoma B. GMS and PAS
C. Meningioma C. GMS and Congo red
D. Medulloblastoma D. Bodian and PAS
E. Myxopapillary ependymoma E. Ziehl-Neelson and Congo red
70. The structures seen here represent
A. Pick bodies
B. Granular bodies
C. Neuritic plaques
D. Neurofibrillary tangles
E. Hirano bodies
128 NEUROPATHOLOGY REVIEW

Question 72

Question 73

72. The mucicarmine-positive organisms seen here 73. The organism present here represents
represent A. Amebiasis
A. Candida B. Malaria
B. Cryptococcus C. Toxoplasmosis
C. Aspergillus D. Cysticercosis
D. Nocardia E. Schistosomiasis
E. Actinomyces
CHAPTER 12 / FIGURES WITH QUESTIONS 129

Question 74

Question 75 and 76

74. The organism denoted by the arrow in this menin- 76. (See Figure 75) The inclusions seen here are
gitis most likely represents caused by the JC virus. Which of the following is
A. Meningococcus a likely finding on the biopsy?
B. E. coli A. Neutrophilic infiltrate
C. M. tuberculosis B. Alzheimer type I astrocytes
D. Nocardia C. Alzheimer type II astrocytes
E. Actinomyces D. Abscess
75. The lesions denoted by the arrows represent E. Microglial nodules
A. Microglial nodules
B. Cowdry type A inclusions
C. Cowdry type B inclusions
D. Cowdry type C inclusions
E. Cytoplasmic viral inclusions
130 NEUROPATHOLOGY REVIEW

Question 77

Question 78

77. The cytoplasm of these neurons in this normal 78. The viral inclusion seen in the neuron denoted by
brain section are filled with the arrow represents what infection?
A. Neuromelanin A. Herpes simplex
B. Lipofucsin B. Cytomegalovirus
C. Hyaline bodies C. Herpes zoster
D. Marinesco bodies D. SSPE
E. Hemosiderin E. Rabies
CHAPTER 12 / FIGURES WITH QUESTIONS 131

Questions 79 and 80

79. The structure denoted by the arrow on this Bod- 80. (See Figure 79) The structure seen here has been
ian-stained section represents described in all of the following conditions
A. Ganulovacuolar degeneration except
B. Neuritic plaque A. Alzheimers disease
C. Neurofibrillary tangle B. SSPE
D. Hirano body C. Parkinsons disease
E. Pick body D. Herpes encephalitis
E. Dementia pugilistica
132 NEUROPATHOLOGY REVIEW

Questions 81, 82, and 83

81. All of the following are abnormalities evident in 84. The structure illustrated represents
this photomicrograph except A. Choroid plexus
A. Vascular congestion B. Choroid plexus papilloma
B. Herniation C. Choroid plexus carcinoma
C. Widening of the gyri D. Papillary meningioma
D. Narrowing of the sulci E. Papillary ependymoma
E. Meningeal clouding 85. The cell denoted by the arrow represents a(n)
82. (See Figure 81) All are potential complications of A. Astrocyte
the process seen here except B. Oligodendrocyte
A. Ventriculitis C. Neuron
B. Infarct D. Microglial cell
C. Hemorrhage E. Arachnoidal cap cell
D. Secondary glioma development 86. (See Figure 85) This cell is most likely to stain
E. Hydrocephalus positively with which antibody?
83. (See Figure 81) The most likely causative organ- A. GFAP
ism in a 78-yr-old individual who has not been B. Synaptophysin
recently hospitalized would be
C. Epithelial membrane antigen
A. E. coli
D. Common leukocyte antigen
B. Proteus
E. HAM 56
C. Listeria
D. S. pneumoniae
E. N. meningitidis
CHAPTER 12 / FIGURES WITH QUESTIONS 133

Question 84

Questions 85 and 86
134 NEUROPATHOLOGY REVIEW

Question 87

87. The most likely cause of hemorrhage in this 58- 89. (See Figure 88) The pattern of inheritance associ-
yr-old male is ated with this condition is likely to be
A. Hypertension A. X-linked recessive
B. Amyloid B. X-linked dominant
C. Vasculitis C. Autosomal dominant
D. Aneurysm D. Autosomal recessive
E. Oligodendroglioma E. None of the above
88. This leptomeningeal-based lesion is a common 90. The illustration most accurately represents a(n)
CNS finding in A. Tumor
A. Neurofibromatosis type I B. Embolized vascular formation
B. Neurofibromatosis type II C. Artery with vasculitis
C. von Hippel-Lindau syndrome D. Infarct
D. Tuberous sclerosis E. Aneurysm
E. Sturge-Weber disease
CHAPTER 12 / FIGURES WITH QUESTIONS 135

Questions 88 and 89

Question 90
136 NEUROPATHOLOGY REVIEW

Question 91

91. The large cell seen here in a patient with pro- 94. (See Figure 92) This tumor is most likely to arise
gressive multifocal leukoencephalopathy repre- in a(n)
sents a(n) A. 59-yr-old male, frontal lobe
A. Oligodendrocyte B. 60-yr-old female, temporal lobe
B. Neuron C. 14-yr-old male, pineal gland
C. Alzheimer type I astrocyte D. 15-yr-old female, pineal gland
D. Alzheimer type II astrocyte E. 82-yr-old male, pituitary gland
E. Cowdry type A inclusion 95. The lesion seen here represents a(n)
92. The lesion denoted by the arrows represents a(n) A. Arteriovenous malformation
A. Microglial nodule B. Cavernous angioma
B. Abscess C. Venous angioma
C. Secondary structure of Sherer D. Varix
D. Granuloma E. Capillary telangiectasia
E. Vascular proliferation
93. (See Figure 92) The lesion in this case is associ-
ated with what tumor?
A. Lymphoma
B. Germinoma
C. Meningioma
D. Metastatic carcinoma
E. Embryonal carcinoma
CHAPTER 12 / FIGURES WITH QUESTIONS 137

Questions 92, 93, and 94

Question 95
138 NEUROPATHOLOGY REVIEW

Questions 96, 97, and 98

96. The elongated structures (arrow) seen here rep- 99. The organism seen here represents
resent A. Amebiasis
A. Abnormal mitochondria B. Cysticercosis
B. Cilia C. Schistosomiasis
C. Blepharoplasts D. Trypanosomiasis
D. Melanosomes E. Toxoplasmosis
E. Birbeck granules 100. (See Figure 99) The organism causing this lesion
97. (See Figure 96) The most likely diagnosis is is
A. Ependymoma A. Taenia solium
B. Mitochondrial myopathy B. Trypanosoma brucei
C. Melanoma C. Entamoeba histolytica
D. Histiocytosis X D. Trichonella spiralis
E. Erdheim-Chester disease E. Iodamoeba buetschlii
98. (See Figure 96) The cells that contain these struc- 101. The pathology illustrated here represents
tures characteristically stain with antibody A. Oligodendroglioma
A. GFAP B. Astrocytoma
B. HMB-45 C. Ependymoma
C. CD3 D. Reactive astrocytosis
D. CD1a E. Lymphoma
E. HAM 56
CHAPTER 12 / FIGURES WITH QUESTIONS 139

Questions 99 and 100

Question 101
140 NEUROPATHOLOGY REVIEW

Questions 102 and 103

102. The structure denoted by the arrow represents 105. The area seen here is consistent with a(n)
A. Secondary structure of Scherer A. Caudate nucleus
B. Vascular proliferation B. Hippocampus
C. Necrosis C. Dentate nucleus
D. Vascular invasion D. Olivary nucleus
E. None of the above E. None of the above
103. (See Figure 102) The lesion seen here in a 76-yr- 106. (See figure 105) The fascicles of myelinated
old male represents a(n) fibers seen here (arrow) are referred to as
A. WHO grade I astrocytoma A. Lines of Baillarger
B. WHO grade II astrocytoma B. Stria of Gennari
C. WHO grade III astrocytoma C. Bundles of Wilson
D. WHO grade IV astrocytoma D. Polymorphous bundles
E. Ependymoma E. None of the above
104. The photomicrograph here represents what nor-
mal structure?
A. Pineal gland
B. Leptomeninges
C. Pituitary adenohypophysis
D. Pituitary neurohypophysis
E. Pituitary pars intermedia
CHAPTER 12 / FIGURES WITH QUESTIONS 141

Question 104

Questions 105 and 106


142 NEUROPATHOLOGY REVIEW

Questions 107

Question 108

107. The age of the patient here most likely is 108. The photomicrograph here was most likely taken
from which location?
A. 3 mo
A. Frontal lobe cortex
B. 3 yr
B. Frontal lobe white matter
C. 13 yr
C. Caudate
D. 30 yr
D. Hippocampus
E. 60 yr
E. Pineal gland
CHAPTER 12 / FIGURES WITH QUESTIONS 143

Question 109

Question 110

109. The elastic stained section here represents a(n) 110. The melanin pigmented neurons seen here belong
to which neuronal group?
A. Arteriovenous malformation
A. Dentate nucleus
B. Cavernous angioma
B. Substantia nigra
C. Venous angioma
C. Hypoglossal nucleus
D. Hemangioblastoma
D. Red nucleus
E. Capillary telangiectasia
E. Olivary nucleus
144 NEUROPATHOLOGY REVIEW

Question 111

111. What normal structure is represented here? 113. This 38-yr-old male has
A. Pineal gland A. Non-Hodgkins lymphoma
B. Arachnoidal cap cells B. Malaria
C. Choroid plexus C. Medulloblastoma
D. Skeletal muscle D. Amebiasis
E. None of the above E. Toxoplasmosis
112. The mass seen here represents a(n)
A. Astrocytoma
B. Infarct
C. Abscess
D. Granuloma
E. Demyelinating plaque
CHAPTER 12 / FIGURES WITH QUESTIONS 145

Question 112

Question 113
146 NEUROPATHOLOGY REVIEW

Questions 114 and 115

114. The organism seen here by electron microscopy 117. The findings in this lesion represent a(n)
represents A. Contusion
A. Cytomegalovirus B. Acute infarct
B. Toxoplasmosis C. Subacute infarct
C. Cryptococcus D. Remote infarct
D. Progressive multifocal leukoencephalopathy E. Tumor
E. E. coli 118. (See Figure 117) The most likely age of this
115. (See Figure 114) In an immunocompromised lesion is
adult, the most common histologic manifestation A. 24 hr
of an infection by this organism is B. 48 hr
A. Granuloma C. 72 hr
B. Leptomeningitis D. 57 d
C. Abscess E. 3 mo
D. Demyelination
E. Microglial nodule
116. This luxol-fast blue stained section of periventric-
ular tissue shows areas of decreased white matter
staining in a 28-yr-old female. Cells that may be
seen in these regions include all of the following
except
A. Reactive astrocytes
B. Macrophages
C. Neutrophils
D. Lymphocytes
E. Microglial cells
CHAPTER 12 / FIGURES WITH QUESTIONS 147

Question 116

Questions 117 and 118


148 NEUROPATHOLOGY REVIEW

Questions 119 and 120

119. The cell shown here represents a(n) 122. (See Figure 121) The changes seen in the large
neurons is the result of
A. Neuron
A. Ischemia
B. Astrocyte
B. Central chromatolysis
C. Microglial cell
C. Lipofuscin
D. Oligodendroglial cell
D. Neuronophagia
E. Endothelial cell
E. Ferruginization
120. (See Figure 119) The process shown in this cell
represents 123. The accumulation of GM2 ganglioside material in
these cells is characteristic of
A. Ischemic changes
A. Gauchers disease
B. Central chromatolysis
B. Tay-Sachs disease
C. Ferruginization
C. Ceroid-lipofuscinosis
D. Neuronophagia
D. Hunters disease
E. Creutzfeldt astrocyte
E. Leighs disease
121. The section seen here is located where?
124. (See Figure 123) The cells in which the material
A. Frontal lobe cortex
has accumulated are
B. Thalamus
A. Astrocytes
C. Pons
B. Ependymal cells
D. Midbrain
C. Microglial cells
E. Cerebellum
D. Neurons
E. Oligodendrocytes
CHAPTER 12 / FIGURES WITH QUESTIONS 149

Questions 121 and 122

Questions 123 and 124


150 NEUROPATHOLOGY REVIEW

Question 125

125. The structure most likely to be encountered in 127. (See figure 126) All of the following conditions
these neurons in a patient with Parkinsons dis- are likely to result in the pathology seen here
ease is except
A. Neuritic plaque A. Chronic ischemic change
B. Hirano body B. Chronic epilepsy
C. Lewy body C. Alcoholism
D. Neurofibrillary tangle D. Infiltrating astrocytoma
E. Granulovacuolar degeneration E. Dilantin toxicity
126. The paler staining cells denoted by the arrow 128. The patient is a 12-yr-old female with a recent
represent measles infection. She now presents with seizures
and dystonia. The diagnosis on biopsy is
A. Purkinje cells
A. Oligodendroglioma
B. Granular cells
B. Progressive multifocal leukoencephalopathy
C. Lymphocytes
C. Subacute sclerosing panencephalitis
D. Bergmann astrocytes
D. Lymphoma
E. Corpora amylacea
E. None of the above
CHAPTER 12 / FIGURES WITH QUESTIONS 151

Questions 126 and 127

Question 128
152 NEUROPATHOLOGY REVIEW

Question 129

129. The meningioma seen here is best classified as 132. (See Figure 130) The pattern of inheritance associ-
ated with this condition is
A. Syncytial
A. X-linked recessive
B. Fibrous
B. Autosomal dominant
C. Invasive
C. Autosomal recessive
D. Metaplastic
D. Sporadic
E. Rhabdoid
E. X-linked dominant
130. This infant has diffuse white matter abnormalities
on imaging studies. The diagnosis is 133. The changes seen here are most likely related to
A. Alexanders disease A. Fat emboli
B. Metachromatic leukodystrophy B. Methanol ingestion
C. Globoid cell leukodystrophy C. Kernicterus
D. Adrenoleukodystrophy D. Ethanol toxicity
E. Canavans disease E. Arsenic toxicity
131. (See Figure 130) The lesion seen is associated
with a deficiency of
A. Arylsulfatase A
B. Galactocerebroside-B galactosidase
C. Aspartoacylase
D. Phospholipase
E. None of the above
CHAPTER 12 / FIGURES WITH QUESTIONS 153

Questions 130, 131, and 132

Question 133
154 NEUROPATHOLOGY REVIEW

Questions 134, 135, and 136

134. The lesion denoted by the arrow is a(n) 137. The lesion seen here most likely represents a(n)
A. Arteriovenous malformation A. Vascular malformation
B. Air embolism B. Contusion
C. Aneurysm C. Infarct
D. Venous angioma D. Air embolism
E. Moyamoya disease E. Aneurysm
135. (See Figure 134) Microscopically, the defect that 138. The cell denoted by the arrow represents a(n)
defines this lesion is A. Reactive astrocyte
A. A mixture of arterial and venous vessels B. Gemistocyte
B. Intimal fibroplasia C. Creutzfeldt astrocyte
C. Atherosclerosis D. Alzheimer type I astrocyte
D. Loss of arterial media E. Alzheimer type II astrocyte
E. Fragmented elastic lamina of the vessel wall 139. (See Figure 138) This cell is most likely to be
136. (See Figure 134) The other gross pathologic encountered in the setting of
abnormality evident in this picture is A. Tumor
A. Infarct B. Demylinating disease
B. Ganglioglioma C. Diabetic encephalopathy
C. Dandy-Walker syndrome D. Hepatic encephalopathy
D. Subdural hematoma E. Progressive multifocal leukoencephalopathy
E. Fat emboli
CHAPTER 12 / FIGURES WITH QUESTIONS 155

Question 137

Questions 138 and 139


156 NEUROPATHOLOGY REVIEW

Question 140

140. The darker areas have a yellow coloration in this 142. The lesion seen here represents a(n)
autopsy specimen from a 2-wk-old baby with a A. Vascular malformation
history of hemolytic anemia. The lesions seen
B. Acute infarct
represent
C. Remote infarct
A. Multifocal infarcts
D. Multiple sclerosis plaque
B. Hemorrhage
E. Lacunar infarct
C. Bilirubin encephalopathy
D. Fetal alcoholism syndrome
E. Ponto-subicular necrosis
141. The changes seen here are the result of
A. Atrophy
B. Edema
C. Polymicrogyria
D. Subarachnoid hemorrhage
E. Lissencephaly
CHAPTER 12 / FIGURES WITH QUESTIONS 157

Question 141

Question 142
158 NEUROPATHOLOGY REVIEW

Questions 143 and 144

143. The most accurate diagnosis here is 146. (See Figure 145) The gene defect associated with
A. Contusion this lesion is located at chromosome
B. Epidural hematoma A. It is not associated with a gene defect
C. Subdural hematoma B. 22
D. Arteriovenous malformation C. 19
E. Meningitis D. 14
144. (See Figure 143) The lesion seen here is gener- E. 3
ally associated with the rupture of which vessel? 147. The lesion shown here in this 62-yr-old female
A. Middle meningeal artery represents a(n)
B. Bridging veins A. Demyelinative plaque
C. Vertebral artery B. Laminar infarct
D. Sagittal sinus C. Pseudolaminar infarct
E. Middle cerebral artery D. Abscess
145. The changes seen here are most consistent with E. Creutzfeldt-Jakob disease
A. Atherosclerotic vasculopathy
B. Moyamoya syndrome
C. Binswangers disease
D. CADASIL
E. Amyloid angiopathy
CHAPTER 12 / FIGURES WITH QUESTIONS 159

Questions 145 and 146

Question 147
160 NEUROPATHOLOGY REVIEW

Question 148

Question 149

148. The changes seen here are most likely related to 149. The changes seen in this vessel are most likely
the result of
A. Brain stem glioma
A. Lymphoma
B. Syringobulbia
B. Polyarteritis nodosa
C. Degeneration of corticospinal tracts
C. Granulomatous angiitis
D. Chiari malformation
D. Hypersensitivity angiitis
E. Olivopontocerebellar atrophy
E. Amyloid angiopathy
CHAPTER 12 / FIGURES WITH QUESTIONS 161

Question 150

Question 151

150. The organisms seen here represent 151. The leptomeningeal organism denoted by the
arrow represents
A. Trichinosis
A. Schistosomiasis
B. Cysticercosis
B. Malaria
C. Trypanosomiasis
C. Cysticercosis
D. Sarcocystis
D. Echinococcus
E. Toxoplasmosis
E. Strongyloides
162 NEUROPATHOLOGY REVIEW

Questions 152, 153, and 154

152. The lesion seen here represents a(n) 154. (See Figure 152) The lesion seen here may be
A. Granuloma encountered in all of the following conditions
except
B. Microglial nodule
A. Listeriosis
C. Hamartoma
B. Sarcoidosis
D. Thrombus
C. Tuberculosis
E. Vascular proliferation
D. Wegeners vasculitis
153. (See Figure 152) Stains that may be useful in fur-
ther delineating the etiology of this lesion would E. Germinoma
include
A. GMS and PAS
B. GMS and fite
C. GFAP and GMS
D. GFAP and fite
E. GFAP and Ziehl-Neelson
CHAPTER 12 / FIGURES WITH QUESTIONS 163

Questions 155 and 156 Question 157

155. The midbrain lesion seen here represents 157. The abnormality shown in this cortical section
A. Vascular malformation from a 14-yr-old with chronic epilepsy represents
B. Duret hemorrhage A. No abnormality
C. Diffuse axonal injury B. Cortical dysplasia
D. Fat emboli C. Ganglioglioma
E. Lacunar infarct D. Dysembryoplastic neuroepithelial tumor
156. (See Figure 155) The midbrain lesion is often E. Pleomorphic xanthoastrocytoma
seen associated with what other condition?
A. Uncal herniation
B. Central herniation
C. Subfalcial herniation
D. Tonsillar herniation
E. Upward herniation
164 NEUROPATHOLOGY REVIEW

Question 158

158. This partial lobectomy specimen is from a 12-yr- 160. The findings seen here are consistent with
old with tuberous sclerosis. The diagnosis is A. Infarct
A. Subependymal giant-cell astrocytoma B. Mesial temporal sclerosis
B. Cortical tuber C. Anaplastic astrocytoma
C. Metastatic rhabdomyoma D. Carbon monoxide toxicity
D. Lymphoma E. Wernicke encephalopathy
E. Hemangioblastoma 161. (See Figure 160) The underlying cause of the
159. The findings in this myelin-stained section of spi- pathology seen here is
nal cord are most consistent with A. Poor cerebral perfusion
A. Thiamine deficiency B. Not known
B. Wilsons disease C. Vitamin deficiency
C. Menkes disease D. Possibly suicide related
D. Subacute combined degeneration E. A risk from prior radiation therapy
E. Herpes myelitis
CHAPTER 12 / FIGURES WITH QUESTIONS 165

Question 159

Questions 160 and 161


166 NEUROPATHOLOGY REVIEW

Question 162

162. The changes in the hippocampus of this 8-yr-old 164. The GFAP immunostain seen here is highlighting
with intractable seizures is the result of what cells?
A. Hippocampal sclerosis A. Ependymal cells
B. Cortical dysplasia B. Astrocytes
C. Ganglioglioma C. Microglial cells
D. Cortical tuber D. Oligodendroglial cells
E. Neuronal heterotopia E. Neurons
163. The changes seen in the leptomeninges in this
68-yr-old male with bacterial meningitis represent
A. Granuloma formation
B. Abscess
C. Vasculitis with thrombosis
D. Infarct
E. Hemorrhage
CHAPTER 12 / FIGURES WITH QUESTIONS 167

Question 163

Question 164
168 NEUROPATHOLOGY REVIEW

Question 165

Question 166

165. The findings here are most likely related to 166. The pathology seen here in this 48-yr-old male
A. Infarct with endocarditis and atrial fibrillation most likely
represents
B. Ethanol
A. Demyelinating disease
C. Carbon monoxide
B. Embolic acute infarcts
D. Aluminum toxicity
C. Air embolism
E. Manganese toxicity
D. Multiple organizing abscesses
E. Storage disease
CHAPTER 12 / FIGURES WITH QUESTIONS 169

Questions 167, 168, and 169

167. The structure denoted by the arrow represents a 169. (See Figure 167) All of the following are likely
A. Muscle fascicle pathologic findings in this patient except
B. Neuromuscular junction A. Scattered type I hypertrophy
C. Muscle spindle B. Loss of neurons in Onufs nucleus
D. Blood vessel C. Increased muscle spindles
E. Sarcomeric unit D. Atrophy of anterior spinal roots
168. (See Figure 167) This biopsy is from a 3-mo-old E. Fascicular atrophy of the muscle
with hypotonia and tongue fasciculations. The
diagnosis is
A. Werdnig-Hoffmann disease
B. Central core disease
C. Duchenne dystrophy
D. Centronuclear myopathy
E. Pompes diease
170 NEUROPATHOLOGY REVIEW

Question 170

Question 171

170. The vessel seen here from a vastus lateralis mus- 171. The changes in the biopsy are most consistent
cle biopsy shows changes diagnostic of with a diagnosis of
A. Necrotizing vasculitis A. Polymyositis
B. Inclusion body myositis B. Dermatomyositis
C. Trichinosis C. Neurogenic atrophy
D. Dermatomyositis D. Vasculitis
E. Polymyositis E. Acid maltase deficiency
CHAPTER 12 / FIGURES WITH QUESTIONS 171

Questions 172 and 173

172. The pathologic findings on the ATPase pH 4.6 173. (See Figure 172) All of the following are associ-
stain are suggestive of ated with this disease process except
A. Dermatomyositis A. Arthralgia
B. Neurogenic atrophy B. Calcinosis
C. Muscular dystrophy C. Steroid resistance
D. Inclusion body myositis D. Malignancy
E. Spinal muscular atrophy E. Non-necrotizing vasculitis
172 NEUROPATHOLOGY REVIEW

Questions 174 and 175

174. The structures observed in the myofiber denoted 177. (See Figure 176) This change is a common fea-
by the arrow on this Gomori trichrome stain ture observed in
represent A. Polymyositis
A. A freeze artifact B. Neurogenic atrophy
B. Glycogen storage material C. Myotonic dystrophy
C. Rimmed vacuoles D. Mitochondrial myopathy
D. Nemaline rods E. Chloroquine myopathy
E. Viral inclusions 178. On this ATPase-stained section, the dark staining
175. (See Figure 174) The presence of this finding in fibers are
the setting of an inflammatory myopathic biopsy A. Type I
is highly suggestive of
B. Type IIA
A. Angiopathic polymyositis
C. Type IIB
B. Viral myositis
D. Type IIC
C. Fungal myositis
E. Type III
D. Inclusion body myositis
E. Myasthenia gravis
176. The NADH-stained section shown reveals
A. Ring fibers
B. Moth-eaten fibers
C. Target fibers
D. Targetoid fibers
E. Core fibers
CHAPTER 12 / FIGURES WITH QUESTIONS 173

Questions 176 and 177

Question 178
174 NEUROPATHOLOGY REVIEW

Question 179

179. The positive staining on this alkaline phosphatase 181. (See Figure 180) The structure denoted by the
stain represents arrow represents a(n)
A. Degenerating fibers A. I band
B. Regenerating fibers B. A band
C. Acutely denervated fibers C. H band
D. Ragged red fibers D. M band
E. A mitochondrial defect E. Z band
180. The fibrillary structures observed within the 182. The vacuolar structures seen here likely represent
nucleus of this myocyte are characteristic of what A. Glycogen storage disease
entity? B. Lipid storage disease
A. Mitochondrial myopathy C. Mitochondrial degeneration
B. Inclusion body myositis D. Core formations
C. Glycogen storage disease E. A freeze artifact
D. Amyloid
E. A parasite
CHAPTER 12 / FIGURES WITH QUESTIONS 175

Questions 180 and 181

Question 182
176 NEUROPATHOLOGY REVIEW

Questions 183 and 184

183. The abnormality seen ultrastructurally represents 185. The absence of staining on this cytochrome oxi-
an abnormality of dase stain is indicative of a possible
A. T system A. Glycogen storage disease
B. Mitochondria B. Lipid storage disease
C. Sarcoplasmic reticulum C. Viral infection
D. Ribosomes D. Mitochondrial myopathy
E. Golgi E. None of the above
184. (See Figure 183) The abnormality seen here is 186. The curvilinear bodies seen here ultrastructurally
most commonly associated with are most commonly associated with
A. Amyloid deposition A. Mitochondrial encephalomyopathy
B. Ragged red fibers B. Ceroid lipofuscinosis
C. Autophagic vacuoles C. Lafora body disease
D. Core fibers D. Globoid cell leukodystrophy
E. Nemaline rods E. Metachromatic leukodystrophy
CHAPTER 12 / FIGURES WITH QUESTIONS 177

Question 185

Question 186
178 NEUROPATHOLOGY REVIEW

Questions 187 and 188

187. The cystalloid structures denoted by the small 190. (See Figure 189) The most common mutation
arrow represent associated with the hereditary form of this condi-
tion involves what gene?
A. Renaut bodies
A. Thyroglobulin
B. Myelin
B. Notch 3
C. Schmidt-Lanterman clefts
C. Transthyretin
D. Pi granules of Reich
D. Notch 5
E. Corpuscles of Erzholz
E. None of the above
188. (See Figure 187) The structure denoted on the
left by the larger arrow is a(n) 191. The arrow corresponds to a(n)
A. Schwann cell A. Schwann cell nucleus
B. Collagen B. Endothelial cell
C. Unmyelinated axon C. Fibroblast
D. Myelinated axon D. Mast cell
E. Renaut body E. Renaut body
189. The finely fibrillar material (arrow) in the wall of 192. (See Figure 191) The pathology seen here is most
this epineurial vessel stains positively with Congo likely to be encountered in the setting of
red. The best diagnosis is A. Traumatic neuroma
A. Vasculitic neuropathy B. Amyloid neuropathy
B. CADASIL C. Vasculitic neuropathy
C. Amyloid neuropathy D. Charcot-Marie-Tooth disease
D. Adrenoleukodystrophy E. Thiamine deficiency neuropathy
E. Porphyria
CHAPTER 12 / FIGURES WITH QUESTIONS 179

Questions 189 and 190

Questions 191 and 192


180 NEUROPATHOLOGY REVIEW

Question 193

193. The changes seen in this teased nerve preparation 195. This muscle is from an 8-yr-old male who is
most likely would be seen in wheelchair bound. The most likely diagnosis is
A. Vasculitic neuropathy A. Polymyositis
B. Charcot-Marie-Tooth disease B. Dermatomyostitis
C. Guillain-Barre syndrome C. Duchenne muscular dystrophy
D. CIDP D. Myotonic muscular dystrophy
E. Diphtheritic neuropathy E. Becker muscular dystrophy
194. The change seen here represents 196. (See Figure 195) Confirmation of the diagnosis in
this case would involve
A. Axonal sprouting
A. T-cell subset analysis
B. Demyelination
B. Serology testing for ANA
C. Remyelination
C. Sedimentation rate
D. Onion bulb formation
D. Dystrophin analysis
E. Renaut body
E. Emerin analysis
CHAPTER 12 / FIGURES WITH QUESTIONS 181

Question 194

Questions 195 and 196


182 NEUROPATHOLOGY REVIEW

Questions 197 and 198

197. The lesions seen on this NADH stain represent 199. The pattern of atrophy observed here is sugges-
tive of
A. Moth-eaten fibers
A. Dermatomyositis
B. Rimmed vacuoles
B. Spinal muscular atrophy
C. Nemaline rods
C. Inclusion body myositis
D. Target fibers
D. Steroid myopathy
E. Targetoid fibers
E. Diabetic neuropathy-associated neurogenic
198. (See Figure 197) This biopsy is most likely to be
atrophy
seen in the setting of
200. The vacuoles seen here stain with acid phospha-
A. Myasthenia gravis
tase and are filled with PAS-positive material.
B. Lambert-Eaton syndrome The diagnosis is
C. Werdnig-Hoffman disease A. Carnitine deficiency
D. Neurogenic atrophy B. Myophosphorylase deficiency
E. Polymyositis C. Acid maltase deficiency
D. Succinate dehydrogenase deficiency
E. Phosphofructokinase deficiency
CHAPTER 12 / FIGURES WITH QUESTIONS 183

Question 199

Question 200
13 Answers to Figures with Questions

1. A (Chapter 5.III.A) The plexiform neurofibroma 12. A (Chapter 5.III.A) Meningoangiomatosis is


is almost exclusively seen in the setting of neuro- characterized by a perivascular proliferation of
fibromatosis type I. meningothelial cells and is associated with neuro-
fibromatosis type I.
2. E (Chapter 3.IV.A) Dural-based meningioma,
the most commonly encountered cytogenetic 13. C (Chapter 3.X.G) Squamous cell carcinoma
abnormality is a deletion associated with chromo- with large keratinized epithelial-appearing cells
some 22. infiltrating dura.
14. C (Chapter 3.II.E) The location, appearance, and
3. C (Chapter 3.I.F) Perinecrotic pseudopalisading
immunohistochemical profile are classic for cen-
in a glioblastoma multiforme.
tral neurocytoma.
4. B(chapter 3 I L) Subependymal giant-cell astro-
15. B (Chapter 3.VII.A) The nuclear palisading
cytoma arising in the setting of tuberous sclerosis.
(Verocay bodies) and mixed compact (Antoni A)
5. A (Chapter 1.I.B) Rosenthal fibers in a pilocytic and loose regions (Antoni B) are characteristic of
astrocytoma. a schwannoma.
6. D (Chapter 3.VIII.A) Angiotropic large cell 16. A (Chapter 3.IV.B) The presence of vacuolated
lymphomas frequently are CD20 positive (B-cell epithelioid cells (physaliphorous) in a bone-based
immunophenotype) and are typically confined to sacrococcygeal mass is suggestive of a chordoma.
vascular lumina. 17. B (Chapter 3.VI.C) The structure denoted by the
7. C (Chapter 3.X.C) The mixture of large germ arrow is a Homer Wright pseudorosette in a
cells and lymphocytes is characteristic of medulloblastoma.
germinoma. 18. C (Chapter 3.VI.C) The cells seen are from a
8. D (Chapter 3.VII.D) The haphazard proliferation medulloblastoma.
of peripheral neural tissue with collagen in the 19. B (Chapter 4.II.H) The lesion shown here (pore-
arm is best designated as traumatic neuroma. ncephaly) is usually the result of an ischemic/
9. A (Chapter 5.I.D) The absence of convolution infarct event.
seen in this brain is characteristic of lissencephaly 20. E (Chapter 7.I.B) The zebra bodies seen by elec-
(agyria). tron microscopy are a common findings in Tay-
10. C (Chapter 3.IV.A) The tumor shown is marked Sachs disease.
by cells arranged in a whorled configuration, char- 21. B (Chapter 5.I.F) The lesion seen represents a
acteristic of a syncytial or meningotheliomatous partial agenesis of the corpus callosum.
meningioma. 22. D (Chapter 3.III.B) The well-demarcated appear-
11. D (Chapter 1.I.G) A meningioma with numer- ance of this intramedullary spinal cord mass is
ous laminated calcifications known as psammoma more commonly seen with ependymoma versus
bodies. an astrocytoma.

185
186 NEUROPATHOLOGY REVIEW

23. C (Chapter 5.III.C) The subependymal giant-cell 39. B (Chapter 5.I.B) The changes seen are most
astrocytoma seen here is associated with tuberous characteristic of a Chiari malformation.
sclerosis. 40. D (Chapter 5.I.A) Congenital dilatation of the
24. A (Chapter 3.X.A) The nested architectural pat- central canal is termed hydromyelia.
tern (zellballen), chromogranin positivity, and 41. E (Chapter 5.III.A) The axillary freckling and
location are classic for paraganglioma. cafe-au-lait macule are features of neurofibroma-
25. A (Chapter 3.II.A) The most common site of ori- tosis type I. All the listed lesions may be seen in
gin for oligodendroglioma is frontal lobe (it has this setting except subependymal giant cell astro-
the most white matter). cytoma (a feature of tuberous sclerosis).
26. D (Chapter 3.I.N) The lesion depicted here 42. A (Chapter 3.I.B) Obliteration of the graywhite
(asymmetrial enlargement of the brain stem) is junction is a gross feature of infiltrating gliomas
most consistent with a brain stem glioma. (in this case astrocytoma).
27. B (Chapter 3.X.D) Adamantinomatous pattern 43. B (Chapter 3.II.A) About 2% of oligodendroglio-
of a craniopharyngioma. mas present with evidence of hemorrhage.
28. A (Chapter 3.IX.A) The location and appear- 44. C (Chapter 3.I.F) The gross appearance is most
ance of the mass is most consistent with a colloid consistent with a glioblastoma multiforme.
cyst of the third ventricle.
45. C (Chapter 3.III.E) The lesion depicted repre-
29. D (Chapter 3.V.A) The lateral ventricle is the sents a myxopapillary ependymoma.
most likely location of choroid plexus papillomas
arising in childhood. 46. B (Chapter 6.I.D) Multiple white matter lesions
marked by reactive astrocytes, macrophages, and
30. C (Chapter 3.X.G) The sharp interface between perivascular lymphocytes are suggestive of multi-
tumor and brain parenchyma and immunohisto- ple sclerosis.
chemical profile is most consistent with mela-
noma (most glioblastoma multiforme show cross- 47. A (Chapter 6.II.D) Numerous Rosenthal fibers
immunoreactivity with cytokeratin AE1/3). in the setting of diffuse demyelination and mega-
lencephaly are suggestive of Alexanders disease.
31. E (Chapter 5.III.B) Bilateral acoustic schwanno-
mas are diagnostic of neurofibromatosis type II, 48. D (Chapter 3.III.D) Subependymomas are
which is related to a chromosome 22 abnor- marked by loose clustering of cell nuclei arranged
mality. against a fibrillary background. Microcystic
change is fairly common.
32. A (Chapter 3.X.E) The most common secreting
pituitary adenoma is a prolactinoma. 49. D (Chapter 6.I.D) Periventricular white matter
plaques are a characteristic feature of multiple
33. B (Chapter 5.I.D) Polymicrogyria, seen in the
sclerosis.
figure, is associated with Zellwegers syndrome,
a peroxisome abnormality. 50. D (Chapter 1.I.H) The Sommer sector (CA1)
region is denoted by the arrow.
34. A (Chapter 5.II.I) The germinal matrix hemor-
rhage is most likely to occur in the premature 51. B (Chapter 8.III.C) The changes seen are charac-
infant. teristic of atrophyventricular dilatation, narrow-
ing of gyri, widening of sulci. This patient had
35. E (Chapter 5.I.F) The defect seen represents
Alzheimers disease.
cavum septi pellucidi.
36. B (Chapter 3.III.B) The cilia and ciliary body 52. A (Chapter 2.II.C) The partially cavitated lesion
attachments are characteristic ultrastructural fea- denoted by the small arrow represents a remote
tures of ependymoma. infarct.
37. A (Chapter 3.I.M) The histologic appearance 53. D (Chapter 1.I.H) The large arrow is pointing at
(pleomorphic astrocytic cells), clinical presenta- a normal appearing pineal gland.
tion, and low rate of cell proliferation are most 54. E (Chapter 1.I.E) The cell characterized by an
consistent with a pleomorphic xanthoastrocytoma. elongated nucleus and scant cytoplasm represents
38. C (Chapter 3.I.H) The gliosarcoma consists of a microglial cell.
reticulin-rich sarcomatous areas and GFAP-posi- 55. C (Chapter 9.V.A) The smaller arrow is point-
tive glioblastomatous areas. ing at a Toxoplasma gondii cyst.
CHAPTER 13 / ANSWERS TO FIGURES WITH QUESTIONS 187

56. C (Chapter 9.V.A) Rarely, microglial nodules, 73. A (Chapter 9.V.B) The organism seen repre-
as seen here, are associated with Toxoplasmosis sents Amebiasis in a background of necrosis.
infection. 74. A (Chapter 9.II.C) The cocci seen here most
57. B (Chapter 9.IV.M) Multiple white matter likely represent Meningococcus.
lesions in an immunocompromised individual 75. B (Chapter 9.IV.A) The large intranuclear viral
should raise the possibility of PML as a diag- inclusions seen here represent Cowdry type A
nosis. inclusions.
58. D (Chapter 9.IV.L) The Cowdry A nuclear inclu- 76. B (Chapter 9.IV.M) Alzheimer type I astrocytes
sions seen here are most characteristic of CMV are a common finding in progressive multifocal
infection. leukoencephaly (result of JC virus infection)
59. B (Chapter 9.IV.L) The viral inclusions are asso- 77. B (Chapter 1.I.A) The intracytoplasmic material
ciated with the ependymal lining present and ven- in many of the neurons here represents lipofuscin
tricular space representing a ventriculitis. or aging pigment.
60. D (Chapter 9.III.E) The broad-branching, non- 78. E (Chapter 9 IV I ) The intraneuronal cytoplas-
septate hyphae seen here are characteristic of mic Negri body is characteristic of rabies.
Mucormycosis.
79. C (Chapter 8.III.E) The neuronal cytoplasmic sil-
61. E (Chapter 9.III.E) Fusarium hyphae are mor- ver positive structure represents a neurofibrillary
phologically similar to Aspergillus in tissue sec- tangle.
tions.
80. D (Chapter 8.III.E) Neurofibrillary tangles are
62. E (Chapter 9.VI.B) The clinical history and not found in Herpes encephalitis.
pathology (cortical spongiform degeneration with
neuronal loss) are characteristic of Creutzfeldt- 81. B (Chapter 9.II.C) Herniation is not evident in
Jakob disease. this case. The brain is edematous and demon-
strates vascular congestion and meningeal cloudi-
63. C (Chapter 9.VI.B) The star-burst-like structure
ness consistent with a purulent meningitis.
represents a kuru plaque.
82. D (Chapter 9.II.C) Secondary glioma develo-
64. D (Chapter 1.I.B) The structure represents cor-
ment is not associated with purulent meningitis.
pora amylacea, a laminated basophilic polygluco-
san body associated with astrocytic foot pro- 83. D (Chapter 9.II.C) Streptococcus pneumoniae is
cesses. the most likely cause of community acquired bac-
terial meningitis in the elderly.
65. B (Chapter 8.III.G) The granules with a clear
halo represent granulovacuolar degeneration. 84. A (Chapter 1.I.F) The structure shown repre-
sents a normal choroid plexus, marked by fibro-
66. A (Chapter 8.III.G) Granulovacuolar degenera-
vascular cores lined by epithelioid cells.
tion is most likely to be seen in the setting of Alz-
heimers disease. 85. C (Chapter 1.I.A) The cell shown represents a
67. A (Chapter 6.I.D) The periventricular areas of neuron.
demyelination (plaques) is characteristic of multi- 86. B (Chapter 1.I.A) Neurons are most likely to
ple sclerosis. stain with antibody to synaptophysin.
68. B (Chapter 9.III.D) Dichotomously branching 87. A (Chapter 2.V.A) Hypertension is the most
septate hyphae is characteristic of Aspergillus. common cause of nontraumatic intracerebral hem-
69. B (Chapter 3.III.B) The presence of true ependy- orrhage.
mal rosettes and perivascular pseudorosettes in 88. E (Chapter 5.III.D) Leptomeningeal venous
this tumor is characteristic of an ependymoma. angioma is a common finding in Sturge-Weber
70. C (Chapter 8.III.F) The structures seen here rep- disease.
resent neuritic plaques. 89. E (Chapter 5.III.D) There is no pattern of inheri-
71. A (Chapter 8.III.F) A Bodian stain will high- tance associated with Sturge-Weber disease.
light the neuritic component of the plaque and a 90. B (Chapter 2.IV.B) The lesion illustrated repre-
Congo red will stain the amyloid core. sents an arteriovenous malformation that has been
72. B (Chapter 9.III.B) The thick mucopolysaccha- embolized.
ride wall of Cryptococcus stains with muci- 91. C (Chapter 9.IV.M) Large atypical astrocytes,
carmine. Alzheimer type I astrocytes, are commonly
188 NEUROPATHOLOGY REVIEW

encountered in the setting of progressive multifo- 109. A (Chapter 2.IV.B) The presence of arterial and
cal leukoencephalopathy. venous vessels with intervening neural paren-
92. D (Chapter 3.X.C) The collection of epithelioid chyma is characteristic of an arteriovenous
histocytes represents a non-necrotizing gran- angioma.
uloma. 110. B (Chapter 1.I.A) Melanin-pigmented neurons
93. B (Chapter 3.X.C) Granulomas may be seen in are seen in substantia nigra, locus ceruleus, and
germinomas. The large cells seen here represent the dorsal motor nucleus of the vagus nerve.
malignant germ cells. 111. E (Chapter 1.I.H) The nests of epithelioid cells
94. C (Chapter 3.X.C) Germinomas are most likely separated by fibrovascular septae is characteristic
to arise in young males in the pineal gland region. of the pituitary adenohypophysis.
95. B (Chapter 2.IV.D) The sinusoidal-type vessels 112. C (Chapter 9.II.D) The central necrosis rimmed
arranged in a back-to-back fashion is characteris- by a collagenous wall is characteristic of an
tic of a cavernous angioma. organizing abscess.
96. E (Chapter 3.VIII.B) The elongated tennis- 113. B (Chapter 9.V.C) The granules of darkly pig-
racket-like structures seen here represent Birbeck mented material within red cells in blood vessel
granules. lumina is characteristic of malaria.
97. D (Chapter 3.VIII.B) Birbeck granules are a dis- 114. A (Chapter 9.IV.L) The organisms seen here
tinctive feature of Langerhans cell histiocytes of have the characteristic ultrastructural appearance
histiocytosis X. of cytomegalovirus.
98. D (Chapter 3.VIII.B) Langerhans cell histio-
115. E (Chapter 9.IV.L) In an immunocompromised
cytes stain with CD1a antibody.
adult, cytomegalovirus infection most commonly
99. B (Chapter 9.V.D) The lesion seen here repre- manifests as microglial nodules.
sents cysticercosis.
116. C (Chapter 6.I.D) Neutrophils are not a salient
100. A (Chapter 9.V.D) The organism responsible feature microscopically of multiple sclerosis.
for cysticerosis is Taenia solium (the pork
tapeworm). 117. D (Chapter 2.II.C,D) Cavitary changes in the
cortex with surrounding gliosis and a relative spar-
101. D (Chapter 3.I.D) The mildly increased cellular-
ing of the molecular layer is consistent with a
ity and larger cells with abundant eosinophilic
remote infarct.
cytoplasm are characteristic of reactive astro-
cytosis. 118. E (Chapter 2.II.C,D) Remote infarct findings
102. B (Chapter 3.I.F) The lesion denoted here repre- are seen in association with the 3 mo of age
sents glomeruloid vascular proliferation. parameter.
103. D (Chapter 3.I.A) The prominent vascular prolif- 119. A (Chapter 1.I.A) The cell shown here repre-
eration in this astrocytoma is sufficient enough to sents a neuron.
warrant a diagnosis of WHO grade IV astro- 120. B (Chapter 1.I.A) Cytoplasmic swelling with
cytoma. loss and peripheralization of Nissl substance and
104. D (Chapter 1.I.H) The loose arrangement of eccentric nucleus are salient features of central
spindled cells is characteristic of the pituitary neu- chromatolysis.
rohypophysis. 121. E (Chapter 1.I.H) The section is from the cere-
105. A (Chapter 1.I.H) The area seen here may repre- bellum with the molecular layer to the left, and
sent either the caudate or putamen. two large Purkinje cells and smaller round granu-
106. C (Chapter 1.I.H) The fascicles of myelinated lar cells to the right.
fibers seen in the caudate are known as the pencil 122. A (Chapter 1.I.A) The Purkinje cells are marked
bundles of Wilson. by ischemic changesshrunken cell bodies with
107. A (Chapter 1.I.H) The presence of the external dark-staining nuclei and indistinguishable
granular cell layer of the cerebellum is a finding nucleoli.
most consistent with an age of 3 mo. 123. B (Chapter 7.I.B) The accumulation of GM2
108. B (Chapter 1.I.H) The photomicrograph repre- ganglioside material is a feature of Tay-Sachs
sents white matter parenchyma. disease.
CHAPTER 13 / ANSWERS TO FIGURES WITH QUESTIONS 189

124. D (Chapter 7.I.B) The distended cells repre- 141. B (Chapter 4.I.B) The changes seen (expansion
sented neurons filled with GM2 ganglioside of gyri and narrowing of sulci) are characteristic
material. of cerebral edema.
125. C (Chapter 8.IV.C) Lewy bodies are often 142. C (Chapter 2.II.C) The cavitary defect repre-
encountered in substantia nigra (neuromelanin-pig- sents a remote infarct.
mented) neurons. 143. C (Chapter 4.II.B) The hemorrhage seen here
126. D (Chapter 1.I.B) The proliferation of cells in represents a subdural hematoma.
the region of the Purkinje cell layer are the 144. B (Chapter 4.II.B) Subdural hematomas are asso-
Bergmann astrocytes. ciated with tearing of bridging veins which trans-
127. D (Chapter 1.I.B) Loss of Purkinje cells with verse through the subarachnoid space.
astrocytosis is associated with all of the listed con- 145. D (Chapter 2.VII.B) The thickened vessel
ditions except infiltrating astrocytoma. walls with granularity is highly suggestive of
128. C (Chapter 9.IV.G) The antecedent measles CADASIL.
infection with Cowdry A inclusions is typical of 146. C (Chapter 2.VII.B) The notch 3 gene responsi-
SSPE (subacute sclerosing panencephalitis) ble for CADASIL is located on chromosome
129. C (Chapter 3.IV.B) This meningioma is brain 19q12.
invasive, a feature some associate with malig- 147. C (Chapter 2.II.D) The lesion seen here repre-
nancy. sents an acute infarct surrounded by a zone of
130. C (Chapter 6.II.B) Collections of white matter edema in a pseudolaminar configuration (involv-
globoid cells is characteristic of Krabbes disease ing more than one cortical layer).
or globoid cell leukodystrophy. 148. C (Chapter 2.II.D) The asymetrical loss of corti-
131. B (Chapter 6.II.B) Krabbes disease is the result cospinal tracts is likely related to an ipsilateral
of a deficiency in galactocerebroside-B galacto- cerebral infarct.
sidase. 149. C (Chapter 2.VI.A) The changes seen are those
of a granulomatous vasculitis.
132. C (Chapter 6.II.B) Krabbes disease is inherited
as an autosomal recessive condition. 150. D (Chapter 10.III.G) The organisms seen in the
skeletal muscle represents Sarcocystis infection.
133. B (Chapter 7.II.B) The bilateral basal ganglia
hemorrhagic necrosis is a feature of methanol 151. E (Chapter 9.V.G) The elongated organism rep-
toxicity. resents Strongyloides in a patient who had dissem-
inated disease.
134. C (Chapter 2.III.A) The lesion denoted by the
arrow represents a saccular aneurysm arising 152. A (Chapter 9.II.F) The collection of epithelioid
from the posterior inferior cerebellar artery histiocytes and lymphocytes represents a gran-
(PICA). uloma.
153. B (Chapter 9.II.F) The best combination of
135. D (Chapter 2.III.A) Loss of the media defines
stains to further evaluate this lesion would be a
the saccular aneurysm.
GMS (for fungal organisms) and fite stain (for a
136. A (Chapter 2.III.C) There is evidence of friable, mycobacterial organism).
necrotic tissue consistent with infarct involving
154. A (Chapter 9.II.C) Listeria infection typically
the cerebellum.
presents with a suppurative leptomeningitis with
137. A (Chapter 2.IV.C) The lesion shown here repre- abscess formation, usually not accompanied by
sents part of a vascular malformation (venous granulomas. All of the other conditions may be
angioma). associated with granuloma formation.
138. E (Chapter 1.I.B) The nuclear swelling and chro- 155. B (Chapter 4.I.D) The midline brain stem hemor-
matin clearing is characteristic of an Alzheimer rhage represents a Duret hemorrhage.
type II astrocyte. 156. A (Chapter 4.I.D) The Duret hemorrhage is typi-
139. D (Chapter 1.I.B) Alzheimer type II astrocytes cally associated with uncal or lateral transtentorial
are most likely encountered in hepatic encephalop- herniation.
athy (elevated ammonia levels). 157. B (Chapter 5.I.C) The disorganized cortical
140. C (Chapter 5.II.O) The changes seen are charac- architecture is a common pattern of cortical dys-
teristic of bilirubin encephalopathy (kernicterus). plasia.
190 NEUROPATHOLOGY REVIEW

158. B (Chapter 5.III.C) The enlarged gyrus repre- 175. D (Chapter 10.III.C) Rimmed vacuoles in the
sents a cortical tuber. setting of an inflammatory myopathy are highly
159. D (Chapter 7.I.T) The findings seen here are suggestive of inclusion body myositis.
most consistent with those of subacute combined 176. A (Chapter 10.VIII.F) Ring fibers are marked
degeneration due to vitamin B12 deficiency. by a ring of myofibrils that is oriented circumfer-
entially around the longitudinally oriented fibrils
160. E (Chapter 7.I.S) Discoloration in the mamillary
in the rest of the fiber.
bodies with gliosis due to hemorrhage/hemosid-
erin and loss of neurons is seen in the setting of 177. C (Chapter 10.VIII.F) Ring fibers are a rela-
Wernicke encephalopathy. tively common finding in patients with myotonic
dystrophy.
161. C (Chapter 7.I.S) Thiamine deficiency underlies
the pathologic changes seen with Wernicke 178. A (Chapter 10.I.D) The dark fibers are type I on
encephalopathy. ATPase pH 4.6.
162. A (Chapter 5.I.J) Focal neuronal loss and gliosis 179. B (Chapter 10.I.D) Regenerating myofibers are
in the hippocampus (especially CA1, CA4, and highlighted on the alkaline phosphatase stain.
dentate regions) characterizes hippocampal 180. B (Chapter 10.III.C) The tubulofilaments seen
sclerosis. within this myofiber nucleus are characteristic of
163. C (Chapter 9.II.C) The leptomeningeal vessel those seen in inclusion body myositis.
shows evidence of vasculitis and organizing 181. E (Chapter 1.II.G) The dark band marked by the
thrombus. arrow represents the Z band.
164. B (Chapter 1.I.B) The GFAP immunostain is 182. E (Chapter 10.I.C) The holes seen in all of the
highlighting astrocytic cells in the cortex. myofibers represent a freeze artifact sustained dur-
ing processing of the biopsy specimen.
165. B (Chapter 7.II.C) The cerebellar vermal atro-
phy seen here is most likely related to ethanol. 183. B (Chapter 10.VI.A) The paracrystalline inclu-
sions are observed in mitochondria.
166. B (Chapter 2.II.C) The areas of darker discolor-
ation represent multifocal acute infarcts, most 184. B (Chapter 10.VI.A) Ragged red fibers is a light
likely embolic in origin. microscopic finding also associated with mito-
chondrial myopathies.
167. C (Chapter 1.II.I) The lesion denoted by the
185. D (Chapter 10.I.D) The partial cytochrome oxi-
arrow represents a muscle spindle (intrafusal
dase deficiency may be indicative of a mitochon-
fibers enclosed by a connective tissue capsule).
drial abnormality.
168. A (Chapter 10.IV.B and Chapter 8.IV.N) The
186. B (Chapter 7.I.F) Ultrastructural evidence of cur-
rounded myofiber atrophy in this infant is charac-
vilinear bodies supports a diagnosis of ceroid lipo-
teristic of Werdnig-Hoffman disease.
fucinosis.
169. B (Chapter 10.IV.B and Chapter 8.IV.N) There 187. D (Chapter 1.III.F) The cystalloid structures
is often sparing of Onufs nucleus in Werdnig- within the Schwann cell cytoplasm are pi gran-
Hoffman disease. ules of Reich.
170. A (Chapter 10.III.I) The changes seen in the ves- 188. C (Chapter 1.III.E) The structure denoted by the
sel are those of a necrotizing vasculitis (polyarteri- larger arrow is an unmyelinated axon.
tis nodosa).
189. C (Chapter 11.III.I) The congophilic material
171. A (Chapter 10.III.A) Chronic endomysial represents amyloid deposition.
inflammation in the absence of perifascicular atro-
190. C (Chapter 11.III.I) Mutations in the transthy-
phy is consistent with polymyositis.
retin or prealbumin gene are associated with
172. A (Chapter 10.III.B) The perifascicular pattern hereditary forms of amyloid neuropathy.
of injury is consistent with dermatomyositis.
191. A (Chapter 1.III.F) The cell nucleus is from a
173. C (Chapter 10.III.B) Dermatomyositis is usually Schwann cell.
responsive to steroid therapy. 192. D (Chapter 11.III.F) The onion bulb structure is
174. C (Chapter 10.III.C) The vacuoles seen repre- most commonly associated with Charcot-Marie-
sent rimmed vacuoles or autophagic vacuoles. Tooth disease.
CHAPTER 13 / ANSWERS TO FIGURES WITH QUESTIONS 191

193. A (Chapter 11.III.I) The fragmentation of 197. D (Chapter 10.IV.A) The three zones of staining
myelin is consistent with axonal degeneration, a (central pale, dark intermediate, and normal
prominent feature of vasculitic neuropathy. peripheral) is characteristic of target fibers.
194. A (Chapter 11.II.A) The changes here are consis- 198. D (Chapter 10.IV.A) Target fibers are a com-
tent with axonal regeneration (sprouting). mon feature of neurogenic atrophy.
195. C (Chapter 10.VIII.C) The marked variation in 199. B (Chapter 1.II.D) Fascicular atrophy is associ-
fiber size and fibrosis are most suggestive of a ated with spinal muscular atrophy.
dystrophic process. The clinical history favors
Duchenne muscular dystrophy. 200. C (Chapter 10.VI.B) The lysosomal associated
196. D (Chapter 10.VIII.C) Dystrophin analysis (acid phosphatase positive) acid maltase defi-
would be most useful in confirming the diagnosis ciency is associated with glycogen (PAS positive)
accumulation in vacuoles.
of Duchenne muscular dystrophy.
14 Written Self-Assessment Questions

1. Nissl substance is composed of 5. Meckel-Gruber syndrome is associated with all of


the following except
A. Smooth endoplasmic reticulum
A. Autosomal dominant pattern of inheritance
B. Rough endoplasmic reticulum
B. Occipital encephalocele
C. Golgi structures
C. Polydactyly
D. Neuromelanin
D. Polycystic kidneys
E. Lipofuscin
E. Hepatic fibrosis
2. Anoxic damage related to carbon monoxide
poisoning would be best classified as 6. Which HLA type is associated with the develop-
ment of multiple sclerosis?
A. Anoxic anoxia
A. HLA B7
B. Anemic anoxia
B. HLA D8
C. Histotoxic anoxia
C. HLA DR15
D. Stagnant anoxia
D. HLA DW6
E. None of the above
E. HLA DR4
3. What WHO grade is pleomorphic xanthoastro-
7. All of the following are normal aging changes
cytoma?
in the brain except
A. Grade I A. Loss of brain weight
B. Grade II B. Leptomeningeal thickening
C. Grade III C. Increased corpora amylacea
D. Grde IV D. Rare neurofibrillary tangles
E. Does not have WHO grade E. Decreased astrocyte number and size
4. Cerebral edema is marked by all of the following 8. The most common organism responsible for an
except intracranial epidural abscess is
A. Increased brain weight A. E. coli
B. Narrowing of sulci B. Staphylococcus
C. Decreased ventricular size C. Streptococcus
D. Narrowing of gyri D. Listeria
E. Prominent pacchionian granulations E. Gram-negative anaerobes

193
194 NEUROPATHOLOGY REVIEW

9. The alkaline phosphatase stain is useful for identi- 15. The most common site of origin for low-grade
fying fibrillary astrocytoma is
A. Degenerating myofibers A. Frontal lobe
B. Regenerating myofibers B. Parietal lobe
C. Acutely denervated fibers C. Temporal lobe
D. Mitochondrial abnormalities D. Occipital lobe
E. Amyloid deposition E. Spinal cord
10. Congenital dilatation of the central canal is 16. Cerebral edema as a result of an impaired cell
known as Na-K membrane pump is referred to as
A. Syrinx A. Vasogenic
B. Syringobulbia B. Cytotoxic
C. Hydromyelia C. Tumoral
D. Diplomyelia D. Transient
E. Myeloschisis E. None of the above
11. Histologically, all of the following are features of 17. Area cerebrovasculosa is associated with
multiple sclerosis except A. Anencephaly
A. Neutrophils B. Meningocele
B. Lymphocytes C. Chiari malformation
C. Reactive astrocytes D. Edwards syndrome
D. Macrophages E. Schizencephaly
E. Myelin loss 18. Acute-onset multiple sclerosis with prominent
12. All of the following features are characteristic of optic nerve and spinal cord involvement repre-
neurogenic muscle disease except sents which subtype?
A. Angular atrophic fibers A. Marburg type
B. Target fibers B. Balo type
C. Group atrophy C. Schilder type
D. Fiber type grouping D. Devic type
E. Ring fibers E. Concentric sclerosis type
13. Marinesco bodies are most likely going to be 19. Phosphorylated tau protein is associated with the
found in which cell type? development of
A. Neuron A. Neurofibrillary tangles
B. Astrocyte B. Neuritic plaques
C. Oligodendrocyte C. Amyloid angiopathy
D. Ependymal cell D. Granulovacuolar degeneration
E. Arachnoidal cap cell E. Hirano bodies
14. The most senstive neurons to anoxic damage are 20. All of the following are potential complications
in what region? of bacterial leptomeningitis except
A. Endplate of hippocampus A. Infarct
B. Sommer sector B. Hemorrhage
C. Cortex layer IV C. Demyelination
D. Thalamus D. Vasculitis
E. Pons E. Hydrocephalus
CHAPTER 14 / WRITTEN SELF-ASSESSMENT QUESTIONS 195

21. Which of the following is the most likely cause 27. All of the following are features of gliosis as
of meningitis in a 2-wk-old infant? opposed to diffuse astrocytoma except
A. Staph. epidermidis A. Rosenthal fibers
B. Staph. aureus B. Collagen deposition
C. Strep. agalactiae C. Microcystic change
D. Strep. pneumoniae D. Absent calcification
E. Listeria E. Reactive astrocytes
28. Kernohans notch is associated with which of the
22. Cyclopia is most commonly associated with
following?
A. Downs syndrome A. Central herniation
B. Holoprosencephaly B. Subfalcial herniation
C. Chiari malformation, type II C. Uncal herniation
D. Dandy-Walker syndrome D. Tonsillar herniation
E. None of the above E. Upward herniation
23. Myofibers with basophilic cytoplasm, nuclear 29. All of the following are commonly associated
enlargement, and prominent nucleation are charac- with the Chiari type I malformation except
teristic of A. Cerebellar tonsillar herniation
A. Targetoid fibers B. Adult presentation
B. Degenerating fibers C. Syringomyelia
C. Regenerating fibers D. Klippel-Feil anomaly
D. Nemaline rods E. Arachnoidal adhesions
E. Moth-eaten fibers 30. Waterhouse-Friderichsen syndrome is most typi-
cally associated with which organism?
24. All are characteristic of an axonal degenerative
process except A. Group B Strep
A. Axonal sprouting B. E. coli
C. Listeria
B. Myelin ovoids
D. N. meningitidis
C. Bands of Bungner
E. H. influenza
D. Macrophages
31. All of the following conditions are associated
E. Shortened internodal distances with granulomatous inflammation in the CNS
25. Bergmann astrocytes are found in the except
A. Basal ganglia A. Tuberculosis
B. Cerebral cortex B. Listeriosis
C. Subcortical white matter C. Sarcoidosis
D. Cerebellum D. Foreign body giant-cell reaction
E. Coccidioidomycosis
E. Hippocampus
32. All of the following are features of acute hemor-
26. Hypoglycemic neuronal necrosis preferentially
rhagic leukoencephalitis except
affects all of the following areas except
A. Cerebral edema
A. Cortexlayers II and III
B. Vessel wall necrosis
B. CA1 region of hippocampus
C. White matter hemorrhages
C. Dentate
D. Immunocompromised state
D. Caudate E. About half of cases present with antecedent
E. Purkinje cells upper respiratory infection
196 NEUROPATHOLOGY REVIEW

33. Neurofibrillary cytoskeletal alterations are a fea- 39. All of the following are genetic alterations associ-
ture of which toxicity? ated with diffuse astrocytoma except
A. Carbon monoxide A. p53 mutations
B. Aluminum B. Increased PDGFR alpha expression
C. Arsenic C. Gain 7q
D. Trisomy 22
D. Lead
E. Loss of heterozygosity 10p
E. Manganese
40. Which vessel may get compressed in lateral trans-
34. -Amyloid is encoded for on chromosome tentorial herniation resulting in infarct of the cal-
A. 1 carine cortex?
B. 14 A. Basilar artery
C. 19 B. Posterior cerebral
D. 21 C. Posterior inferior cerebellar artery
E. 22 D. Anterior inferior cerebellar artery
E. None of the above
35. All of the following are features of polymyositis
except 41. Hypoplasia of the cerebellar vermis with cystic
dilatation of the 4th ventricle and agenesis of the
A. May be associated with rheumatoid arthritis corpus callosum would be most accurately termed
B. CPK often elevated A. Chiari type II malformation
C. Endomysial chronic inflammation B. Dandy-Walker syndrome
D. Autophagic vacuoles C. Cerebellar dysplasia
E. More common in females D. Holotelencephaly
36. The most distinguishing feature pathologically of E. Schizencephaly
dermatomyositis is 42. Which of the following stains is most useful in
A. Chronic inflammation highlighting mycobacterial organisms?
B. Myofiber degeneration A. Hematoxylin and eosin
B. PAS
C. Myofiber regeneration
C. GMS
D. Perifascicular atrophy
D. Ziehl-Neelsen
E. Fascicular atrophy
E. Dieterle
37. Rosenthal fibers may be encountered in all of the 43. Which of the following is true regarding meta-
following conditions except chromatic leukodystrophy?
A. Alexanders disease A. Chromosome 21q gene defect
B. Reactive astrocytosis B. Autosomal dominant
C. Fibrillary astrocytoma C. Arcuate U fibers not spared
D. Pilocytic astrocytoma D. White matter with macrophages filled with
glucocerebrosides
E. Pineal gland cyst
E. Arylsulfatase A deficiency
38. Cerebral blood flow constitutes approximately
what percentage of cardiac output? 44. CERAD criteria for the diagnosis of Alzheimers
disease rely on which pathologic feature?
A. 5%
A. Neurofibrillary tangles
B. 10% B. Neuritic plaques
C. 15% C. Amyloid angiopathy
D. 20% D. Lewy bodies
E. 30% E. None of the above
CHAPTER 14 / WRITTEN SELF-ASSESSMENT QUESTIONS 197

45. All are features of dermatomyositis except 51. The most common glioma type in the CNS is
A. Steroid refractory A. Oligodendroglioma
B. Heliotropic rash B. Ependymoma
C. Gottrons papules C. Glioblastoma multiforme
D. Soft tissue calcinosis D. Low-grade diffuse astrocytoma
E. Association with malignancy E. Pilocytic astrocytoma
46. Miller-Dieker syndrome is associated with agyria 52. Rupture of the middle meningeal artery is associ-
related to deletion on chromosome 17 involving ated with
which gene?
A. Duret hemorrhage
A. LIS-1
B. Epidural hemorrhage
B. p53
C. Subdural hemorrhage
C. N-myc
D. Subarachnoid hemorrhage
D. Retinoblastoma gene
E. Parenchymal hemorrhage
E. None of the above
53. Cortical dysplasia is associated with all of the fol-
47. All of the following are common features of men- lowing except
ingovascular syphilitic CNS infection except
A. Tuberous sclerosis
A. Prominent plasma cell infiltrates
B. Dysembryoplastic neuroepithelial tumor
B. Gummata
C. Neurofibromatosis type I
C. Heubners endarteritis obliterans
D. Neurofibromatosis type II
D. Arteritis
E. Epidermal nevus syndrome
E. Demyelination
54. All are true regarding globoid cell leukodystro-
48. All are true regarding or seen with inclusion body
phy except
myositis except
A. Deficiency of galactocerebrosidase
A. Steroid resistent
B. Autosomal recessive
B. Caused by viral infection
C. Onset of symptoms in first year of life
C. Rimmed vacuoles
D. Tubulofilamentous inclusions ultrastructurally D. Sparing of arcuate subcortical fibers

E. Mitochondrial abnormalities E. Atrophy of brain


49. Alzheimers II astrocytes are most commonly 55. All of the following are pathologic features of
associated with what condition? Alzheimers disease except
A. Hepatic encephalopathy A. Granulovacuolar degeneration
B. Progressive multifocal leukoencephalopathy B. Hirano bodies
C. Gliosis C. Bunina bodies
D. Pilocytic astrocytoma D. Lewy bodies
E. Alexanders disease E. Amyloid angiopathy
50. Macrophages may be first microscopically evident 56. Which fungus is a budding yeast with a thick
in an infarct at what time? mucopolysaccharide wall?
A. Within 12 h A. Cryptococcus
B. 24 h B. Candida
C. 48 h C. Aspergillus
D. 1 wk D. Mucormycosis
E. 34 wk E. Coccidioidomycosis
198 NEUROPATHOLOGY REVIEW

57. The diabetic patient with a periorbital infection 63. All are features of metastatic carcinoma except
involving organisms with nonseptate hyphae is A. Discrete gross lesion
most likely infected with
B. Collagenous stroma
A. Aspergillus
C. Discrete cell borders
B. Candida
D. Cytokeratin immunoreactivity
C. Mucormycosis
E. Perinecrotic pseudopalisading
D. Blastomycosis
64. An acute subdural hematoma is associated with a
E. Actinomyces
skull fracture (in)
58. Lissencephaly results from defects in develop-
A. Never
ment that occur during the
B. 25% of cases
A. 5th7th wk of gestation
C. 50% of cases
B. 8th10th wk of gestation
C. 11th13th wk of gestation D. 75% of cases

D. 15th17th wk of gestation E. Almost always

E. 20th22nd wk of gestation 65. Agyria with hydrocephalus, retinal dysplasia,


encephalocele, no muscle disease, and death
59. A protoplasmic body is a salient feature of which in infancy is most characteristic of
of the following?
A. Cerebro-ocular dysplasia
A. Alzheimers disease
B. Neu-Laxova syndrome
B. Demyelinating disease
C. Zellwegers syndrome
C. Fungal infection
D. Miller-Dieker syndrome
D. Congenital myopathy
E. Walker-Warburg syndrome
E. Tumor
66. Most forms of adrenoleukodystrophy are related
60. An infarct of what minimum volume of brain tis-
to an abnormality on which chromosome?
sue will almost invariably result in dementia
A. 1
A. 10 mL
B. 22
B. 25 mL
C. 19
C. 50 mL
D. 80 mL D. 6

E. 120 mL E. None of the above


61. The fried egg appearance of oligodendrocytes 67. Target fibers are most closely associated with
represents which process?
A. Cell cytoplasm A. Polymyositis
B. Perinuclear edema B. Dermatomyositis
C. Artifact of delayed fixation C. Myotonic dystrophy
D. Increased cytoplasmic lipid D. Neurogenic atrophy
E. None of the above E. Mitochondrial myopathy
62. The least common site for a lacunar infarct is the 68. Diphtheritic neuropathy is best classified as being
A. Putamen A. The result of Schwann cell dysfunction
B. Caudate B. The result of myelin sheath damage
C. Pons C. The result of vessel damage
D. Mamillary bodies D. The result of axonal damage
E. Thalamus E. None of the above
CHAPTER 14 / WRITTEN SELF-ASSESSMENT QUESTIONS 199

69. Neurofibrillary tangles are best highlighted on 75. Eosinophilic granular bodies are a common
which stain? feature of
A. PAS A. Glioblastoma multiforme
B. Trichrome B. Pilocytic astrocytoma
C. Cytochrome oxidase C. Subependymal giant-cell astrocytoma
D. Bodian D. Nasal glioma
E. Congo red E. None of the above
70. The Huntington gene is located on which chro- 76. Traumatic basilar subarachnoid hemorrhage is
mosome? related to the tear of what vessel?
A. 1 A. Basilar artery
B. 4 B. Vertebral artery
C. 14 C. Subclavian artery
D. 19 D. Aorta
E. 21 E. External carotid artery
71. Infection with which organism is associated with 77. A brain weight of 2350 g is properly termed
sulfur granule formation? A. Megalocephaly
A. Coccidioidomycosis B. Schizencephaly
B. Blastomycosis C. Microencephaly
C. Actinomycosis D. Megaloencephaly
D. Nocardiosis E. Normal
E. None of the above 78. Other organ systems that frequently contain inclu-
sions in adenoleukodystrophy include all of the
72. Cowdry type A inclusions are typical of all the
following except
following organisms except
A. Testis
A. Polio virus
B. Peripheral nerve
B. CMV
C. Liver
C. Epstein-Barr virus
D. Lymph nodes
D. Herpes virus
E. Pancreas
E. JC virus
79. All of the following pathologies are typical of a
73. Ciliary body attachments in ependymal cells are CNS viral infection except
known as
A. Granulomas
A. Kinetoplasts
B. Microglial nodules
B. Chloroplasts
C. Neuronophagia
C. Microtubuloblasts
D. Chronic inflammation
D. Blepharoplasts
E. Central chromatolysis
E. None of the above
80. All of the following are true regarding subacute
74. What percentage of saccular aneurysms are sclerosing panencephalitis (SSPE) except
bilateral? A. Most present <15 yr of age
A. < 5% B. Most recover without serious sequelae
B. 10% C. The result of measles virus infection
C. 20% D. Causes demyelination
D. 50% E. Associated with neurofibrillary tangle
E. 85% formation
200 NEUROPATHOLOGY REVIEW

81. Fascicular muscle atrophy is most commonly asso- 87. All are features of the pleomorphic xanthoastro-
ciated with cytoma except
A. Myasthenia gravis A. Temporal lobe location
B. Kugelberg-Welander disease B. Cyst with enhancing mural nodule radiographi-
cally
C. Amyotrophic lateral sclerosis
C. Reticulin rich
D. Polymyositis
D. Nuclear pleomorphism
E. Mitochondrial myopathy
E. Increased mitotic activity
82. Which of the following is a trinucleotide repeat
88. Langerhans cell histocytosis is characteristically
disorder?
marked by which antibody?
A. Progressive supranuclear palsy A. CD1a
B. Diffuse Lewy body disease B. CD1b
C. Corticobasal degeneration C. CD1c
D. Hallervorden-Spatz disease D. CD4
E. Huntingtons chorea E. CD8
83. The most common site of a Mortons neuroma is 89. Raccoon eyes are associated with
the A. Hinge fracture
A. Median nerve B. Basilar skull fracture
B. Peroneal nerve C. Diastatic fracture
C. Radial nerve D. Frontal depressed skull fracture
D. Intermetatarsal nerves E. None of the above
E. Sural nerve 90. All of the following are characteristic of Downs
syndrome except
84. All are true regarding Negri bodies except
A. Reduced frontal lobe
A. Seen in Rabies
B. Narrowing of the inferior temporal gyrus
B. Intraneuronal
C. Small cerebellum
C. Intranuclear
D. Poor myelination
D. Eosinophilic E. Alzheimer-like pathology in middle-aged
E. Hippocamal neurons frequently involved adults
85. Which of the following immunostains is a useful 91. Which of the following is a characteristic feature
marker for microglial cells? of Alexanders disease?
A. GFAP A. Megalencephaly
B. Synaptophysin B. Arcuate subcortical fibers spared
C. HAM56 C. Eosinophilic granular bodies
D. S-100 protein D. Presents typically in the 2nd decade of life
E. Phospholipid protein defect
E. CLA
92. A grossly normal brain with gliosis in the caudate
86. All of the following are conditions associated and putamen translates into what Vonsattel grade
with saccular aneurysms except for Huntingtons chorea?
A. Tuberous sclerosis A. Grade 0
B. Polycystic kidney disease B. Grade 1
C. Coarctation of the aorta C. Grade 2
D. Marfans syndrome D. Grade 3
E. Ehler-Danlos syndrome E. Grade 4
CHAPTER 14 / WRITTEN SELF-ASSESSMENT QUESTIONS 201

93. The spread of which organism from the periphery 99. All of the following are true regarding ganglio-
to the central nervous system occurs primarily via glioma except
retrograde axoplasmic transport? A. Most commonly arise in temporal lobe
A. Poliovirus B. Associated with cortical dysplasia
B. Arbovirus C. Associated with chronic epilepsy
C. CMV D. Frequently contain small necrotic foci that do
D. Herpes virus not alter its good prognosis
E. PML E. Calcifications common
94. Hippocampal sclerosis is characterized by marked 100. A zellballen architectural pattern microscopically
loss of neurons in which location? characterizes which tumor?
A. Caudate A. Schwannoma
B. Neurofibroma
B. Amygdala
C. Ependymoma
C. Sommer sector
D. Paraganglioma
D. CA2
E. Chordoma
E. None of the above
101. The wounding capacity of a gunshot wound is
95. Circulating antibodies to the acetylcholine recep- most related to
tor are seen in
A. Size of bullet
A. McArdles disease
B. Shape of bullet
B. MELAS syndrome
C. Velocity of bullet
C. Myasthenia gravis D. Weight of bullet
D. Lambert-Eaton syndrome E. Size of gun
E. None of the above 102. Focal coagulative necrosis adjacent to the lateral
96. All are true regarding Guillian-Barre syndrome ventricles in a 30-wk-old fetus is best termed
except A. Ulegyria
A. Associated with HLA B27 B. Multilocular cystic encephalomalacia
B. Ascending paralysis C. Periventricular leukomalacia
C. Decreased deep tendon reflexes D. Diffuse lobar sclerosis
D. Endoneurial chronic inflammation E. Multiple sclerosis
E. Segmental demyelination 103. Congenital CMV infection is characterized by all
97. Layer III of the cerebral cortex is known as the the following except
A. Chorioretinitis
A. External granular layer
B. Gliosis
B. Internal granular layer
C. Hydrocephalus
C. Polymorphic layer
D. Megalencephaly
D. Outer pyramidal layer
E. Microcephaly
E. Inner pyramidal layer
104. All of the following characterize Lambert-Eaton
98. Dolichoectasia is associated with the development syndrome except
of which of the following?
A. Proximal muscle weakness
A. Fusiform aneurysm
B. Decreased neurotransmission with repeated
B. Mycotic aneurysm stimulation
C. Saccular aneurysm C. Autonomic dysfunction
D. Berry aneurysm D. Associated with small cell carcinoma of lung
E. Charcot aneurysm E. Antibodies to presynaptic calcium channels
202 NEUROPATHOLOGY REVIEW

105. The motor oil appearance of fluid in a cystic 111. Chemoresponsiveness in anaplastic oligodendrogli-
craniopharyngioma is the result of omas has been linked to deletions on which chro-
A. Cholesterol mosome?
B. Keratin A. 1
B. 3
C. Necrosis
C. 11
D. Calcification
D. 17
E. Glycogen
E. 22
106. Leprosy-associated neuropathy may be marked by
112. Static-loading blunt head injury is the result of
all of the following except
forces applied gradually to the head, such as
A. Granulomas with a
B. Schwann cell invasion by organisms A. Car rolling over a head
C. Perineurial fibrosis B. Hammer blow to head
D. Symmetric polyneuropathy C. Gunshot to head
E. Toxin-mediated response D. Trauma sustained in a landslide
107. Autosomal dominant transmission of Parkinsons E. Pedestrian hit by a car
disease is linked with what gene? 113. All of the following are common sites of bili-
A. Tau protein gene rubin deposition in kernicterus except
A. Dentate
B. Agitonian
B. Inferior olive
C. Ubiquitin
C. Cranial nerve nucleus VIII
D. -Synuclein
D. Claustrum
E. SOD1
E. Substantia nigra
108. Nemaline rods are composed of
114. Canavans disease is the result of a deficiency in
A. -Actinin A. Arylsulfatase B
B. Dystrophin B. Lipophosphorylase
C. Emerin C. Aspartoacylase
D. Sarcoplasmic reticulum D. Proteolipid protein
E. None of the above E. Not known
109. The pencil bundles of Wilson are a feature of 115. All of the following are common features of the
which structure? congenital myopathies except
A. Substantia nigra A. Disorders of myofiber maturation/reorgani-
zation
B. Dentate
B. Often fatal
C. Putamen
C. Retarded motor development
D. Locus ceruleus
D. Associated with skeletal dysmorphism
E. Hippocampus
E. Many show familial tendencies
110. The most common organism associated with the 116. All of the following are true regarding Lyme-
development of mycotic aneurysms is
associated neuropathy except
A. Mucor A. Caused by Borrelia burgdorferi
B. Aspergillus B. Acquired from a mosquito bite
C. Candida C. May be associated with arthritis
D. Cryptococcus D. Perivascular chronic inflammation
E. Blastomycosis E. Axonal degeneration
CHAPTER 14 / WRITTEN SELF-ASSESSMENT QUESTIONS 203

117. For those hard-to-identify Lewy bodies, which of 123. All are true regarding central neurocytoma except
the following stains would be most useful? A. Presents usually in young adults
A. PAS B. Most patients die within 2 yr of diagnosis
B. Congo red C. Synaptophysin positive
C. Ubiquitin D. Intraventricular location
D. GFAP E. Dense secretory core granules ultrastructurally
E. Gomori methenamine silver 124. A burst lobe represents an example of a
118. Which of the following is true regarding progres- A. Skull fracture
sive multifocal leukoencephalopathy?
B. Gunshot wound
A. Caused by RNA virus
C. Coup contusion
B. Most patients present <age 12 yr D. Contrecoup lesion
C. Alzheimers II astrocytes E. Gliding contusion
D. Gray matter microglial nodules 125. The neurofibromin gene is located on chro-
E. Oligodendroglial cell inclusions mosome
119. Vacuolar myelopathy is a feature of A. 1
A. Syphilis B. 22
B. HIV C. 3
C. HTLV-I D. 9
D. PML E. 17
E. None of the above 126. The salient pathologic features of Pelizaeus-
Merzbacher disease include all of the following
120. Ragged red fibers may be seen in all the follow-
except
ing except
A. Perivascular demyelination
A. MELAS
B. Brain atrophy
B. MERFF
C. Astrocytosis
C. Kearns-Sayre syndrome
D. Prominent cerebellar and brain stem
D. Inclusion body myositis involvement
E. Central core myopathy E. White matter macrophages
121. The endplate of the hippocampus (Ammons 127. Which of the following is not true regarding
horn) is referred to as HTLV-I infection?
A. Dentate A. Caused by retrovirus
B. CA1 B. Causes tropical spastic paraparesis
C. CA2 C. Myelin loss in spinal cord
D. CA3 D. Cowdry A viral inclusions
E. CA4 E. Demyelination of optic nerve
122. Which vascular malformation is characterized by 128. Paraneoplastic neuropathy is most commonly asso-
back-to-back vessels? ciated with which tumor type?
A. AVM A. Breast carcinoma
B. Venous angioma B. Lung carcinoma
C. Cavernous angioma C. Melanoma
D. Capillary telangiectasia D. Lymphoma
E. Varix E. Renal cell carcinoma
204 NEUROPATHOLOGY REVIEW

129. Where should one look for Lewy bodies? 135. The most important feature in predicting the out-
A. Neuronal nucleus come in ependymomas is
B. Neuronal cytoplasm A. Mitosis counts
B. Presence of necrosis
C. Astrocytic nucleus
C. Cellularity
D. Astrocytic cytoplasm
D. Vascular proliferation
E. Both neuronal and astrocytic cytoplasm
E. Extent of resection
130. All of the following may be seen with Kearns-
136. All of the following areas are common sites of
Sayre syndrome except
diffuse axonal injury except the
A. Deafness A. Midbrain
B. Acromegaly B. Corpus callosum
C. Ophthalmoplegia C. Fornix
D. Retinitis pigmentosa D. Corona radiata
E. Heart block E. Superior cerebellar peduncles
131. Straight filament (tau positive) tangles with glio- 137. Which of the following is a major diagnostic cri-
sis and neuropil threads in the basal ganglia and terion for neurofibromatosis type I?
brain stem region support a diagnosis of A. Six or more cafe-au-lait spots >15 mm in a
A. Alzheimers disease child
B. Progressive supranuclear palsy B. One plexiform neurofibroma
C. Striatonigral degeneration C. One Lisch nodule
D. Corticobasal degeneration D. Cousin with neurofibromatosis type I.
E. Facial freckling
E. None of the above
138. A distinguishing feature of myxopapillary ependy-
132. Type II glycogenosis is related to a defect on
moma versus ordinary ependymoma is
chromosome
A. Pseudorosettes
A. 1
B. True rosettes
B. 12
C. GFAP positivity
C. 17
D. Mucoid stroma
D. 22 E. Papillary architecture
E. X 139. The most common pathologic manifestation of
133. Perifascicular atrophy is a feature of Toxoplasmosis infection in the CNS in an immu-
A. Dermatomyositis nocompromised adult is
A. Abscess
B. Polymyositis
B. Hemorrhage
C. Neurogenic atrophy
C. Infarct
D. Myotonic dystrophy
D. Leptomeningitis
E. Steroid use
E. Granulomas
134. The most common cause of nontraumatic hemor- 140. Glycogen storage diseases are manifested typi-
rhage is cally on a muscle biopsy by
A. Vascular malformation A. Myonecrosis
B. Aneurysm B. Vacuolar changes
C. Coagulopathy C. Extensive fibrosis
D. Hypertension D. Fascicular atrophy
E. Vasculitis E. Targetoid fibers
CHAPTER 14 / WRITTEN SELF-ASSESSMENT QUESTIONS 205

141. Durcks nodes are a feature of 147. Meningiomas with a high likelihood of recurrence
A. CMV include all of the following except

B. Toxoplasmosis A. Metaplastic

C. Amebiasis B. Rhabdoid

D. Malaria C. Papillary

E. None of the above D. Clear cell

142. All are true regarding Hallervorden-Spatz disease E. Atypical


except 148. Neurofibromatosis type II is related to abnormal-
A. Onset in childhood ity on chromosome

B. Autosomal dominant A. 1

C. May cause choreoathetosis B. 17

D. Globus pallidus rust brown colored C. 3

E. Increased iron deposition in brain D. 22

143. Cysticercosis is caused by E. 9

A. Pork tapeworm 149. Tay-Sachs disease is caused by a deficiency in

B. Beef tapeworm A. -Galactosidase

C. Liver fluke B. Arylsulfatase

D. Lung fluke C. Hexosaminidase A

E. Nematode D. Glucocerebrosidase

144. Anti-MAG antibodies are most commonly associ- E. Iduronidase


ated with 150. Which of the following is true regarding type-IV
A. Paraneoplastic neuropathy glycogenosis?

B. Myasthenia gravis A. Myophosphorylase deficiency

C. Dysproteinemic neuropathy B. Autosomal dominant condition

D. Guillian-Barre disease C. Chromosome 17 defect

E. None of the above D. Females > males

145. The longitudinal boundaries defining the sarcom- E. Normal ischemic exercise test
ere are the 151. An oil-red-O stain may be most helpful in diag-
A. A bands nosing

B. I bands A. McArdles disease

C. M lines B. Mitochondrial disease

D. Z bands C. Carnitine deficiency

E. H bands D. Myotonic dystrophy

146. All of the following are true regarding amyloid E. Central core myopathy
angiopathy except 152. Chagas disease is caused by
A. Associated with Downs syndrome A. S. japonicum
B. Cause of lobar hemorrhage B. T. gondii
C. Highlighted with a congo red stain C. T. brucei
D. Chromosome 21 abnormality D. T. cruzi
E. Associated with Picks disease E. T. gambiense
206 NEUROPATHOLOGY REVIEW

153. All are true regarding Tay-Sachs disease except 159. The most common cytogenetic abnormality in
A. GM2 gangliosidosis meningiomas involves chromosome

B. Cherry red spot in macula A. 1


B. 2
C. Balloon neurons
C. 10
D. May see normal brain weight
D. 17
E. Hepatosplenomegaly
E. 22
154. Multisystem atrophy includes which of the follow-
ing conditions? 160. Retraction balls or spheroids and punctate hemor-
rhages are most characteristic of
A. Olivopontocerebellar atrophy
A. Contusion
B. Hallervorden-Spatz disease
B. Infarct
C. Corticobasal degeneration
C. Diffuse axonal injury
D. Friedrichs ataxia
D. Burst lobe
E. Huntingtons disease
E. None of the above
155. All are salient features of Creutzfeldt-Jakob
pathology except 161. Which of the following is associated with tuber-
ous sclerosis?
A. Spongiform change
A. Pilocytic astrocytoma
B. Microglial nodules
B. Optic nerve glioma
C. Gliosis
C. Subependymal giant cell astrocytoma
D. Neuronal loss
D. Brain stem glioma
E. Kuru plaques
E. Medulloblastoma
156. Charcot-Marie-Tooth disease may be caused by
162. Abnormal accumulation of sphingomyelin and
defects associated with which chromosomes?
cholesterol is a feature of
A. 1 and 9
A. Tay-Sach disease
B. 1 and 17 B. Niemann-Pick disease
C. 9 and 17 C. Gauchers disease
D. 17 and 22 D. Battens disease
E. X and 22 E. Mucolipidoses
157. Increased myofiber central nuclei may be seen in 163. The most common cause of death in Friedrichs
all of the following except ataxia is
A. Exercise-induced hypertrophy A. Brain infarct/stroke
B. Neurogenic atrophy B. Heart disease
C. Myotubular myopathy C. Herniation
D. Myotonic dystrophy D. Coagulopathy related bleed
E. Charcot-Marie-Tooth disease E. Associated neoplasm
158. Granulomatous vasculitis is a common feature of 164. All are true regarding Friedrichs ataxia except
A. Polyarteritis nodosa A. Autosomal recessive
B. Wegeners vasculitis B. Chromosome 9q defect
C. Lupus associated vasculitis C. GAA trinucleotide repeat disorder
D. Hypersensitivity angiitis D. Posterior columns spared
E. None of the above E. Ganglion cell/posterior root degeneration
CHAPTER 14 / WRITTEN SELF-ASSESSMENT QUESTIONS 207

165. The prion protein gene is located on which chro- 171. Solitary fibrous tumor can be distinguished from
mosome? meningioma by immunoreactivity to
A. 1 A. Vimentin
B. 17 B. CD34
C. 20 C. S-100 protein
D. 22 D. Cytokeratins AE1/3
E. None of the above E. Actin
166. Dilatation of muscle sarcoplasmic reticulum with 172. A Duret hemorrhage is located in the
tubular aggregates is a feature of A. Frontal lobe
A. Myasthenia gravis B. Basal ganglia
B. Myoadenylate deficiency C. Mamillary bodies
C. Malignant hyperthermia D. Cerebellum
D. Periodic paralysis E. Pons
E. None of the above 173. All of the following are associated with tuberous
167. Which is true regarding Dejerine-Sottas disease? sclerosis except
A. Autosomal dominant A. Schwannoma
B. X-linked B. Adenoma sebaceum
C. Presents in young adulthood C. Angiomyolipoma
D. Nerve enlargement D. Cardiac rhabdomyoma
E. Primarily axonal degenerative disease E. Lymphangioleiomyomatosis
168. Phytanoyl CoA -hydroxylase deficiency causes 174. Cherry red spots in the macula is a prominent fea-
A. Fabrys disease ture of all the following except

B. Refsums disese A. Tay-Sachs disease

C. Riley-Day syndrome B. Niemann-Pick disease

D. Canavans disease C. Gauchers disease

E. Metachromatic leukodystrophy D. Sialodosis

169. Intermediate filaments that link Z discs together E. They are seen in all of the above
in muscle are known as 175. Type IIb muscle fiber atrophy is a feature of
A. Actin A. Alcohol myopathy
B. Desmin B. Steroid myopathy
C. Myosin C. Chloroquine myopathy
D. Dystrophin D. AZT myopathy
E. Vimentin E. Cimetidine myopathy
170. The highest yield area to look for vasculitis on a 176. All of the following are true regarding primary
biopsy would be CNS lymphoma except
A. Leptomeninges A. More common in males
B. Cerebral cortex B. Most are B-cell type
C. White matter C. Most are diffuse small cell type
D. Hippocampus D. Most are parenchymal based
E. Basal ganglia E. Most are angiocentric histologically
208 NEUROPATHOLOGY REVIEW

177. von Hippel-Lindau disease is associated with all 183. Physaliphorous cells are a feature of which
of the following except tumor?
A. Pheochromocytoma A. Meningioma
B. Hemangiopericytoma B. Hemangioblastoma
C. Renal cell carcinoma C. Hemangiopericytoma
D. Secondary polycythemia vera D. Chondroma
E. Autosomal dominant inheritance pattern E. None of the above
178. Spinocerebellar degeneration with sensory neurop- 184. All of the following are autosomal dominant
athy as a result of chronic intestinal fat malabsorp- disorders except
tion is caused by deficiency of
A. Neurofibromatosis type I
A. Vitamin A
B. Neurofibromatosis type II
B. Vitamin B
C. Neurocutaneous melanosis
C. Thiamine
D. Tuberous sclerosis
D. Vitamin E
E. Vintamin K E. Sturge-Weber disease
179. A SOD1 gene defect on chromsome 21 is associ- 185. A trigeminal nevus flammeus is a marker of
ated with what disorder? A. Epidermal nevus syndrome
A. Beckers muscular dystrophy B. Cowden disease
B. Myotonic dystrophy C. Sturge-Weber disease
C. Refsums disease D. Gorlen syndrome
D. Amyotrophic lateral sclerosis E. Tuberous sclerosis
E. Werdnig-Hoffman disease 186. Sea blue histocytes in the bone marrow are a
180. All of the following inclusions are seen fre- feature of
quently in amyotrophic lateral sclerosis except A. Tay-Sachs disease
A. Hirano bodies B. Niemann-Pick disease
B. Bunina bodies
C. Ceroid-lipofucinosis
C. Hyaline inclusions
D. Cystinosis
D. Basophilic inclusions
E. Lafora body disease
E. Skeins
187. Giant aneurysm is used to denote a saccular
181. Which of the following cells is a normal constit- aneurysm greater than
uent of the endoneurial compartment
A. 0.5 cm
A. Lymphocytes
B. 1.0 cm
B. Eosinophils
C. 1.5 cm
C. Mast cells
D. Basophils D. 2.0 cm
E. Neutrophils E. 2.5 cm
182. CADASIL is associated with a mutation on the 188. The most common site of origin of choroid
notch 3 gene located on which chromosome? plexus papillomas in children is
A. 1 A. Lateral ventricle
B. 17 B. Third ventricle
C. 19 C. Fourth ventricle
D. 21 D. Brain stem
E. 22 E. Spinal cord
CHAPTER 14 / WRITTEN SELF-ASSESSMENT QUESTIONS 209

189. Cowden syndrome is associated with which of 195. Secondary polycythemia vera is associated with
the following? A. Hemangiopericytoma
A. Medulloblastoma B. Hemangioblastoma
B. Cerebellar gangliocytoma C. Meningioma
C. Melanoma D. Solitary fibrous tumor
D. Lymphoma E. Choroid plexus papilloma
E. Melanotic schwannoma 196. The accumulation of autofluorescent lipopigments
190. Which of the following is the result of a defect in neurons resulting in granular, fingerprint, or
on chromosome 1q? curvilinear inclusions by electron microscopy is
known as
A. Niemann-Pick disease
A. Gauchers disease
B. Gauchers disease
B. Battens disease
C. Morquios disease
C. Mucopolysaccharidoses
D. Sialodosis
D. Cystinosis
E. Mucolipidoses E. Fucosidosis
191. All of the following are characteristic of Leighs 197. Neuronal-PAS positive deposits, skin telangiecta-
disease except sia, renal insufficiency, and autonomic dysfunc-
A. Associated with mitochondrial defects tion are features of
B. Autosomal recessive A. Sialodosis
C. Most survive to early adulthood B. Tay-Sachs disease
D. Vascular proliferation in periventricular gray C. Fucosidosis
matter (brain stem) D. Fabrys disese
E. Lactic acidosis E. Lafora body disease
192. Ethanol is associated with all the following 198. All the following are related to gene defects on
except the X chromosome except
A. Anterior vermal atrophy A. Fabrys disease
B. Peripheral neuropathy B. Duchenne dystrophy
C. Corpus callosum demyelination C. Adrenoleukodystrophy
D. Thiamine deficiency D. Hunters mucopolysaccharidosis
E. Neurofibrillary tangles E. Lafora body disease
193. Lamellated cytoplasmic structures located within 199. Loss of anterior horn cells, tongue fasciculations,
the Schwann cell cytoplasm that increase with and hypotonia in a 2-mo-old is most likely due to
age are referred to as a disorder associated with what chromosome?
A. Corpuscles of Erzholz A. 1
B. 5
B. Lipofuscin
C. 14
C. Schmidt-Lanterman clefts
D. 16
D. Pi granules of Reich
E. 17
E. Space of Klebs
200. The ratio of myelinated to unmyelinated axons in
194. Moyamoya is characterized by all of the follow- a sural nerve biopsy would be best approximated
ing except by which of the following?
A. Amyloid deposition A. 1 : 2
B. Peak incidence in the 1st and 4th decades B. 1 : 4
C. Females > males C. 1 : 6
D. May see hemorrhage D. 1 : 8
E. Vessel occlusion with intimal fibroplasia E. 1 : 10
210 NEUROPATHOLOGY REVIEW

201. The most common cause of aseptic meningitis 206. The single best immunomarker for medul-
is loblastoma is which of the following?
A. Bacteria A. Cytokeratin
B. Virus B. GFAP
C. Fungus C. S-100
D. Parasite D. Synaptophysin
E. Prion protein disease E. Chromogranin
202. All are true regarding familial insomnia syndrome 207. Poor prognosis in medulloblastoma is associated
except with all of the following except
A. Age >3 yr
A. Most patients survive 10-20 years
B. Metastases at presentation
B. Autosomal dominant
C. Subtotal resection
C. Associated autonomic dysfunction
D. Large cell variant
D. A prion-protein-associated disease
E. Melanotic variant
E. Thalamic gliosis
208. Which of the following are true regarding pri-
203. The saltatory conduction pattern in the peripheral mary angiitis of the CNS?
nerve is most closely related to which stucture? A. ESR may be normal
A. Node of Ranvier B. Most common in older adults, typically
B. Periaxonal space of Klebs >60 yr
C. Neurofilaments C. Veins involved more than arteries
D. Schmidt-Lanterman clefts D. Marked by prominent eosinophilia
E. Vasa vasorum E. Almost always granulomatosis
204. The major defining histopathologic feature of a 209. Melanotic schwannoma is associated with which
saccular aneurysm is of the following
A. Breaks in the internal elastic membrane A. Turcot syndrome
B. Absence of the media B. Cowden disease
C. Neurofibromatosis type I
C. Inflammation
D. Tuberous sclerosis
D. Atherosclerotic change
E. Carneys compex
E. Intimal fibroplasia
210. The most common form of hereditary amyloid-
205. Genetic alterations associated with medul- osis peripheral neuropathy is related to a mutation
loblastoma most frequently involve chromosome of which gene?
A. 1 A. Light chain
B. 10 B. Light chain
C. 17 C. -Amyloid
D. 19 D. Transthyretin
E. 20 E. Albumin
15 Answers to the Written
Self-Assessment Questions

1. B (Chapter 1.I.A) Nissl substance is composed of 15. A (Chapter 3.I.B) Frontal lobe (with the most
rough endoplasmic reticulum. white matter) is the most common site of origin
for the low-grade fibrillary astrocytoma.
2. B (Chapter 2.I.A) Carbon monoxide poisoning
represents an example of anemic anoxia (insuf- 16. B (Chapter 4.I.C) Cytotoxic edema is caused by
ficent oxygen content in blood). impairment of the cell Na-K pump.
3. B (Chapter 3.I.A) The pleomorphic xanthoastro- 17. A (Chapter 5.I.A) Area cerebrovasculosa (cystic
cytoma is a WHO grade II tumor. mass with vessels) is associated with anence-
4. D (Chapter 4.I.B) Cerebral edema causes expan- phaly.
sion of the gyri. 18. D (Chapter 6.I.D) Devic-type multiple sclerosis
5. A (Chapter 5.I.A) Meckel-Gruber syndrome is is characterized by predominantly optic nerve and
an autosomal recessive condition. spinal cord involvement.
6. C (Chapter 6.I.D) HLA-DR15 is associated with 19. A (Chapter 8.III.E) Phosphorylated tau proteins
the development of multiple sclerosis. are associated with the development of neuro-
fibrillary tangles.
7. E (Chapter 7.I.) Increased astrocyte number and
size are seen with normal aging. 20. C (Chapter 9.II.C) Demyelination is not a fre-
quent complication of bacterial meningitis.
8. C (Chapter 9.II.A) Streptococcus is the most
common cause of an intracranial epidural abscess. 21. C (Chapter 9.II.C) Streptococcus agalactiae
9. B (Chapter 10.I.D) Alkaline phosphatase stains (group B) is the most common cause of neonatal
regenerating myofibers black. bacterial meningitis (along with E. coli).

10. C (Chapter 5.I.A) Hydromyelia is characterized 22. B (Chapter 5.I.B) Cyclopia is most commonly
by congenital dilatation of the central canal of the associated with holoprosencephaly.
spinal cord. 23. C (Chapter 10.III.A) Regenerating fibers are
11. A (Chapter 6.I.D) Neutrophils are not a histo- marked by cytoplasmic basophilia, nuclear
logic feature of the multiple sclerosis plaque. enlargement, and nucleolation.
12. E (Chapter 10.II.D) Ring fibers are more com- 24. E (Chapter 11.II.A) Shortened internodal dis-
monly encountered in myopathic processes such tances is a feature of remyelination.
as muscular dystrophy. 25. D (Chapter 1.I.B). Bergmann astrocytes are found
13. A (Chapter 1.I.A) Marinesco bodies are intra- in the cerebellum near the Purkinje cells.
nuclear neuronal inclusions. 26. E (Chapter 2.II.I) The Purkinje cells of the cere-
14. B (Chapter 2.I.C) Sommer sector (CA1) neurons bellum are relatively spared in hypoglycemic neu-
are the most sensitive to anoxic injury. ronal necrosis.

211
212 NEUROPATHOLOGY REVIEW

27. C (Chapter 3.I.D) True microcystic degeneration 44. B (Chapter 8.III.J) The CERAD criteria are
is more commonly a feature of glioma than based on a semiquantitative assessment of plaque
gliosis. frequency in the neocortex.
28. C (Chapter 4.I.D) Uncal or lateral transtentorial 45. A (Chapter 10.III.B) Most patients with dermato-
herniation may cause a Kernohans notch (dam- myositis respond well to steroid or immunosup-
age to contralateral cerebral peduncle). pressive therapy.
29. D (Chapter 5.I.B) Klippel-Feil abnormalities are 46. A (Chapter 5.I.C) A LIS-1 gene defect is associ-
more typically a feature of Chiari type II malfor- ated with the development of Miller-Dieker syn-
mation. drome agyria.
30. D (Chapter 9.II.C) Waterhouse-Friderichsen 47. E (Chapter 9.II.I) Demyelination is not a typical
(adrenal gland hemorrhage) is most commonly feature of meningovascular syphilitic disease.
associated with Neisseria meningitidis. 48. B (Chapter 10.III.C) The etiology of inclusion
31. B (Chapter 9.II.A) Listeriosis does not usually body myositis remains unknown.
result in granuloma formationit more typically 49. A (Chapter 1.I.B) Alzheimer type II astrocytes
manifests as acute leptomeningitis with abscesses. may be prominently seen in hepatic diseases
32. D (Chapter 6.I.F) There is no predisposition of marked by elevated ammonia levels.
immunocompromised individuals to develop acute 50. B (Chapter 2.II.D) Macrophage infiltration in an
hemorrhagic leukoencephalitis. infarct begins at about 24 hr.
33. B (Chapter 7.II.D) Aluminum toxicity is associ- 51. C (Chapter 3.I.F) Glioblastoma multiforme is
ated with neurofibrillary cytoskeletal alterations. the most common glioma type.
34. D (Chapter 8.III.I) -Amyloid in Alzheimers 52. B (Chapter 4.II.A) Epidural hemorrhage is asso-
disease is encoded for on chromosome 21. ciated with rupture of the middle meningeal
artery.
35. D (Chapter 10.III.A) Autophagic (rimmed) vacu-
53. D (Chapter 5.I.C) Cortical dysplasia is not asso-
oles are not a salient feature of polymyositis but
ciated with neurofibromatosis type II.
are more suggestive of inclusion body myositis.
54. A (Chapter 6.II.B) Globoid cell leukodystrophy
36. D (Chapter 10.III.B) Perifascicular atrophy is
is the result of a deficiency of galactocerebroside-
the most salient pathologic feature of dermatomy-
B galactosidase.
ositis.
55. C (Chapter 8.IV.M) Bunina bodies are more typ-
37. C (Chapter 1.I.B) Rosenthal fibers are fre- ically seen in amyotrophic lateral sclerosis. Lewy
quently seen in all the listed conditions except bodies may be encountered in a subset of Alzhei-
fibrillary astrocytoma. mers patients.
38. D (Chapter 2.II.A) Cerebral blood flow consti- 56. A (Chapter 9.III.B) Cryptococcus is a budding
tutes about 20% of cardiac output. yeast with a thick mucopolysaccharide wall.
39. D (Chapter 3.I.B) Trisomy 22 is generally not a 57. C (Chapter 9.III.E) The features described are
feature of diffuse astrocytomas. most likely attributable to Mucormycosis.
40. B (Chapter 4.I.D) Compression of the posterior 58. C (Chapter 5.I.D) Lissencephaly or agyria is asso-
cerebral artery in lateral transtentorial herniation ciated with developmental abnormalities which
may result in an infarct of the calcarine cortex. occur during the 11th13th wk of gestation.
41. B (Chapter 5.I.B) The constellation of findings 59. C (Chapter 9.III.G) A protoplasmic body is seen
listed is classic for Dandy-Walker syndrome. in Blastomycosis infection (fungal infection).
42. D (Chapter 9.II.F) A Ziehl-Neelsen stain is the 60. E (Chapter 8.III.K) Infarcts of >100 mL volume
most useful one listed in identifying mycobacte- are associated with the clinical development of
rial organisms. dementia.
43. E (Chapter 6.II.A) Metachromatic leukodystro- 61. C (Chapter 1.I.C) The fried egg appearance of
phy is the result of an arylsulfatase A deficiency oligodendrocytes is related to delays in formalin
(autosomal recessive on chromosome 22q). Arcu- fixation (an artifact).
ate fibers are preserved and white matter macro- 62. D (Chapter 2.II.E) The least common site for a
phages are filled with sulfatides. lacunar infarct is in the mamillary bodies.
CHAPTER 15 / ANSWERS TO THE WRITTEN SELF-ASSESSMENT QUESTIONS 213

63. E (Chapter 3.I.F) Perinecrotic pseudopalisading 83. D (Chapter 11.III.A) Mortons neuroma arises
is a feature more commonly observed in glio- associated with the interdigital nerve of the foot
blastoma multiforme than metastatic carcinoma. at intermetatarsal sites.
64. C (Chapter 4.II.B) About 50% of acute subdural 84. C (Chapter 9.IV.I) Negri bodies are intraneuro-
hematomas are associated with a skull fracture. nal cytoplasmic inclusions.
65. E (Chapter 5.I.D) The features described are 85. C (Chapter 1.I.E) Microglial cells are HAM56
most characteristic of the Walker-Warburg syn- positive.
drome.
86. A (Chapter 2.III.A) Tuberous sclerosis is not
66. E (Chapter 6.II.C) Most forms of adrenoleuko- associated with the development of saccular aneu-
dystrophy are X-linked. rysms.
67. D (Chapter 10.IV.A) Target fibers are associated
87. E (Chapter 3.I.M) Increased mitotic activity is
with neurogenic atrophy.
not a feature of the typical pleomorphic xantho-
68. A (Chapter 11.II.C) Diphtheritic neuropathy is a astrocytoma.
demyelinative neuropathy related primarily to
Schwann cell dysfunction. 88. A (Chapter 3.VIII.B) The Langerhans cell his-
tiocytes stain with CD1a.
69. D (Chapter 8.III.E) Silver stains (bodian) are
useful in highlighting neurofibrillary tangles. 89. B (Chapter 4.III.D) Raccoon eyes (periorbital
70. B (Chapter 8.IV.B) The huntington gene is ecchymoses) are associated with basilar skull
located on chromosome 4p. fractures.
71. C (Chapter 9.III.I) Actinomycosis bacteria are 90. B (Chapter 5.I.G) Narrowing of the superior
associated with sulfur granule formation. temporal gyrus is seen in Downs syndrome.
72. A (Chapter 9.IV.A) Poliovirus more typically 91. A (Chapter 6.II.D) Megalencephaly is a com-
results in Cowdry type B inclusions. mon feature of Alexanders disease.
73. D (Chapter 1.I.D) Blepharoplasts are the ciliary 92. B (Chapter 8.IV.B) Vonsattel grade I lesions of
body attachments in ependymal cells. Huntingtons chorea are grossly normal with up
74. C (Chapter 2.III.A) Approximately 20% of sac- to 50% neuronal loss in the striatum and gliosis
cular aneurysms are bilateral. in the caudate and putamen.
75. B (Chapter 3.I.J) Eosinophilic granular bodies 93. D (Chapter 9.IV.J) Retrograde axoplasmic trans-
are observed in a majority of pilocytic astrocy- port is associated with herpes virus.
tomas. 94. C (Chapter 5.I.J) Sommer sector (CA1), CA4,
76. B (Chapter 4.II.C) Traumatic basilar subarach- and the dentate are the areas most severely
noid hemorrhages are related to vertebral artery affected (neuronal loss and gliosis ) in hippo-
tears. campal sclerosis.
77. D (Chapter 5.I.E) A brain weight of 2350 g 95. C (Chapter 10.IV.C) Myasthenia gravis is char-
would represent megaloencephaly. acterized by the presence of circulating antibodies
78. E (Chapter 6.II.C) All of the organs listed may to the acetylcholine receptor.
contain inclusions in adrenoleukodystrophy 96. A (Chapter 11.III.B) Guillian-Barre syndrome is
except for the pancreas. not associated with HLA B27.
79. A (Chapter 9.IV.A) Granulomatous inflamma-
97. D (Chapter 1.I.H) Layer III of the cerebral cor-
tion is not a typical feature of viral infections in
tex is known as the outer pyramidal layer.
the CNS.
80. B (Chapter 9.IV.G) Most patients with SSPE 98. A (Chapter 2.III.C) Dolichoectasia (elongation,
die from the disease. widening, and tortuosity of the artery) is associ-
ated with the formation of fusiform aneurysms.
81. B (Chapter 10.IV.B) Fascicular atrophy is com-
monly encountered in muscle biopsies of patients 99. D (Chapter 3.I.U) Necrosis is distinctly uncom-
with spinal muscular atrophy, including mon in gangliogliomas.
Kugelberg-Welander disease. 100. D (Chapter 3.X.A) Paraganglioma is composed
82. E (Chapter 8.IV.B) Huntingtons chorea is a tri- of cells arranged in nests or lobules referred to as
nucleotide (CAG) repeat disorder. zellballen.
214 NEUROPATHOLOGY REVIEW

101. C (Chapter 4.V.A) The wounding capacity of a 121. E (Chapter 1.I.H) The endplate of the hippo-
gunshot wound is most related to the velocity of campus is designated CA4.
the bullet. 122. C (Chapter 2.IV.D) Cavernous angiomas are
102. C (Chapter 5.II.G) Periventricular coagulative characterized by back-to-back vessels.
necrosis in a premature infant is termed periven- 123. B (Chapter 3.II.E) The prognosis of central neu-
tricular leukomalacia. rocytoma is good. Most patients live well beyond
103. D (Chapter 9.IV.L) Congenital CMV infection 2 yr of diagnosis.
does not result in megalencephaly. 124. D (Chapter 4.IV.B) A burst lobe represents a
104. B (Chapter 10.IV.D) Enhanced neurotransmis- form of contrecoup contusion.
sion with repeated stimulation is seen with
125. E (Chapter 5.III.A) The neurofibromin gene
Lambert-Eaton syndrome.
(neurofibromatosis type I) is located on chromo-
105. A (Chapter 3.X.D) Cholesterol accounts for the some 17q.
motor oil quality of craniopharyngioma cyst
126. A (Chapter 6.II.F) A relative perivascular white
fluid.
matter sparing (tigroid demyelination) in charac-
106. E (Chapter 11.III.D) Leprosy neuropathy is not teristic of Pelizaeus-Merzbacher disease.
a toxin-mediated process.
127. D (Chapter 9.IV.O) Viral inclusions are not
107. D (Chapter 8.IV.C) Autosomal dominant trans- seen with HTLV-1 infection.
mission of Parkinsons disease is linked with the
-synuclein gene. 128. B (Chapter 11.III.E) Paraneoplastic neuropathy
is most often encountered with lung carcinoma
108. A (Chapter 10.V.B) Nemaline rods are derived
(especially small cell carcinoma).
from -actinin or Z-band material.
129. B (Chapter 8.IV.C) Lewy bodies are intraneuro-
109. C (Chapter 1.I.H) The pencil bundles of Wilson
nal cytoplasmic inclusions.
are thin fascicles of myelinated fibers seen in the
caudate and putamen. 130. B (Chapter 10.VI.A) Acromegaly is not associ-
ated with Kearns-Sayre syndrome.
110. B (Chapter 2.III.B) Aspergillus is the most com-
mon cause of mycotic (fungal) aneurysms. 131. B (Chapter 8.IV.E) The features described are
111. A (Chapter 3.II.B) Chromosome 1p and 19q characteristic of progressive supranuclear palsy.
deletions have been associated with chemorespon- 132. C (Chapter 10.VI.B) Type II glycogenosis (acid
siveness in oligodendrogliomas. maltase deficiency) is caused by a chromosome
112. D (Chapter 4.V.B) Blunt head injury sustained 17q defect.
in a landslide is an example of static loading. 133. A (Chapter 1.II.D) Ischemia-related perifascicu-
113. D (Chapter 5.II.O) The least likely site of biliru- lar atrophy is a feature of dermatomyositis.
bin deposition in kernicterus is in the claustrum. 134. D (Chapter 2.V.A) Hypertension is the most
114. C (Chapter 6.II.E) Canavans disease is caused common cause of nontraumatic hemorrhage.
by a deficiency in aspartoacylase. 135. E (Chapter 3.III.B) Extent of surgical resection
115. B (Chapter 10.V.A) Most congenital myopathies is the most important factor in predicting out-
are nonprogressive or only slowly progressive. come in ependymomas.
116. B (Chapter 11.III.D) Lyme disease is acquired 136. A (Chapter 4.V.C) Midbrain is the least com-
from a tick bite. mon site (listed) of diffuse axonal injury.
117. C (Chapter 8.IV.C) The ubiquitin immunostain 137. B (Chapter 5.III.A) One plexiform neurofibroma
can be useful in identifying Lewy bodies. is a major criterion for the diagnosis of neuro-
118. E (Chapter 9.IV.M) Oligodendroglial cell fibromatosis type I.
Cowdry A inclusions are characteristic of PML. 138. D (Chapter 3.III.E) Mucoid stroma (mucin posi-
119. B (Chapter 9.IV.N) The term vacuolar myelopa- tive) is a distinguishing feature of myxopapillary
thy is used to described HIV-associated spinal ependymoma. All of the other features may be
cord changes. observed in ordinary ependymomas.
120. E (Chapter 10.VI.A) Ragged red fibers are asso- 139. A (Chapter 9.V.A) Abscess is the most common
ciated with mitochondrial abnormalities and are pathologic manifestation of Toxoplasmosis infec-
generally not seen with central core disease. tion in adults who are immunocompromised.
CHAPTER 15 / ANSWERS TO THE WRITTEN SELF-ASSESSMENT QUESTIONS 215

140. B (Chapter 10.VI.B) Glycogen storage diseases 159. E (Chapter 3.IV.A) Deletions on chromosome
are typically vacuolar myopathies. 22 are the most common cytogenetic abnormality
141. D (Chapter 9.V.C) Dhrcks nodes (foci of necro- in meningioma.
sis rimmed by microglia) are a feature of malaria. 160. C (Chapter 4.V.C) Diffuse axonal injury is mani-
142. B (Chapter 8.IV.F) Hallervorden-Spatz disease fested by punctate or streak-like hemorrhages and
is an autosomal recessive condition. retraction balls or spheroids.
143. A (Chapter 9.V.D) Cysticercosis is caused by 161. C (Chapter 5.III.C) The subependymal giant-cell
the pork tapeworm (Taenia solium). astrocytoma is associated with tuberous sclerosis.
144. C (Chapter 11.III.E) Anti-MAG antibodies are 162. B (Chapter 7.I.D) An accumulation of sphingo-
encountered in 5090% of cases of dysproteine- myelin and cholesterol as a result of sphingomye-
mic neuropathy. linase deficiency is a feature of Niemann-Pick
disease.
145. D (Chapter 1.II.G) The sarcomere runs from Z
band to Z band. 163. B (Chapter 8.IV.K) The most common cause of
death in Friedrichs ataxia is related to heart
146. E (Chapter 2.V.A) Amyloid angiopathy is not disease.
associated with Picks disease; it is associated
with Alzheimers disease. 164. D (Chapter 8.IV.K) Posterior column degenera-
tion is commonly seen in Friedrichs ataxia.
147. A (Chapter 3.IV.A) Metaplastic meningiomas
are less likely to recur than the other types listed. 165. C (Chapter 9.VI.A) The prion protein gene is
located on chromosome 20.
148. D (Chapter 5.III.B) The merlin gene of neurofi-
bromatosis type II is located on chromosome 22. 166. D (Chapter 10.VI.F) Dilated sarcoplasmic reticu-
lum and tubular aggregates are features of the
149. C (Chapter 7.I.B) Tay-Sachs disease is a result
periodic paralyses (channelopathies).
of a hexosaminidase A deficiency.
167. D (Chapter 11.III.F) Nerve enlargement is a
150. A (Chapter 10.VI.B) Type V glycogenosis
common finding in Dejerine-Sottas disease
(McArdles disease) is caused by myophosphory-
(HMSN III).
lase deficiency.
168. B (Chapter 11.III.G) Refsums disease is due to
151. C (Chapter 10.VI.C) An oil-red-O stain for lipid
phytanoyl CoA -hydroxylase deficiency.
would be most useful in diagnosing carnitine
deficiency (a lipid-storage myopathy). 169. B (Chapter 1.II.G) Desmin filaments link Z
discs together and join them to the plasmalemma.
152. D (Chapter 9.V.F) Trypanosoma cruzi (T. cruzi)
is the causative agent of Chagas disease. 170. A (Chapter 2.VI.A) The leptomeningeal vessels
are the highest yield area to look for vasculitis on
153. E (Chapter 7.I.B) Visceral organ involvement
a biopsy.
including hepatosplenomegaly is not a prominent
feature of Tay-Sachs disease. 171. B (Chapter 3.IV.B) Solitary fibrous tumors, in
154. A (Chapter 8.IV.I) Multisystem atrophy includes contrast to meningiomas, are generally CD34
olivopontocerebellar atrophy, Shy-Drager syn- positive.
drome, and striatonigral degeneration. 172. E (Chapter 4.I.D) Duret hemorrhages are mid-
155. B (Chapter 9.VI.B) Microglial nodules are not a line brain stem hemorrhages associated with lat-
feature of Creutzfeldt-Jakob disease. eral transtentorial herniation.

156. B (Chapter 11.III.F) Abnormalities on chromo- 173. A (Chapter 5.III.C) Schwannoma is not associ-
somes 1, 17, and X are associated with Charcot- ated with tuberous sclerosis.
Marie-Tooth disease. 174. C (Chapter 7.I.E) Cherry red spots are not a
157. B (Chapter 1.II.B) Increased central nuclei (nor- salient feature of Gauchers disease.
mally <35% of all fibers) is not typically seen in 175. B (Chapter 10.VII.E) Steroid myopathy is
neurogenic atrophy. marked by a type IIb muscle fiber atrophy.
158. B (Chapter 2.VI.A) Wegeners vasculitis is com- 176. C (Chapter 3.VIII.A) Most primary CNS lymph-
monly associated with granulomatous inflam- omas are diffuse large B-cell type (REAL classi-
mation. fication).
216 NEUROPATHOLOGY REVIEW

177. C (Chapter 5.III.E) von Hippel-Lindau is associ- 193. D (Chapter 1.III.F) The pi granules of Reich are
ated with hemangioblastoma, not hemangioperi- lamellated structures that increase in number with
cytoma. increasing age in the Schwann cell cytoplasm.
178. D (Chapter 11.III.H) Vitamin E deficiency may 194. A (Chapter 2.VII.D) There is no amyloid deposi-
manifest as spinocerebellar degeneration with a tion associated with moya moya.
sensory neuropathy caused by fat malabsorption. 195. B (Chapter 3.IV.E) Hemangioblastoma may pro-
duce erythropoietin resulting in a secondary poly-
179. D (Chapter 8.IV.M) SOD1 gene defect on chro-
cythemia vera.
mosome 21 is associated with amyotrophic lateral
196. B (Chapter 7.I.F) The features described are
sclerosis.
characteristic of Battens disease or the ceroid-
180. A (Chapter 8.IV.M) Hirano bodies are not a fre- lipofuscinoses.
quent finding in amyotrophic lateral sclerosis. 197. E (Chapter 7.I.L) All the features listed are
181. C (Chapter 1.III.B) Mast cells are normally pres- salient features of Fabrys disease.
ent in the endoneurium in small numbers. 198. E (Chapter 7.I.N) Lafora-body disease is associ-
182. C (Chapter 2.VII.B) The notch 3 gene of ated with a chromosome 6q defect.
CADASIL is located on chromosome 19. 199. B (Chapter 8.IV.N) The presentation is consis-
tent with Werdnig-Hoffman disease associated
183. E (Chapter 3.IV.B) Physaliphorous cells are a with a chromosome 5 abnormality.
feature of chordoma.
200. B (Chapter 1.III.J) The best approximation of a
184. E (Chapter 5.III.C) Sturge-Weber disease is not myelinated : unmyelinated axon ratio is 1 : 4.
an autosomal dominant condition. 201. B (Chapter 9.IV.D) Viral infections account for
185. C (Chapter 5.III.C) A nevus flammeus (port the majority of aseptic meningitis cases.
wine stain) in the trigeminal region is associated 202. A (Chapter 9.VI.E) Most patients with familial
with Sturge-Weber disease. insomnia syndrome die within a few years of
symptom onset.
186. B (Chapter 7.I.D) Sea blue histiocytes are a fea-
ture of Niemann-Pick disease. 203. A (Chapter 1.III.H) Discontinuous saltatory con-
duction is most related to the nodes of Ranvier.
187. E (Chapter 2.III.A) Giant aneurysms are >2.5 204. B (Chapter 2.III.A) Absence of the media
cm in size. defines the saccular aneurysm.
188. A (Chapter 3.V.A) The lateral ventricle is the 205. C (Chapter 3.VI.C) Medulloblastomas are associ-
most common site of origin of choroid plexus ated with chromosome 17 alterations.
papilloma in children; 4th ventricle in adults. 206. D (Chapter 3.VI.C) Synaptophysin is the single
189. B (Chapter 5.III.H) The dysplastic cerebellar best marker for medulloblastoma.
gangliocytoma (Lhermette-Duclos) is associated 207. A (Chapter 3.VI.C) Age <3 yr is associated
with Cowden syndrome. with a poor prognosis in medulloblastoma.
190. B (Chapter 7.I.E) Gauchers disease is caused 208. A (Chapter 2.VI.A) Erythrocyte sedimentation
by glucocerebrosidase deficiency related to a rates are often normal or only slight elevated in
defect on chromosome 1q. primary angiitis of the CNS.
209. E (Chapter 3.VII.A) Melanotic schwannoma
191. C (Chapter 7.I.O) Most patients with Leighs (often with psammoma bodies) is associated with
disease die in the 1st decade of life. Carneys complex.
192. E (Chapter 7.II.C) Ethanol is not associated 210. D (Chapter 11.III.I) Transthyretin (prealbumin)
with neurofibrillary tangle formation. is related to hereditary amyloid neuropathy.
Bibliography

Birch, R., Bonney, G., Wynn Parry, C. B. (1998). Surgical Graham, D. I., Lantos, P. L. (eds.) (1997). Greenfields
Disorders of the Peripheral Nerves. Churchill Living- Neuropathology. Oxford University Press, New York.
stone. Edinburgh. Kleihues, P., Cavenee, W. K. (eds.) (2000). Pathology
Burger, P. C., Scheithauer, B. W. (1994). Tumors of the and Genetics: Tumours of the Nervous System. IARC
Central Nervous System. Armed Forces Institute of Press, Lyon, France
Pathology, Washington, DC. Loughlin, M. (1993). Muscle Biopsy. A Laboratory Inves-
Burger, P. C., Scheithauer, B. W., Vogel, F. S. (1991). tigation. Butterworth-Heinemann, Oxford.
Surgical Pathology of the Nervous System and Its Cov- Nelson, J. S., Parisi, J. E., Schochet, S. S., Jr. (eds.) (1993).
erings. Churchill Livingstone, New York. Principles and Practice of Neuropathology. Mosby, St.
deGirolami, U., Anthony, D. C., Frosch, M. D. (1999). Louis, MO.
Peripheral nerve and skeletal muscle and the central Ortiz-Hidalgo, C., Weller, R. O. (1997). Peripheral ner-
nervous system. In: Pathologic Basis of Disease vous system. In: Histology for Pathologists (Sternberg,
(Cotran, R. S., Kumar, V., Collins, T., eds), W.B. Saun- S. S., ed.). Lippincott-Raven, Philadelphia.
ders Co., Philadelphia. Prayson, R. A., Cohen, M. L. (2000). Practical Differential
Davis, R. L., Robertson, D. M. (eds.) (1999). Textbook of Diagnosis in Surgical Neuropathology. Humana Press,
Neuropathology. Williams & Wilkins, Baltimore, MD. Totowa, NJ.
Engel, A. G., Franzini-Armstrong, C. (eds.) (1993). Prin- Schochet, S. S. Jr. (1986). Diagnostic Pathology of Skele-
ciples and Practice of Neuropathology. Mosby, St. tal Muscle and Nerve. Appleton-Century-Crofts, Nor-
Louis, MO. walk, CT.
Fuller, G. N., Burger, P. C. (1997). Central nervous sys- Vinters, H. V., Farrell, M. A., Mischel, P. S., Anders, K.
tem. In: Histology for Pathologists (Sternberg, S. S., H. (1998). Diagnostic Neuropathology. Marcel Dekker,
ed.), Lippincott-Raven, Philadelphia. Inc., New York.

217
Index 219
INDEX
A Amputation neuroma, 86
A band, 4 Amyelia, 41
A2B5, 21 Amyloid, 5960, 77, 179 (Fig.)
Abrasions, 38 Amyloid angiopathy, 10
Abscess, 66, 68, 69, 71, 145 (Fig.) Amyloid neuropathy, 89, 179 (Fig.)
Absidia, 68 Amyotrophic lateral sclerosis, 63
Acanthamoeba, 71 Anaplastic astrocytoma, 16
Acid maltase, 80, 183 (Fig.) Anaplastic ependymoma, 23
Acid phosphatase, 75, 80, 81, 83 Anaplastic oligodendroglioma, 22
Actin, 4 ANCA , 11
Actinomyces, 69 Anemic anoxia, 7, 9
Activator protein deficiency, 55 Anencephaly, 41
Acute disseminated encephalomyelitis, 52 Anesthetic stimulation test, 81
Acute hemorrhagic leukoencephalitis, 52 Aneurysm (see saccular aneurysm)
Acyl-CoA dehydrogenase, 81 Angiokeratoma corporis diffusum, 57
Adenohypophysis, 4, 144 (Fig.) Angiomatous meningioma, 24
Adenoma sebaceum, 48 Angiotropic large cell lymphoma, 29, 94 (Fig.)
Adrenoleukodystrophy, 5354, 88 Anti-MAG antibody, 87
Adrenomyeloneuropathy, 53 Anoxia, 7, 45, 149 (Fig.)
Agenesis of corpus callosum, 32, 44,102 (Fig.) Anoxic anoxia, 7
Agenesis of septum pellicidium, 45 Anti-Hu antibody, 87
Aging brain, 59 Antoni A/B patterns, 28, 99 (Fig.)
Agyria, 43, 96 (Fig.) APC gene, 49
AIDS, 25, 29, 7071 Aplasia of vermis, 45
Air embolism, 9 Apolipoprotein E, 59
Alcian blue, 24 Apophysomyces, 68
Alcohol, 47, 58, 81, 89, 168 (Fig.) Apoplexy, 31
Aldolase, 76 Aqueductal malformations, 42
Alexanders disease, 54, 117 (Fig.) Arachnoid cap cells, 3, 24
Alkaline phosphatase, 75, 174 (Fig.) Arachnoid cyst, 30
Alpha-actinin, 79 Arachnoiditis ossificans, 3
Alpha-fucosidase, 57 Arbovirus, 69
Alpha-galactosidase A, 57 Area cerebrovasculosa, 41
Alpha-ketoacid dehydrogenase, 58 Arsenic, 58
Alpha-L-iduronidase, 56 Arteriovenous malformation, 10, 135 (Fig.),
Alpha-synuclein, 61 143 (Fig.)
Aluminum, 58 Arylsulfatase A, 53
Alzheimer type I astrocyte, 2, 71, 136 (Fig.) Aseptic meningitis, 6971
Alzheimer type II astrocyte, 2, 54, 58, 155 (Fig.) Ash-leaf patch, 48
Alzheimers disease, 10, 5960, 120 (Fig.) Aspartoacylase, 54
Amebiasis, 71, 128 (Fig.) Aspergillus, 10, 68, 126 (Fig.)
Ammons horn, 3 Astroblastoma, 20

219
220 Index

Astrocyte, 2, 167 (Fig.) C


Astrocytoma, 1520, 115 (Fig.) CADASIL, 12, 159 (Fig.)
Ataxiatelangiectasia, 49 Caf-au-lait macules, 47, 114 (Fig.)
Atherosclerotic angiopathy, 12 Caffeine stimulation test, 81
ATPase, 76, 173 (Fig.) Calcinosis, 77
Atrophy (muscle), 4 Calpain 3, 83
Atypical meningioma, 2425 Canavans disease, 54
Atypical teratoid tumor, 27 Candida, 68, 78
Axillary freckling, 114 (Fig.) Capillary telangiectasia, 10
Axon, 1 Carbon monoxide, 9, 58
Axonal degeneration, 8589, 180 (Fig.) Carcinoma (see also metastatic disease), 98 (Fig.)
Axonal regeneration, 85, 181 (Fig.) Carney complex, 28
Axonal spheroid, 2 Carnitine, 81
AZT, 77, 81 Carnitine palmitoyltransferase, 81
Caveolin 3, 83
Cavernous malformation, 10, 137 (Fig.)
B Cavum septi pellucidi, 45, 111 (Fig.)
Bacterial myositis, 77 CD1a, 29
Balloon cells, 43 CD4, 71, 76
Bal multiple sclerosis, 52 CD5, 29
Bands of Bungner, 85 CD8, 77
Basal cell nevus syndrome, 49 CD10, 29
Basal ganglia (architecture), 3, 141 (Fig.) CD11c, 29
Basal gray matter necrosis, 46 CD15, 29
Basilar fracture (skull), 37 CD20, 29
Battens disease, 56 CD23, 29
Battle sign, 3738 CD30, 29
Becker dystrophy, 82 CD34, 25
Bergmann astrocytes (gliosis), 2, 7, 151 (Fig.) CD68, 29
Beriberi, 58, 8889 CD79a, 29
Beta-amyloid, 10, 60 CDK4, 16
Beta-galactosidase, 55, 56 Cellular schwannoma, 28
Beta-glucuronidase, 56 Central chromatolysis, 1, 148 (Fig.)
Bilirubin encephalopathy, 47, 156 (Fig.) Central core disesae, 79
Binswangers disease, 12 Central herniation, 35
Birbeck granules, 29, 138 (Fig.) Central neurocytoma, 22, 98 (Fig.)
Blastomycosis, 68 Central pontine myelinolysis, 5253, 58
Blepharoplast, 2, 22, 23 Centrinuclear myopathy, 79
Borrelia burgdorferi, 67, 87 CERAD criteria (Alzheimers disease), 60
Braak and Braak staging (Alzheimers Cerebellar cortical dysplasia, 45
disease), 60 Cerebellar heterotopia, 45
Brain stem glioma, 19, 105 (Fig.) Cerebellar liponeurocytoma, 22
Bunina bodies, 63 Cerebellum (architecture), 3, 149 (Fig.)
Burst lobe, 38 Cerebral neuroblastoma, 27
Butterfly glioma, 17 Cerebral perfusion pressure, 8
Index 221

Cerebritis, 66, 68 Corpora amylacea, 2, 59, 125 (Fig.)


Cerebro-ocular dysplasia-muscular dystrophy Corpora arenacea, 4
syndrome, 44 Corpuscles of Erzholz, 6
Ceroid-lipofuscinoses, 56, 177 (Fig.) Cortex (architecture), 3
Chagas disease, 72 Cortical dysplasia, 19, 21, 22, 43, 45, 48, 163 (Fig.)
Channelopathies, 81 Corticobasal degeneration, 62
CharcotBouchard microaneurysms, 12 Coup contusion, 38
CharcotMarieTooth disease, 87 Cowden syndrome, 21, 49
Cherry red spot, 55, 56 Cowdry A inclusion, 2, 69, 71, 123 (Fig.), 129
Chiari malformation, 4243, 113 (Fig.) (Fig.), 151 (Fig.)
Chlamydoabsidia, 68 Cowdry B inclusion, 2, 69
Chloroquine, 82 Coxsackie virus, 69, 77
Chondroid chordoma, 25 CPK, 76, 78, 82
Chordoid glioma, 21 Craniolacunia, 43
Chordoid meningioma, 24 Craniopharyngioma, 31, 105 (Fig.)
Chordoma, 25, 99 (Fig.) Craniorachischisis totalis, 41
Choroid plexus, 3, 133 (Fig.) Cranium bifudum, 41
Choroid plexus carcinoma, 26 Creutzfeldt astrocyte, 2
Choroid plexus cyst, 30 CreutzfeldtJakob disease, 72, 124 (Fig.)
Choroid plexus papilloma, 26, 107 (Fig.) Cryptococcus, 68, 78, 128 (Fig.)
Chromogranin , 30 Crystal violet, 10, 60
Chronic inflammatory demyelinating Curvilinear bodies, 82, 177 (Fig.)
polyradiculopathy, 8687 Cyclopia, 42, 45
ChurgStrauss vasculitis, 11 Cysticercosis, 72, 139 (Fig.)
Class A nerve fibers, 5 Cystinosis, 57
Class B nerve fibers, 5 Cytochrome oxidase, 76
Class C nerve fibers, 5 Cytokeratin, 17, 2427, 30, 32
Clear cell meningioma, 25 Cytomegalovirus, 7071, 87, 123 (Fig.), 146 (Fig.)
Clostridium, 77 Cytotoxic edema, 35
Coccidiomycosis, 68
Cockaynes disease, 54 D
Colloid cyst, 29, 106 (Fig.) DandyWalker syndrome, 43, 45
Colpocephaly, 44 DejerineSottas disease, 88
Comminuted fracture, 37 Demyelinating disease, 5153
Compound fracture, 37 Demyelinative neuropathy, 8688
Compression neuropathy, 86 Dendrite, 1
Congenital fiber type disproportion, 80 Denervation atrophy, 78
Congenital muscular dystrophy, 83 Dense core granules, 22
Congenital myopathies, 79 Dentorubropallidoluysial atrophy, 63
Congo red, 10, 60, 72, 75 Depressed fracture (skull), 37
Congophilic angiopathy, 10 Dermatomyositis, 77, 171 (Fig.)
Connexin-3.2, 88 Dermoid cyst, 30
Contrecoup contusion, 38 Desmin, 5
Contusion, 3738 Desmoplastic infantile ganglioglioma, 20, 21
Core fibers, 78 Desmoplastic astrocytoma of infancy, 20
222 Index

Desmoplastic medulloblastoma, 27 Entrapment neuropathy, 86


DJvic multiple sclerosis, 52 Ependyma, 2
Diabetic neuropathy, 89 Ependymal cyst, 30
Diastatic fracture (skull), 37 Ependymal rosettes, 22, 23, 127 (Fig.)
Diastematomyelia, 42 Ependymitis, 66
Dieterle, 67 Ependymoblastoma, 27
Diffuse anoxic encephalopathy, 8 Ependymoma, 23, 102 (Fig.), 111 (Fig.), 127
Diffuse axonal injury, 36, 39 (Fig.)
Diffuse Lewy body disease, 61 Epidermal growth factor receptor (EGFR), 17
Diffuse lobar sclerosis, 46 Epidermal nevus syndrome, 43
Diffuse melanocytosis, 26 Epidermoid cyst, 30
Diphtheria neuropathy, 87 Epidural abscess, 65
Diplomyelia, 42 Epidural hematoma, 36
Distal axonopathy, 86 Epineurium, 5
Dolichoectasia, 10 Epithelial membrane antigen (EMA), 5, 17,
Downs syndrome, 10, 45 2328, 30
Duchenne dystrophy, 82, 181 (Fig.) EpsteinBarr virus, 29
Drcks nodes, 72 ErdheimChester disease, 29
Duret hemorrhage, 35, 163 (Fig.) Erythema chronicum migrans, 68, 87
Dynamic loading, 38 Esterase, 7576, 78
Dysembryoplastic neuroepithelial tumor, 22, 43 Estrogen receptors, 24
Dysmyelinating disease, 51, 5354 tat cribl, 8
Dysplastic gangliocytoma of the cerebellum, 1, 49 tat lacunair, 8
Dysproteinemic neuropathy, 87 tat marbl, 46
Dystrophin, 82 Ethanol (see alcohol)
External herniation, 36
E
Eastern equine encephalitis, 69
Ecchordosis physaliphora, 25 F
Ecchymoses, 38 Fabrys disease, 57
Echovirus, 69 Facio-scapulo-humeral dystrophy, 8283
E. coli, 66 Familial insomnia, 7273
Edema, 35, 157 (Fig.) Fascicular atrophy, 4, 78, 183 (Fig.)
Edwards syndrome, 45 Fat emboli, 39
Emerin, 82 Feigin tumor, 18
EmeryDreyfuss dystrophy, 82 Ferruginization, 1, 46
Encephalitis, 6667, 70, 71 Fetal alcohol syndrome, 47, 58
Encephalocele, 20 Fiber type grouping, 4
Endocarditis, 9 Fibrillary astrocyte, 2
Endodermal cyst, 30 Fibrous meningioma, 24
Endomysium, 4 FlexnerWintersteiner rosettes, 22
Endoneurium, 5 Fracture (skull), 3637
Endothelial proliferation, 1517, 23 Frataxin, 63
Entamoeba histolytica, 71 Freckling (axillary or inguinal), 47
Enterovirus, 77 Friedreichs ataxia, 63
Index 223

Fucosidosis, 57 Granuloma, 6669, 71, 72, 77, 87, 137 (Fig.),


Fungal myositis, 78 162 (Fig.)
Fusiform aneurysm, 10 Granulovacuolar degeneration, 59, 125 (Fig.)
Grenkers myelinopathy, 9
Grouped atrophy, 4
G GuillainBarre syndrome, 86
Galactocerebroside-B galactosidase, 53 Gummata, 67
Galactosialidoses, 57 Gunshot wound, 38
Gangliocytoma, 21
Ganglioglioma, 21, 43
Ganglioneuroblastoma, 27 H
Ganglionic hemorrhage, 10 HallervordenSpatz disease, 62
Gaucher cell, 56 HAM 56, 3
Gauchers disease, 56 Hamartoma, 48
Gemistocytes, 2 HandSchllerChristian disease, 29
Gemistocytic astrocytoma, 16 H band, 4
Germinal matrix hemorrhage, 46, 110 (Fig.) Heliotropic rash, 77
Germinoma, 31, 95 (Fig.), 137 (Fig.) Hemangioblastoma, 26, 48
GerstmannStrausslerScheinker disease, 72 Hemangiopericytoma, 25
Giant aneurysm, 9 Hemophagocytic lymphohistiocytosis, 29
Giant cell arteritis, 11 Hemorrhage, 10, 134 (Fig.)
Gitter cell, 3 Hemorrhagic infarct, 8
Glial fibrillary acidic protein (GFAP), 2, 15, Heparin N-sulfatase, 56
1724, 26, 27, 167 (Fig.) Hepatic encephalopathy, 58, 155 (Fig.)
Glial heterotopia, 20 Hereditary motor and sensory neuropathies,
Gliding contusion, 38 8788
Glioblastoma multiforme, 1617, 19, 93 (Fig.), Hereditary sensory and autonomic
116 (Fig.) neuropathies, 88
Gliofibroma, 20 Herniation, 35
Gliomatosis cerebri, 18 Herniation contusion, 38
Gliosarcoma, 18, 113 (Fig.) Herpes simplex, 70
Gliosis, 16 Herpes zoster, 70
Globoid cell leukodystrophy, 53, 88, 153 (Fig.) Heubners endarteritis obliterans, 67
Globose cells, 60 Hexosaminidase, 55
Glucocerebrosidase, 56 H. influenza, 66
Glycogenoses, 80 Hinge fracture, 37
GM1 gangliosidosis, 55 Hippocampal sclerosis, 45, 166 (Fig.)
GM2 gangliosidosis, 55 Hippocampus (architecture), 3
Gomori methenamine silver (GMS), 68, 69 Hirano body, 59
Gorlen syndrome, 49 Histoplasmosis, 69
Gottrons papules, 77 Histotoxic anoxia, 7
Gram negative bacilli, 66 HIV, 71, 77, 87
Granular bodies, 18, 21 Hodgkins lymphoma, 29
Granular ependymitis, 3 Holoprosencephaly, 42, 45
Granular layer (cerebellum), 3, 142 (Fig.) Holotelencephaly 42
Granular layer (cortex), 3 Homer Wright rosette, 23, 27, 100 (Fig.)
224 Index

HTLV-1, 71 Kuru plaque, 72, 124 (Fig.)


Hunter disease, 56
Huntington, 61
Huntingtons chorea, 61 L
Hurler-Scheie disease, 56 Laceration, 38
Hyaline inclusion, 1, 63 Lacunar infarct, 8
Hydranencepaly, 46 Lafora body, 57
Hydromyelia, 42, 114 (Fig.) Lafora body disease, 57
Hyperostosis, 24 LambertEaton syndrome, 79
Hypertensive angiopathy, 12 Langerhans cell histiocytosis, 29, 138 (Fig.)
Hyperthryoid myopathy, 82 Lateral transtentorial herniation, 35
Hypertrophy (muscle), 4 Lead, 58
Hyphae, 68 Leighs disease, 57, 80
Hypoglycemia, 9 Leiomyoma, 25
Lepromatous leprosy, 87
Leprosy, 87
I&J Leptomeningitis, 6569, 71, 129 (Fig.), 132 (Fig.)
I band, 5 LettererSiwe disease, 29
Inclusion body myositis, 77, 172 (Fig.), 175 (Fig.) Leu-7, 21
Iduronate sulfatase, 56 Lewy body, 59, 61
Infarct (infarction), 8, 120 (Fig.), 147 (Fig.), LhermitteDuclos, 21, 49
154 (Fig.), 157 (Fig.), 159 (Fig.), 160 LiFraumeni syndrome, 26, 49
(Fig.), 168 (Fig.) Limb girdle dystrophy, 83
Infectious aneurysm, 9 Line of Baillarger, 3
Influenza, 70, 77 Linear fracture (skull), 37
Intrafusal fibers, 5 Lipid storage myopathy, 81
Iodamoeba butschlii, 71 Lipofuscin, 1, 59, 130 (Fig.)
Ioxodid tick, 67 Lipoma, 32
Ischemia (see anoxia) LIS-1 gene, 43
Ischemic exercise test , 81 Lisch nodule, 47
JC virus, 71 Lissauers dementia, 67
Joubert syndrome, 45 Lissencephaly, 43, 96 (Fig.)
Listeria, 66
Lobulated fiber, 83
K&L Low grade astrocytoma, 15
KearnsSayre syndrome, 80 Lyme disease, 6768, 87
Kernicterus, 47 Lymphoma, 29
Kernohans notch, 35 Lymphoplasmocyte rich meningioma, 24
Ki-67, 16 Lymphorrhages, 79
KlippelFeil abnormality, 43 Lyssa body, 70
Khachaturian criteria (Alzheimers disease), 60
Korsakoff syndrome, 58
Krabbes disease, 53, 153 (Fig.) M
Kufs disease, 56 Mad cow disease, 72
KugelbergWelander disease, 63, 78 Malaria, 7172, 145 (Fig.)
Kuru, 72 Malignant peripheral nerve sheath tumor, 28
Index 225

Malignant hypertension, 12 MIB-1, 16


Malignant hyperthermia, 79, 81 Microcephalus vera, 43
Malignant meningioma, 2425 Microcephaly, 44, 45, 56, 58, 70
Malignant mixed glioma, 22 Microcystic meningioma, 24
MALT lymphoma of dura, 29 Microencephaly, 44
Manganese, 58 Microglia, 3, 69, 121 (Fig.)
Mannosidase, 57 Microglial nodule, 3, 69, 71, 72, 121 (Fig.)
Mannosidoses, 57 Microgliosis, 3
Maple syrup urine disease, 58 Micropolygyria, 44
Marburg multiple sclerosis, 52 MillerDieker syndrome, 43
Marchiafava-Bignami disease, 53, 58 Mitochondrial myopathy, 80, 176 (Fig.), 177 (Fig.)
Marinesco bodies, 1 Mixed glioma,22
MaroteauxLamy disease, 56 Mbius syndrome, 47
Mast cells, 5 Molecular layer (cerebellum), 3
M band, 4 Molecular layer (cortex), 3
McArdles disease, 80 Morquioss disease, 56
Measles, 70 Mortons neuroma, 29, 86
Meckel Gruber syndrome, 41 Motheaten fibers, 77, 80, 83
Medulloblastoma, 27, 49, 100 (Figs.) Motor unit, 5
Medullomyoblastoma , 27 Moyamoya syndrome, 12
Megalocephaly, 44 MPTP-A, 61
Megalencephaly, 44, 54 Mucicarmine, 24, 68
Melanin (neuromelanin), 1, 143 (Fig.), 150 (Fig.) Mucolipidoses, 57
Melanocyte, 3 Mucopolysaccharidoses, 56
Melanocytoma, 25 Mucormycosis, 68, 123 (Fig.)
Melanoma, 26, 107 (Fig.) Multi-infarct dementia, 60
Melanotic medulloblastoma, 27 Multilocular cystic encephalomalacia, 46
Melanotic schwannoma, 28 Multiple sclerosis, 5152, 117 (Fig.), 119
MELAS, 57, 80 (Fig.), 126 (Fig.), 147 (Fig.)
Meningioma, 2425, 92 (Fig.), 96 (Fig.), 97 Multisystem atrophy syndrome, 62
(Fig.), 152 (Fig.) Mumps, 70
Meningitis (see leptomeningitis) Muscle spindle, 5, 169 (Fig.)
Meningoangiomatosis, 97 (Fig.) Muscular dystrophy, 82
Meningocele, 41 Myasthenia gravis, 7879, 82
Meningomyelocele, 41 Mycobacterium avium intracellulare, 67
Meningothelial meningioma, 24, 96 (Fig.) Mycobacterium leprae, 87
Menkes disease, 58 Mycotic aneurysm, 10
Merlin, 47 Myelin, 5, 51
Merosin, 83 Myelin ovoids, 85
MERRF, 57, 80 Myelin protein zero, 88
Mesial temporal sclerosis, 45 Myelinated axons, 6
Metachromasia, 53 Myelocystocele, 41
Metachromatic leukodystrophy, 53, 88 Myeloschisis, 41
Metaplastic meningioma, 24 Myoadenylate deaminase, 81
Metastatic disease, 17, 3132 Myoepithelioma, 26
Methanol, 58, 153 (Fig.) Myophosphorylase, 8081
226 Index

Myosin, 4 Notch 3 gene, 12


Myotonia congenita, 81 Nuclear bags, 78
Myotonic dystrophy, 83
Myotonin protein kinase, 83 OP
Myotubular myopathy, 79 Oculopharyngeal dystrophy, 83
Myxopapillary ependymoma, 23, 116 (Fig.), Oil-red-O, 75, 81
118 (Fig.) Oligoastrocytoma, 22
Oligodendrocyte, 2
N Oligodendroglioma, 21, 104 (Fig.), 115 (Fig.)
N-acetylgalactosaminase-4-sulfatase, 56 Olivopontocerebellar atrophy, 62
N-acetylglucosamine-1-phosphotransferase, 57 Onion bulb, 86-88, 179 (Fig.)
NADH, 76, 78, 79, 81 Optic nerve glioma, 20, 47
Naegleria, 71 P16, 16
Nasal glioma, 20 P19, 16
Negri body, 70, 131 (Fig.) P53 (see also TP53), 16, 17, 49
Neisseria meningitidis, 66 Pachygyria, 43
Nemaline myopathy, 79 Pachymeningitis cervicalis hypertrophicans, 67
Nemaline rods, 79, 80 Papillary glioneuronal tumor, 22
NeuLaxova syndrome, 44 Papillary meningioma, 25
Neuraminidase, 56 Papova virus, 71
Neuritic plaques, 59, 60, 127 (Fig.) Paracrystalline inclusions (mitochondria), 80,
Neurocutaneous melanosis syndrome, 26, 49 176 (Fig.)
Neurofibrillary tangle , 58, 5962, 70, 131 (Fig.) Paraganglioma, 30, 103 (Fig.)
Neurofibroma,28, 47 Paraneoplastic neuropathy, 87
Neurofibromatosis type I, 18, 20, 28, 43, 47 Parasitic myositis, 77
Neurofibromatosis type II, 24, 28, 47-48, 108 (Fig.) Parkinsons disease, 61
Neurofibromin, 47 Parkinsonism-dementia complex of Guam, 62
Neurofilament protein, 27, 30, 31 Pars intermedia, 4
Neurofilaments, 1 PAS, 24, 53, 54, 57, 67, 68, 69, 72, 75, 80, 81
Neurohypophysis, 4, 141 (Fig.) Patau syndrome, 45
Neurokeratin, 85 PDGFR alpha, 16
Neuroma, 28 PelizaeusMerzbacher disease, 54
Neuromuscular junction, 5 Pencil bundles of Wilson, 3, 141 (Fig.)
Neuron, 1, 133 (Fig.), 148 (Fig.) Penetrating wound, 38
Neuron specific enolase, 31 Perforating wound, 38
Neuronal heterotopia, 43 Periaxonal space of Klebs, 6
Neuronophagia, 2 Perifascicular atrophy, 4, 77, 171 (Fig.)
Neuropil threads, 62 Perimysium, 4
Nevus flammeus, 48 Perinatal telencephalic leukoencephalopathy, 46
Niemann-Pick cell, 56 Perineurioma, 28
Niemann-Pick disease, 5556 Perineurium, 5
Nissl substance, 1 Periodic paralysis, 81, 82
Nocardiosis, 69 Peripheral myelin protein, 88
Node of Ranvier, 6 Peripheral nerve (histology), 56
Normal pressure hydrocephalus, 59 Periungual fibroma, 48
Index 227

Perivascular pseudorosette, 22, 23, 127 (Fig.) Primitive neuroectodermal tumor, 27


Periventricular leukomalacia, 46, 47 Prion protein, 72
Peroxisome, 44 Prism stacks, 88
Phenylketonuria, 58 Progesterone receptors, 24
Phospholipid protein, 54 Progressive multifocal leukoencephalopathy,
Physaliphorous cells, 25 71, 122 (Fig.), 129 (Fig.)
Phytanoyl CoA alpha hydroxylase, 88 Progressive supranuclear palsy, 62
Pick bodies, 61 Prolactinoma, 31, 109 (Fig.)
Pick cells, 60 Proteinase, 3
Picks disease, 6061 Proteus, 66
Pi granules of Reich, 6, 178 (Fig.) Protoplasmic astrocyte, 2
Pilocytic astrocyte, 2 Protoplasmic astrocytoma, 19
Pilocytic astrocytoma, 18 Protoplasmic body, 68
Pineal cyst, 4 Psammoma body, 3, 24, 28, 97 (Fig.)
Pineal gland (architecture), 4, 120 (Fig.) Psammomatous meningioma, 24, 97 (Fig.)
Pineoblastoma, 49 Pseudohyphae, 68, 69
Pineocytoma, 3031 PTEN mutation, 21
Pineocytomatous rosettes, 31 Purkinje cell layer (cerebellum), 3, 149 (Fig.)
Pituitary adenoma, 31, 109 (Fig.) Pyramidal layer (cortex), 3
Pituitary carcinoma, 31
Pituitary gland (architecture), 4, 141 (Fig.), 144 (Fig.)
Plaque (demyelination), 52 QR
Plasmodium falciparum, 71 Rabies, 70, 130 (Fig.)
Platybasia, 43 Raccoon eyes, 3738
Pleomophoric xanthoastrocytoma, 19, 112 (Fig.) Radiation, 17
Plexiform neurofibroma, 28, 47, 91 (Fig.) Ragged red fibers, 80
Plexiform schwannoma, 28 Rathkes cleft cyst, 30
Polio, 69 RB 1 gene, 16, 49
Polyarteritis nodosa, 11, 78, 170 (Fig.) Reactive astrocyte, 2, 16, 139 (Fig.)
Polycythemia vera, 26 Reduviid bug, 72
Polyglucosan, 2, 57 Reed Sternberg cell, 29
Polymicrogyria, 44, 109 (Fig.) Refsums disease, 88
Polymorphic layer (cortex), 3 Renaut bodies, 5
Polymyositis, 76, 170 (Fig.) Resistance sector, 3
Pompes disease, 80 Respirator brain, 8
Pontoneocerebellar hypoplasia, 45 Reticulin, 19, 25
Ponto-subicular necrosis, 47 Retinoblastoma syndrome, 49
Porencephaly, 46, 101 (Fig.) Rhabdoid meningioma, 25
Porphyria, 88 Rhabdoid tumor, 27
Postencephalitic Parkinsons disease, 6162 Rhabdomyolysis, 81
Potts disease, 66 Rheumatoid arthritis, 77
Prealbumin, 89 Rhizomucor, 68
Presenilin-1, 59, 60 Rhizopus, 68
Presenilin-2, 59, 60 RileyDay syndrome, 88
Primary angiitis of the central nervous system, 11 Rimmed vacuole 77,172 (Fig.)
Ring fibers, 77, 83, 173 (Fig.)
228 Index

RosaiDorfman disease, 29 Sommer sector, 4, 7, 119 (Fig.)


Rosenthal fiber, 2, 4, 16, 18, 20, 31, 54, 57, 94 Sphenoid dysplasia, 47
(Fig.), 117 (Fig.) Spheroids, 46
Rosettes, 22 Sphingomyelinase, 55
Ryanodine receptor-1 gene, 79 Spina bifida, 41
Spinal cord (architecture), 4
Spinal muscular atrophy, 6364, 78, 183 (Fig.)
S Stagnant anoxia, 7, 8
S-100 protein, 2, 6, 17, 20, 21, 24, 25, 26, 28-30 Staphylococcus, 10, 65, 66, 77
Saccular aneurysm, 9, 154 (Fig.) Static loading, 38
Saltatory conduction, 6 Status marmoratus, 46
Sandoff disease, 55 Steiner, 67
Sanfilippo disease, 56 Steroid myopathy, 82
Sarcocystis, 161 (Fig.) Streptococcus, 10, 65, 66
Sarcoglycan, 83 Stria of Gennari, 3
Sarcoidosis, 67, 89 Striatonigral degeneration, 62
Sarcoma, 25 Stroke, 8
Sarcomere, 4 Strongyloides, 161 (Fig.)
Sarcoplasm, 5 SturgeWeber disease, 48, 135 (Fig.)
Sarcoplasmic reticulum, 5 Subacute combined degeneration, 58, 165 (Fig.)
Schilder multiple sclerosis, 52 Subacute sclerosing panencephalitis, 70, 151 (Fig.)
Schistosomiasis, 72 Subarachnoid hemorrhage, 37, 46
Schizencephaly, 44 Subdural abscess, 65
SchmidtLanterman clefts, 6 Subdural hematoma, 36, 46, 158 (Fig.)
Schwann cell, 6, 179 (Fig.) Subdural hygroma, 47
Schwannoma, 28, 48, 99 (Fig.), 108 (Fig.) Subependymal giant cell astrocytoma, 19, 48,
SDH (succinate dehydrogenase), 76 93 (Fig.), 103 (Fig.)
Secondary structures of Scherer, 16, 28 Subependymoma, 23
Secretory meningioma, 24 Subfalcial herniation, 36
Segmental demyelination, 86-89 Sudan (black), 53, 54, 81
Selective vulnerability, 7 Sulfonated alcian blue, 75
Septic aneurysm, 9 Sulfur granules, 69
Septic embolism, 66, 68 Survival motor neuron gene, 78
Septic thrombosis, 8 Sustentacular cells, 30
Shadow plaque, 52 SV40 virus, 26, 71
Shagreen patch, 48 Synaptophysin, 1, 22, 27, 30, 31
Shingles, 70 Synovial cyst, 30
Sialodosis, 5657 Syphilis, 67
Silver staining, 5961 Syringobulbia, 42
Skein, 63 Syringomyelia, 4
Skeletal muscle (histology), 45
Shy disease, 56
ShyDrager syndrome, 62 T
Smooth muscle actin, 27 Tabes dorsalis, 67
SOD1, 63 Taenia solium, 72, 139 (Fig.)
Solitary fibrous tumor, 25 Takayasus arteritis, 11
Index 229

Target fibers, 78, 182 (Fig.) Tumor-like demyelinating lesion, 52


Targetoid fibers, 78 Turcot syndrome, 49
Tau protein, 59, 61, 62 Type grouping, 78
TaySachs disease, 55, 101 (Fig.), 149 (Fig.) Type I atrophy, 4
T.brucei, 72 Type II atrophy, 4
T.cruzi, 72
Thiamine, 58, 88, 165 (Fig.)
Thioflavin S/T, 10, 60 UV
Thymic hyperplasia, 78 Ubiquitin, 6163
Thymoma, 78 Ulegyria, 46
Tigroid demyelination, 54 Uncal herniation (see lateral transtentorial
Tonsillar herniation, 36 herniation)
Torpedoes, 2, 72 Unmyelinated axons, 6, 178 (Fig.)
Toruloma, 68 Upward herniation, 36
Touraine syndrome, 49 Uremic neuropathy, 89
Toxoplasma gondii, 71, 121 (Fig.) Vacuolar myelopathy, 71
Toxoplasmosis, 71, 121 (Fig.) Vacuolar myopathy, 81
TP53 mutations, 16, 21 Varicella zoster, 87
Transient ischemic attack, 8 Varix, 10
Transitional meningioma, 24 Vascular malformation, 10
Transthyretin, 89 Vascular proliferation, 1518, 22, 140 (Fig.)
Traumatic neuroma, 86, 95 (Fig.) Vasculitic neuropathy, 89
Traumatic neuropathy, 86 Vasculitis, 11, 77, 78, 160 (Fig.), 167 (Fig.),
Treponema pallidum, 67 170 (Fig.)
Trichinella spiralis, 77 Vasogenic edema, 35
Trichinosis, 77 Venous malformation, 10, 155 (Fig.)
Trichrome (Gomori), 75, 77, 79, 80 Venous thrombosis, 8
Trinucleotide repeat disorder, 61, 63, 83 Ventriculitis, 66, 123 (Fig.)
Triton tumor, 28 Verocay bodies, 28, 99 (Fig.)
Tropheryma whippelli, 67 Vimentin, 17, 2427
Tropical spastic paraparesis, 71 Viral myositis, 77
Trypanosomiasis, 72 Vitamin B12, 58, 89, 165 (Fig.)
TSC1 gene, 48 Vitamin E, 89
TSC2 gene, 48 von HippelLindau disease, 26, 48
Tsetse fly, 72 Vonsattel grading (Huntingtons chorea), 61
T system, 5
Tuber, 48, 164 (Fig.) W
Tuberculoid leprosy, 87 WagnerMeissner-like corpuscles, 28
Tuberculoma, 66 WalkerWarburg syndrome, 44
Tuberculosis, 6667 Wallerian degeneration, 8586
Tuberous sclerosis, 19, 43, 48 WarthinStarry, 67
Tubular aggregates, 81 WaterhouseFriderichson, 66
Tubulofilaments (inclusion body myositis), 77, Wegeners granulomatosis, 11
175 (Fig.) WerdnigHoffmann disease, 6364, 78, 169 (Fig.)
Tuffstone bodies, 88 Wernicke encephalopathy, 58, 165 (Fig.)
230 Index

WestonHurst disease, 52 Zebra bodies, 55, 56, 88, 101 (Fig.)


Whipples disease, 67 Zellballen, 30, 103 (Fig.)
White matter (architecture), 3, 142 (Fig.) Zellwegers syndrome, 44
Whorled fibers, 83 ZiehlNeelsen, 67

XYZ
Z band, 5, 175 (Fig.)
Neuropathology Review
By

Richard A. Prayson, MD
Department of Anatomic Pathology
Cleveland Clinic Foundation, Cleveland, OH

As scientific research expands the scope of neuropathology, the amount of information that both student and practicing
pathologists must assimilate and master can be challenging, and at times even daunting. In Neuropathology Review, Richard
A. Prayson, MD, thoroughly summarizes in outline form the essential elements of the neuropathology curriculum for all
those seeking a concise up-to-date overview of the subject. Topics range from basic information on the spectrum of
neurologic-related disease to pertinent pictorial examples of many neuropathologic conditions. For each topic there are
self-assessment tests based on both textual and pictorial material, along with sample answers and explanations designed
to aid comprehension and understanding.
Concise yet comprehensive, Neuropathology Review provides an effective and easy-to-use refresher course for those
working pathologists wishing to update their skills, as well as for students and residents preparing to take board exams in
neuropathology.

Concise yet comprehensive review of the essentials of neuropathology


Ideal for board preparation
Pictorial tests complete with sample answers
Self-assessment questions provided with sample answers and explanations
Designed for active pathologists as well as residents in training in neurology,
neurosurgery, and pathology

CONTENTS
Normal Histology. Vascular Lesions. Tumors. Trauma. Congenital Malformations, Perinatal Disease, and Phacomatoses.
Demyelinating and Dysmyelinating Diseases. Metabolic and Toxic Diseases. Degenerative Diseases. CNS Infection.
Skeletal Muscle. Peripheral Nerve. Figures with Questions. Answers to Figures with Questions. Written Self-Assessment
Questions. Answers to Written Self-Assessment Questions. Bibliography. Index.

90000

NEUROPATHOLOGY REVIEW
ISBN: 1-58829-024-7
http://humanapress.com 9 781588 290243

Das könnte Ihnen auch gefallen